diff --git a/TST/DM/2102_DM2/TST3/01_2102_DM2.tex b/TST/DM/2102_DM2/TST3/01_2102_DM2.tex new file mode 100644 index 0000000..eda711a --- /dev/null +++ b/TST/DM/2102_DM2/TST3/01_2102_DM2.tex @@ -0,0 +1,203 @@ +\documentclass[a5paper,10pt]{article} +\usepackage{myXsim} +\usepackage{tasks} + +% Title Page +\title{DM2 \hfill ASAIDI Sophian} +\tribe{TST} +\date{\hfillÀ render pour le Mercredi 24 février} + +\xsimsetup{ + solution/print = false +} + +\begin{document} +\maketitle + +\begin{exercise}[subtitle={Loi binomiale}] + Trois personnes s'apprêtent à passer le portique de sécurité. On suppose que pour chaque personne la probabilité que le portique sonne est égale à $0.19$. + + Soit $X$ la variable aléatoire donnant le nombre de personnes faisant sonner le portique, parmi les 3 personnes de ce groupe. + \begin{enumerate} + \item Tracer l'arbre représentant le situation. + \item Justifier que $X$ suit une loi binomiale dont on précisera les paramètres. + \item Quelle est la probabilité qu'une seule personne fasse sonner le portique? + \item Calculer puis interpréter les probabilités suivantes + \[ + P(X = 0) \qquad \qquad P(X \geq 2) + \] + \item Calculer l'espérance de $X$ et interpréter le résultat. + \end{enumerate} +\end{exercise} + +\begin{solution} + \begin{enumerate} + \item + \begin{tikzpicture}[sloped] + \node {.} + child {node {$0$} + child {node {$0$} + child {node {$0$} + edge from parent + node[above] {0.81} + } + child {node {$1$} + edge from parent + node[above] {0.19} + } + edge from parent + node[above] {0.81} + } + child[missing] {} + child {node {$1$} + child {node {$0$} + edge from parent + node[above] {0.81} + } + child {node {$1$} + edge from parent + node[above] {0.19} + } + edge from parent + node[above] {0.81} + } + edge from parent + node[above] {0.81} + } + child[missing] {} + child[missing] {} + child[missing] {} + child { node {$1$} + child {node {$0$} + child {node {$0$} + edge from parent + node[above] {0.81} + } + child {node {$1$} + edge from parent + node[above] {0.19} + } + edge from parent + node[above] {0.81} + } + child[missing] {} + child {node {$1$} + child {node {$0$} + edge from parent + node[above] {0.81} + } + child {node {$1$} + edge from parent + node[above] {0.19} + } + edge from parent + node[above] {0.81} + } + edge from parent + node[above] {0.19} + } ; + \end{tikzpicture} + \item Chaque personne a 2 possibilités (1: fait sonner ou 2: ne fait pas sonner) et l'on fait passer 3 personnes ce qui correspond à une répétition identique et aléatoire. On peut donc modéliser la situation par une loi binomiale. + \[ + X \sim \mathcal{B}(3; 0.76) + \] + \item Probabilité qu'une seule personne fasse sonner le portique. On voit qu'il y a 3 branches qui correspondent à cette situation dont + \[ + P(X = 1) = 3 \times 0.19^1 \times 0.81^2 \approx 0.374 + \] + \item + \[ + P(X = 0) = 0.81^3 \approx 0.531 + \] + \[ + P(X \geq 2) = P(X = 2) + P(X = 3) = 3 \times 0.19^2 \times 0.81^1 + 0.19^3 \approx 0.095 + \] + + \item Il faut d'abord tracer le tableau résumant la loi de probabilité: + \begin{center} + \begin{tabular}{|c|*{4}{c|}} + \hline + Valeur & 0 & 1 & 2 & 3 \\ + \hline + Probabilité & $0.531$ & $0.374$ & $0.088$ &$0.007$ \\ + \hline + \end{tabular} + \end{center} + On peut alors calculer l'espérance + \[ + E[X] = 0 \times 0.531 + 1 \times 0.374 + 2 \times 0.088 + 3 \times 0.007 = 0.57 + \] + On peut donc estimer qu'il y aura en moyenne $0.57$ personnes qui feront sonner le portique sur les 3 personnes. + \end{enumerate} +\end{solution} + +\begin{exercise}[subtitle={Équation puissance}] + Résoudre les équations et inéquations suivantes + \begin{multicols}{2} + \begin{enumerate} + \item $10^x = 7$ + \item $3^x = 35$ + \item $0.8^x \leq 2$ + \item $4 \times 0.06^x = 49$ + \end{enumerate} + \end{multicols} +\end{exercise} + +\begin{solution} + Les solutions ci-dessous ne sont pas justifiée car l'ordinateur ne sait pas faire. Par contre, vous vous devez savoir justifier vos réponses! + \begin{enumerate} + \item $x = \log(7)$ + \item $x = \frac{\log(35)}{\log(3)}$ + \item Il faut faire attention quand on divise par un log car ce dernier peut être négatif ce qui est le cas ici. Il faut donc pense à changer le sens de l'inégalité. + + $x \geq \frac{\log(2)}{\log(0.8)}$ + + \item Il faut penser à faire la division à par $4$ avant d'utiliser le log car sinon, on ne peut pas utiliser la formule $\log(a^n) = n\times \log(a)$. + + $x = \frac{\log(12.25)}{\log(0.06)}$ + \end{enumerate} +\end{solution} + +\begin{exercise}[subtitle={Étude de fonctions}] + Soit $f(x) = 5x^3 - 202.5x^2 - 2970x + 24$ une fonction définie sur $\R$. + \begin{enumerate} + \item Calculer $f'(x)$ la dérivée de $f(x)$. + \item Calculer $f'(33)$ et $f'(-6)$. + \item En déduire une forme factorisée de $f'(x)$. + \item Étudier le signe de $f'(x)$ et en déduire les variations de $f(x)$. + \item Est-ce que la fonction $f(x)$ admet un maximum ou un minimum? Si oui, calculer sa valeur. + \end{enumerate} +\end{exercise} + +\begin{solution} + \begin{enumerate} + \item Dérivée de $f(x)$: $f'(x) = 15x^2 - 405x - 2970$ + \item + \begin{align*} + f'(33) &= 15 \times 33^{2} - 405 \times 33 - 2970\\&= 15 \times 1089 - 13365 - 2970\\&= 16335 - 16335\\&= 0 + \end{align*} + \begin{align*} + f'(-6) &= 15 \times - 6^{2} - 405(- 6) - 2970\\&= 15 \times 36 + 2430 - 2970\\&= 540 - 540\\&= 0 + \end{align*} + Donc $x = 33$ et $x=-6$ sont des racines de $f'(x) = 15x^2 - 405x - 2970$. + \item On en déduit la forme factorisée suivante + \[ + f'(x) = 15 (x - 33)(x--6) + \] + \item Pas de correction disponible + \item À causes des branches extérieurs, la fonction $f(x)$ n'a pas de maximum ou de minimum. + \end{enumerate} +\end{solution} + + + +%\printsolutionstype{exercise} + + + +\end{document} + +%%% Local Variables: +%%% mode: latex +%%% TeX-master: "master" +%%% End: diff --git a/TST/DM/2102_DM2/TST3/02_2102_DM2.tex b/TST/DM/2102_DM2/TST3/02_2102_DM2.tex new file mode 100644 index 0000000..e8cabfe --- /dev/null +++ b/TST/DM/2102_DM2/TST3/02_2102_DM2.tex @@ -0,0 +1,203 @@ +\documentclass[a5paper,10pt]{article} +\usepackage{myXsim} +\usepackage{tasks} + +% Title Page +\title{DM2 \hfill BELARBI Samira} +\tribe{TST} +\date{\hfillÀ render pour le Mercredi 24 février} + +\xsimsetup{ + solution/print = false +} + +\begin{document} +\maketitle + +\begin{exercise}[subtitle={Loi binomiale}] + Trois personnes s'apprêtent à passer le portique de sécurité. On suppose que pour chaque personne la probabilité que le portique sonne est égale à $0.58$. + + Soit $X$ la variable aléatoire donnant le nombre de personnes faisant sonner le portique, parmi les 3 personnes de ce groupe. + \begin{enumerate} + \item Tracer l'arbre représentant le situation. + \item Justifier que $X$ suit une loi binomiale dont on précisera les paramètres. + \item Quelle est la probabilité qu'une seule personne fasse sonner le portique? + \item Calculer puis interpréter les probabilités suivantes + \[ + P(X = 0) \qquad \qquad P(X \geq 2) + \] + \item Calculer l'espérance de $X$ et interpréter le résultat. + \end{enumerate} +\end{exercise} + +\begin{solution} + \begin{enumerate} + \item + \begin{tikzpicture}[sloped] + \node {.} + child {node {$0$} + child {node {$0$} + child {node {$0$} + edge from parent + node[above] {0.42} + } + child {node {$1$} + edge from parent + node[above] {0.58} + } + edge from parent + node[above] {0.42} + } + child[missing] {} + child {node {$1$} + child {node {$0$} + edge from parent + node[above] {0.42} + } + child {node {$1$} + edge from parent + node[above] {0.58} + } + edge from parent + node[above] {0.42} + } + edge from parent + node[above] {0.42} + } + child[missing] {} + child[missing] {} + child[missing] {} + child { node {$1$} + child {node {$0$} + child {node {$0$} + edge from parent + node[above] {0.42} + } + child {node {$1$} + edge from parent + node[above] {0.58} + } + edge from parent + node[above] {0.42} + } + child[missing] {} + child {node {$1$} + child {node {$0$} + edge from parent + node[above] {0.42} + } + child {node {$1$} + edge from parent + node[above] {0.58} + } + edge from parent + node[above] {0.42} + } + edge from parent + node[above] {0.58} + } ; + \end{tikzpicture} + \item Chaque personne a 2 possibilités (1: fait sonner ou 2: ne fait pas sonner) et l'on fait passer 3 personnes ce qui correspond à une répétition identique et aléatoire. On peut donc modéliser la situation par une loi binomiale. + \[ + X \sim \mathcal{B}(3; 0.76) + \] + \item Probabilité qu'une seule personne fasse sonner le portique. On voit qu'il y a 3 branches qui correspondent à cette situation dont + \[ + P(X = 1) = 3 \times 0.58^1 \times 0.42^2 \approx 0.307 + \] + \item + \[ + P(X = 0) = 0.42^3 \approx 0.074 + \] + \[ + P(X \geq 2) = P(X = 2) + P(X = 3) = 3 \times 0.58^2 \times 0.42^1 + 0.58^3 \approx 0.619 + \] + + \item Il faut d'abord tracer le tableau résumant la loi de probabilité: + \begin{center} + \begin{tabular}{|c|*{4}{c|}} + \hline + Valeur & 0 & 1 & 2 & 3 \\ + \hline + Probabilité & $0.074$ & $0.307$ & $0.424$ &$0.195$ \\ + \hline + \end{tabular} + \end{center} + On peut alors calculer l'espérance + \[ + E[X] = 0 \times 0.074 + 1 \times 0.307 + 2 \times 0.424 + 3 \times 0.195 = 1.74 + \] + On peut donc estimer qu'il y aura en moyenne $1.74$ personnes qui feront sonner le portique sur les 3 personnes. + \end{enumerate} +\end{solution} + +\begin{exercise}[subtitle={Équation puissance}] + Résoudre les équations et inéquations suivantes + \begin{multicols}{2} + \begin{enumerate} + \item $10^x = 6$ + \item $10^x = 31$ + \item $0.32^x \leq 15$ + \item $5 \times 0.06^x = 9$ + \end{enumerate} + \end{multicols} +\end{exercise} + +\begin{solution} + Les solutions ci-dessous ne sont pas justifiée car l'ordinateur ne sait pas faire. Par contre, vous vous devez savoir justifier vos réponses! + \begin{enumerate} + \item $x = \log(6)$ + \item $x = \frac{\log(31)}{\log(10)}$ + \item Il faut faire attention quand on divise par un log car ce dernier peut être négatif ce qui est le cas ici. Il faut donc pense à changer le sens de l'inégalité. + + $x \geq \frac{\log(15)}{\log(0.32)}$ + + \item Il faut penser à faire la division à par $5$ avant d'utiliser le log car sinon, on ne peut pas utiliser la formule $\log(a^n) = n\times \log(a)$. + + $x = \frac{\log(1.8)}{\log(0.06)}$ + \end{enumerate} +\end{solution} + +\begin{exercise}[subtitle={Étude de fonctions}] + Soit $f(x) = - 4x^3 + 126x^2 + 2784x - 17$ une fonction définie sur $\R$. + \begin{enumerate} + \item Calculer $f'(x)$ la dérivée de $f(x)$. + \item Calculer $f'(29)$ et $f'(-8)$. + \item En déduire une forme factorisée de $f'(x)$. + \item Étudier le signe de $f'(x)$ et en déduire les variations de $f(x)$. + \item Est-ce que la fonction $f(x)$ admet un maximum ou un minimum? Si oui, calculer sa valeur. + \end{enumerate} +\end{exercise} + +\begin{solution} + \begin{enumerate} + \item Dérivée de $f(x)$: $f'(x) = - 12x^2 + 252x + 2784$ + \item + \begin{align*} + f'(29) &= - 12 \times 29^{2} + 252 \times 29 + 2784\\&= - 12 \times 841 + 7308 + 2784\\&= - 10092 + 10092\\&= 0 + \end{align*} + \begin{align*} + f'(-8) &= - 12 \times - 8^{2} + 252(- 8) + 2784\\&= - 12 \times 64 - 2016 + 2784\\&= - 768 + 768\\&= 0 + \end{align*} + Donc $x = 29$ et $x=-8$ sont des racines de $f'(x) = - 12x^2 + 252x + 2784$. + \item On en déduit la forme factorisée suivante + \[ + f'(x) = -12 (x - 29)(x--8) + \] + \item Pas de correction disponible + \item À causes des branches extérieurs, la fonction $f(x)$ n'a pas de maximum ou de minimum. + \end{enumerate} +\end{solution} + + + +%\printsolutionstype{exercise} + + + +\end{document} + +%%% Local Variables: +%%% mode: latex +%%% TeX-master: "master" +%%% End: diff --git a/TST/DM/2102_DM2/TST3/03_2102_DM2.tex b/TST/DM/2102_DM2/TST3/03_2102_DM2.tex new file mode 100644 index 0000000..aa2b2ad --- /dev/null +++ b/TST/DM/2102_DM2/TST3/03_2102_DM2.tex @@ -0,0 +1,203 @@ +\documentclass[a5paper,10pt]{article} +\usepackage{myXsim} +\usepackage{tasks} + +% Title Page +\title{DM2 \hfill BERTAN Ufuk} +\tribe{TST} +\date{\hfillÀ render pour le Mercredi 24 février} + +\xsimsetup{ + solution/print = false +} + +\begin{document} +\maketitle + +\begin{exercise}[subtitle={Loi binomiale}] + Trois personnes s'apprêtent à passer le portique de sécurité. On suppose que pour chaque personne la probabilité que le portique sonne est égale à $0.04$. + + Soit $X$ la variable aléatoire donnant le nombre de personnes faisant sonner le portique, parmi les 3 personnes de ce groupe. + \begin{enumerate} + \item Tracer l'arbre représentant le situation. + \item Justifier que $X$ suit une loi binomiale dont on précisera les paramètres. + \item Quelle est la probabilité qu'une seule personne fasse sonner le portique? + \item Calculer puis interpréter les probabilités suivantes + \[ + P(X = 0) \qquad \qquad P(X \geq 2) + \] + \item Calculer l'espérance de $X$ et interpréter le résultat. + \end{enumerate} +\end{exercise} + +\begin{solution} + \begin{enumerate} + \item + \begin{tikzpicture}[sloped] + \node {.} + child {node {$0$} + child {node {$0$} + child {node {$0$} + edge from parent + node[above] {0.96} + } + child {node {$1$} + edge from parent + node[above] {0.04} + } + edge from parent + node[above] {0.96} + } + child[missing] {} + child {node {$1$} + child {node {$0$} + edge from parent + node[above] {0.96} + } + child {node {$1$} + edge from parent + node[above] {0.04} + } + edge from parent + node[above] {0.96} + } + edge from parent + node[above] {0.96} + } + child[missing] {} + child[missing] {} + child[missing] {} + child { node {$1$} + child {node {$0$} + child {node {$0$} + edge from parent + node[above] {0.96} + } + child {node {$1$} + edge from parent + node[above] {0.04} + } + edge from parent + node[above] {0.96} + } + child[missing] {} + child {node {$1$} + child {node {$0$} + edge from parent + node[above] {0.96} + } + child {node {$1$} + edge from parent + node[above] {0.04} + } + edge from parent + node[above] {0.96} + } + edge from parent + node[above] {0.04} + } ; + \end{tikzpicture} + \item Chaque personne a 2 possibilités (1: fait sonner ou 2: ne fait pas sonner) et l'on fait passer 3 personnes ce qui correspond à une répétition identique et aléatoire. On peut donc modéliser la situation par une loi binomiale. + \[ + X \sim \mathcal{B}(3; 0.76) + \] + \item Probabilité qu'une seule personne fasse sonner le portique. On voit qu'il y a 3 branches qui correspondent à cette situation dont + \[ + P(X = 1) = 3 \times 0.04^1 \times 0.96^2 \approx 0.111 + \] + \item + \[ + P(X = 0) = 0.96^3 \approx 0.885 + \] + \[ + P(X \geq 2) = P(X = 2) + P(X = 3) = 3 \times 0.04^2 \times 0.96^1 + 0.04^3 \approx 0.005 + \] + + \item Il faut d'abord tracer le tableau résumant la loi de probabilité: + \begin{center} + \begin{tabular}{|c|*{4}{c|}} + \hline + Valeur & 0 & 1 & 2 & 3 \\ + \hline + Probabilité & $0.885$ & $0.111$ & $0.005$ &$0.0$ \\ + \hline + \end{tabular} + \end{center} + On peut alors calculer l'espérance + \[ + E[X] = 0 \times 0.885 + 1 \times 0.111 + 2 \times 0.005 + 3 \times 0.0 = 0.12 + \] + On peut donc estimer qu'il y aura en moyenne $0.12$ personnes qui feront sonner le portique sur les 3 personnes. + \end{enumerate} +\end{solution} + +\begin{exercise}[subtitle={Équation puissance}] + Résoudre les équations et inéquations suivantes + \begin{multicols}{2} + \begin{enumerate} + \item $10^x = 14$ + \item $11^x = 35$ + \item $0.05^x \leq 24$ + \item $4 \times 0.92^x = 47$ + \end{enumerate} + \end{multicols} +\end{exercise} + +\begin{solution} + Les solutions ci-dessous ne sont pas justifiée car l'ordinateur ne sait pas faire. Par contre, vous vous devez savoir justifier vos réponses! + \begin{enumerate} + \item $x = \log(14)$ + \item $x = \frac{\log(35)}{\log(11)}$ + \item Il faut faire attention quand on divise par un log car ce dernier peut être négatif ce qui est le cas ici. Il faut donc pense à changer le sens de l'inégalité. + + $x \geq \frac{\log(24)}{\log(0.05)}$ + + \item Il faut penser à faire la division à par $4$ avant d'utiliser le log car sinon, on ne peut pas utiliser la formule $\log(a^n) = n\times \log(a)$. + + $x = \frac{\log(11.75)}{\log(0.92)}$ + \end{enumerate} +\end{solution} + +\begin{exercise}[subtitle={Étude de fonctions}] + Soit $f(x) = 4x^3 - 306x^2 + 4128x + 39$ une fonction définie sur $\R$. + \begin{enumerate} + \item Calculer $f'(x)$ la dérivée de $f(x)$. + \item Calculer $f'(43)$ et $f'(8)$. + \item En déduire une forme factorisée de $f'(x)$. + \item Étudier le signe de $f'(x)$ et en déduire les variations de $f(x)$. + \item Est-ce que la fonction $f(x)$ admet un maximum ou un minimum? Si oui, calculer sa valeur. + \end{enumerate} +\end{exercise} + +\begin{solution} + \begin{enumerate} + \item Dérivée de $f(x)$: $f'(x) = 12x^2 - 612x + 4128$ + \item + \begin{align*} + f'(43) &= 12 \times 43^{2} - 612 \times 43 + 4128\\&= 12 \times 1849 - 26316 + 4128\\&= 22188 - 22188\\&= 0 + \end{align*} + \begin{align*} + f'(8) &= 12 \times 8^{2} - 612 \times 8 + 4128\\&= 12 \times 64 - 4896 + 4128\\&= 768 - 768\\&= 0 + \end{align*} + Donc $x = 43$ et $x=8$ sont des racines de $f'(x) = 12x^2 - 612x + 4128$. + \item On en déduit la forme factorisée suivante + \[ + f'(x) = 12 (x - 43)(x-8) + \] + \item Pas de correction disponible + \item À causes des branches extérieurs, la fonction $f(x)$ n'a pas de maximum ou de minimum. + \end{enumerate} +\end{solution} + + + +%\printsolutionstype{exercise} + + + +\end{document} + +%%% Local Variables: +%%% mode: latex +%%% TeX-master: "master" +%%% End: diff --git a/TST/DM/2102_DM2/TST3/04_2102_DM2.tex b/TST/DM/2102_DM2/TST3/04_2102_DM2.tex new file mode 100644 index 0000000..f9562f7 --- /dev/null +++ b/TST/DM/2102_DM2/TST3/04_2102_DM2.tex @@ -0,0 +1,203 @@ +\documentclass[a5paper,10pt]{article} +\usepackage{myXsim} +\usepackage{tasks} + +% Title Page +\title{DM2 \hfill BOUALIA Bilel} +\tribe{TST} +\date{\hfillÀ render pour le Mercredi 24 février} + +\xsimsetup{ + solution/print = false +} + +\begin{document} +\maketitle + +\begin{exercise}[subtitle={Loi binomiale}] + Trois personnes s'apprêtent à passer le portique de sécurité. On suppose que pour chaque personne la probabilité que le portique sonne est égale à $0.68$. + + Soit $X$ la variable aléatoire donnant le nombre de personnes faisant sonner le portique, parmi les 3 personnes de ce groupe. + \begin{enumerate} + \item Tracer l'arbre représentant le situation. + \item Justifier que $X$ suit une loi binomiale dont on précisera les paramètres. + \item Quelle est la probabilité qu'une seule personne fasse sonner le portique? + \item Calculer puis interpréter les probabilités suivantes + \[ + P(X = 0) \qquad \qquad P(X \geq 2) + \] + \item Calculer l'espérance de $X$ et interpréter le résultat. + \end{enumerate} +\end{exercise} + +\begin{solution} + \begin{enumerate} + \item + \begin{tikzpicture}[sloped] + \node {.} + child {node {$0$} + child {node {$0$} + child {node {$0$} + edge from parent + node[above] {0.32} + } + child {node {$1$} + edge from parent + node[above] {0.68} + } + edge from parent + node[above] {0.32} + } + child[missing] {} + child {node {$1$} + child {node {$0$} + edge from parent + node[above] {0.32} + } + child {node {$1$} + edge from parent + node[above] {0.68} + } + edge from parent + node[above] {0.32} + } + edge from parent + node[above] {0.32} + } + child[missing] {} + child[missing] {} + child[missing] {} + child { node {$1$} + child {node {$0$} + child {node {$0$} + edge from parent + node[above] {0.32} + } + child {node {$1$} + edge from parent + node[above] {0.68} + } + edge from parent + node[above] {0.32} + } + child[missing] {} + child {node {$1$} + child {node {$0$} + edge from parent + node[above] {0.32} + } + child {node {$1$} + edge from parent + node[above] {0.68} + } + edge from parent + node[above] {0.32} + } + edge from parent + node[above] {0.68} + } ; + \end{tikzpicture} + \item Chaque personne a 2 possibilités (1: fait sonner ou 2: ne fait pas sonner) et l'on fait passer 3 personnes ce qui correspond à une répétition identique et aléatoire. On peut donc modéliser la situation par une loi binomiale. + \[ + X \sim \mathcal{B}(3; 0.76) + \] + \item Probabilité qu'une seule personne fasse sonner le portique. On voit qu'il y a 3 branches qui correspondent à cette situation dont + \[ + P(X = 1) = 3 \times 0.68^1 \times 0.32^2 \approx 0.209 + \] + \item + \[ + P(X = 0) = 0.32^3 \approx 0.033 + \] + \[ + P(X \geq 2) = P(X = 2) + P(X = 3) = 3 \times 0.68^2 \times 0.32^1 + 0.68^3 \approx 0.758 + \] + + \item Il faut d'abord tracer le tableau résumant la loi de probabilité: + \begin{center} + \begin{tabular}{|c|*{4}{c|}} + \hline + Valeur & 0 & 1 & 2 & 3 \\ + \hline + Probabilité & $0.033$ & $0.209$ & $0.444$ &$0.314$ \\ + \hline + \end{tabular} + \end{center} + On peut alors calculer l'espérance + \[ + E[X] = 0 \times 0.033 + 1 \times 0.209 + 2 \times 0.444 + 3 \times 0.314 = 2.04 + \] + On peut donc estimer qu'il y aura en moyenne $2.04$ personnes qui feront sonner le portique sur les 3 personnes. + \end{enumerate} +\end{solution} + +\begin{exercise}[subtitle={Équation puissance}] + Résoudre les équations et inéquations suivantes + \begin{multicols}{2} + \begin{enumerate} + \item $10^x = 45$ + \item $10^x = 5$ + \item $0.69^x \leq 42$ + \item $4 \times 0.04^x = 21$ + \end{enumerate} + \end{multicols} +\end{exercise} + +\begin{solution} + Les solutions ci-dessous ne sont pas justifiée car l'ordinateur ne sait pas faire. Par contre, vous vous devez savoir justifier vos réponses! + \begin{enumerate} + \item $x = \log(45)$ + \item $x = \frac{\log(5)}{\log(10)}$ + \item Il faut faire attention quand on divise par un log car ce dernier peut être négatif ce qui est le cas ici. Il faut donc pense à changer le sens de l'inégalité. + + $x \geq \frac{\log(42)}{\log(0.69)}$ + + \item Il faut penser à faire la division à par $4$ avant d'utiliser le log car sinon, on ne peut pas utiliser la formule $\log(a^n) = n\times \log(a)$. + + $x = \frac{\log(5.25)}{\log(0.04)}$ + \end{enumerate} +\end{solution} + +\begin{exercise}[subtitle={Étude de fonctions}] + Soit $f(x) = 10x^3 - 645x^2 - 7200x - 16$ une fonction définie sur $\R$. + \begin{enumerate} + \item Calculer $f'(x)$ la dérivée de $f(x)$. + \item Calculer $f'(48)$ et $f'(-5)$. + \item En déduire une forme factorisée de $f'(x)$. + \item Étudier le signe de $f'(x)$ et en déduire les variations de $f(x)$. + \item Est-ce que la fonction $f(x)$ admet un maximum ou un minimum? Si oui, calculer sa valeur. + \end{enumerate} +\end{exercise} + +\begin{solution} + \begin{enumerate} + \item Dérivée de $f(x)$: $f'(x) = 30x^2 - 1290x - 7200$ + \item + \begin{align*} + f'(48) &= 30 \times 48^{2} - 1290 \times 48 - 7200\\&= 30 \times 2304 - 61920 - 7200\\&= 69120 - 69120\\&= 0 + \end{align*} + \begin{align*} + f'(-5) &= 30 \times - 5^{2} - 1290(- 5) - 7200\\&= 30 \times 25 + 6450 - 7200\\&= 750 - 750\\&= 0 + \end{align*} + Donc $x = 48$ et $x=-5$ sont des racines de $f'(x) = 30x^2 - 1290x - 7200$. + \item On en déduit la forme factorisée suivante + \[ + f'(x) = 30 (x - 48)(x--5) + \] + \item Pas de correction disponible + \item À causes des branches extérieurs, la fonction $f(x)$ n'a pas de maximum ou de minimum. + \end{enumerate} +\end{solution} + + + +%\printsolutionstype{exercise} + + + +\end{document} + +%%% Local Variables: +%%% mode: latex +%%% TeX-master: "master" +%%% End: diff --git a/TST/DM/2102_DM2/TST3/05_2102_DM2.tex b/TST/DM/2102_DM2/TST3/05_2102_DM2.tex new file mode 100644 index 0000000..af8c50c --- /dev/null +++ b/TST/DM/2102_DM2/TST3/05_2102_DM2.tex @@ -0,0 +1,203 @@ +\documentclass[a5paper,10pt]{article} +\usepackage{myXsim} +\usepackage{tasks} + +% Title Page +\title{DM2 \hfill BOUCHOUX Kevin} +\tribe{TST} +\date{\hfillÀ render pour le Mercredi 24 février} + +\xsimsetup{ + solution/print = false +} + +\begin{document} +\maketitle + +\begin{exercise}[subtitle={Loi binomiale}] + Trois personnes s'apprêtent à passer le portique de sécurité. On suppose que pour chaque personne la probabilité que le portique sonne est égale à $0.7$. + + Soit $X$ la variable aléatoire donnant le nombre de personnes faisant sonner le portique, parmi les 3 personnes de ce groupe. + \begin{enumerate} + \item Tracer l'arbre représentant le situation. + \item Justifier que $X$ suit une loi binomiale dont on précisera les paramètres. + \item Quelle est la probabilité qu'une seule personne fasse sonner le portique? + \item Calculer puis interpréter les probabilités suivantes + \[ + P(X = 0) \qquad \qquad P(X \geq 2) + \] + \item Calculer l'espérance de $X$ et interpréter le résultat. + \end{enumerate} +\end{exercise} + +\begin{solution} + \begin{enumerate} + \item + \begin{tikzpicture}[sloped] + \node {.} + child {node {$0$} + child {node {$0$} + child {node {$0$} + edge from parent + node[above] {0.3} + } + child {node {$1$} + edge from parent + node[above] {0.7} + } + edge from parent + node[above] {0.3} + } + child[missing] {} + child {node {$1$} + child {node {$0$} + edge from parent + node[above] {0.3} + } + child {node {$1$} + edge from parent + node[above] {0.7} + } + edge from parent + node[above] {0.3} + } + edge from parent + node[above] {0.3} + } + child[missing] {} + child[missing] {} + child[missing] {} + child { node {$1$} + child {node {$0$} + child {node {$0$} + edge from parent + node[above] {0.3} + } + child {node {$1$} + edge from parent + node[above] {0.7} + } + edge from parent + node[above] {0.3} + } + child[missing] {} + child {node {$1$} + child {node {$0$} + edge from parent + node[above] {0.3} + } + child {node {$1$} + edge from parent + node[above] {0.7} + } + edge from parent + node[above] {0.3} + } + edge from parent + node[above] {0.7} + } ; + \end{tikzpicture} + \item Chaque personne a 2 possibilités (1: fait sonner ou 2: ne fait pas sonner) et l'on fait passer 3 personnes ce qui correspond à une répétition identique et aléatoire. On peut donc modéliser la situation par une loi binomiale. + \[ + X \sim \mathcal{B}(3; 0.76) + \] + \item Probabilité qu'une seule personne fasse sonner le portique. On voit qu'il y a 3 branches qui correspondent à cette situation dont + \[ + P(X = 1) = 3 \times 0.7^1 \times 0.3^2 \approx 0.189 + \] + \item + \[ + P(X = 0) = 0.3^3 \approx 0.027 + \] + \[ + P(X \geq 2) = P(X = 2) + P(X = 3) = 3 \times 0.7^2 \times 0.3^1 + 0.7^3 \approx 0.784 + \] + + \item Il faut d'abord tracer le tableau résumant la loi de probabilité: + \begin{center} + \begin{tabular}{|c|*{4}{c|}} + \hline + Valeur & 0 & 1 & 2 & 3 \\ + \hline + Probabilité & $0.027$ & $0.189$ & $0.441$ &$0.343$ \\ + \hline + \end{tabular} + \end{center} + On peut alors calculer l'espérance + \[ + E[X] = 0 \times 0.027 + 1 \times 0.189 + 2 \times 0.441 + 3 \times 0.343 = 2.1 + \] + On peut donc estimer qu'il y aura en moyenne $2.1$ personnes qui feront sonner le portique sur les 3 personnes. + \end{enumerate} +\end{solution} + +\begin{exercise}[subtitle={Équation puissance}] + Résoudre les équations et inéquations suivantes + \begin{multicols}{2} + \begin{enumerate} + \item $10^x = 4$ + \item $7^x = 14$ + \item $0.44^x \leq 29$ + \item $6 \times 0.27^x = 10$ + \end{enumerate} + \end{multicols} +\end{exercise} + +\begin{solution} + Les solutions ci-dessous ne sont pas justifiée car l'ordinateur ne sait pas faire. Par contre, vous vous devez savoir justifier vos réponses! + \begin{enumerate} + \item $x = \log(4)$ + \item $x = \frac{\log(14)}{\log(7)}$ + \item Il faut faire attention quand on divise par un log car ce dernier peut être négatif ce qui est le cas ici. Il faut donc pense à changer le sens de l'inégalité. + + $x \geq \frac{\log(29)}{\log(0.44)}$ + + \item Il faut penser à faire la division à par $6$ avant d'utiliser le log car sinon, on ne peut pas utiliser la formule $\log(a^n) = n\times \log(a)$. + + $x = \frac{\log(1.67)}{\log(0.27)}$ + \end{enumerate} +\end{solution} + +\begin{exercise}[subtitle={Étude de fonctions}] + Soit $f(x) = 10x^3 - 840x^2 + 18450x - 1$ une fonction définie sur $\R$. + \begin{enumerate} + \item Calculer $f'(x)$ la dérivée de $f(x)$. + \item Calculer $f'(41)$ et $f'(15)$. + \item En déduire une forme factorisée de $f'(x)$. + \item Étudier le signe de $f'(x)$ et en déduire les variations de $f(x)$. + \item Est-ce que la fonction $f(x)$ admet un maximum ou un minimum? Si oui, calculer sa valeur. + \end{enumerate} +\end{exercise} + +\begin{solution} + \begin{enumerate} + \item Dérivée de $f(x)$: $f'(x) = 30x^2 - 1680x + 18450$ + \item + \begin{align*} + f'(41) &= 30 \times 41^{2} - 1680 \times 41 + 18450\\&= 30 \times 1681 - 68880 + 18450\\&= 50430 - 50430\\&= 0 + \end{align*} + \begin{align*} + f'(15) &= 30 \times 15^{2} - 1680 \times 15 + 18450\\&= 30 \times 225 - 25200 + 18450\\&= 6750 - 6750\\&= 0 + \end{align*} + Donc $x = 41$ et $x=15$ sont des racines de $f'(x) = 30x^2 - 1680x + 18450$. + \item On en déduit la forme factorisée suivante + \[ + f'(x) = 30 (x - 41)(x-15) + \] + \item Pas de correction disponible + \item À causes des branches extérieurs, la fonction $f(x)$ n'a pas de maximum ou de minimum. + \end{enumerate} +\end{solution} + + + +%\printsolutionstype{exercise} + + + +\end{document} + +%%% Local Variables: +%%% mode: latex +%%% TeX-master: "master" +%%% End: diff --git a/TST/DM/2102_DM2/TST3/06_2102_DM2.tex b/TST/DM/2102_DM2/TST3/06_2102_DM2.tex new file mode 100644 index 0000000..b968437 --- /dev/null +++ b/TST/DM/2102_DM2/TST3/06_2102_DM2.tex @@ -0,0 +1,203 @@ +\documentclass[a5paper,10pt]{article} +\usepackage{myXsim} +\usepackage{tasks} + +% Title Page +\title{DM2 \hfill BUDIN Nathan} +\tribe{TST} +\date{\hfillÀ render pour le Mercredi 24 février} + +\xsimsetup{ + solution/print = false +} + +\begin{document} +\maketitle + +\begin{exercise}[subtitle={Loi binomiale}] + Trois personnes s'apprêtent à passer le portique de sécurité. On suppose que pour chaque personne la probabilité que le portique sonne est égale à $0.37$. + + Soit $X$ la variable aléatoire donnant le nombre de personnes faisant sonner le portique, parmi les 3 personnes de ce groupe. + \begin{enumerate} + \item Tracer l'arbre représentant le situation. + \item Justifier que $X$ suit une loi binomiale dont on précisera les paramètres. + \item Quelle est la probabilité qu'une seule personne fasse sonner le portique? + \item Calculer puis interpréter les probabilités suivantes + \[ + P(X = 0) \qquad \qquad P(X \geq 2) + \] + \item Calculer l'espérance de $X$ et interpréter le résultat. + \end{enumerate} +\end{exercise} + +\begin{solution} + \begin{enumerate} + \item + \begin{tikzpicture}[sloped] + \node {.} + child {node {$0$} + child {node {$0$} + child {node {$0$} + edge from parent + node[above] {0.63} + } + child {node {$1$} + edge from parent + node[above] {0.37} + } + edge from parent + node[above] {0.63} + } + child[missing] {} + child {node {$1$} + child {node {$0$} + edge from parent + node[above] {0.63} + } + child {node {$1$} + edge from parent + node[above] {0.37} + } + edge from parent + node[above] {0.63} + } + edge from parent + node[above] {0.63} + } + child[missing] {} + child[missing] {} + child[missing] {} + child { node {$1$} + child {node {$0$} + child {node {$0$} + edge from parent + node[above] {0.63} + } + child {node {$1$} + edge from parent + node[above] {0.37} + } + edge from parent + node[above] {0.63} + } + child[missing] {} + child {node {$1$} + child {node {$0$} + edge from parent + node[above] {0.63} + } + child {node {$1$} + edge from parent + node[above] {0.37} + } + edge from parent + node[above] {0.63} + } + edge from parent + node[above] {0.37} + } ; + \end{tikzpicture} + \item Chaque personne a 2 possibilités (1: fait sonner ou 2: ne fait pas sonner) et l'on fait passer 3 personnes ce qui correspond à une répétition identique et aléatoire. On peut donc modéliser la situation par une loi binomiale. + \[ + X \sim \mathcal{B}(3; 0.76) + \] + \item Probabilité qu'une seule personne fasse sonner le portique. On voit qu'il y a 3 branches qui correspondent à cette situation dont + \[ + P(X = 1) = 3 \times 0.37^1 \times 0.63^2 \approx 0.441 + \] + \item + \[ + P(X = 0) = 0.63^3 \approx 0.25 + \] + \[ + P(X \geq 2) = P(X = 2) + P(X = 3) = 3 \times 0.37^2 \times 0.63^1 + 0.37^3 \approx 0.31 + \] + + \item Il faut d'abord tracer le tableau résumant la loi de probabilité: + \begin{center} + \begin{tabular}{|c|*{4}{c|}} + \hline + Valeur & 0 & 1 & 2 & 3 \\ + \hline + Probabilité & $0.25$ & $0.441$ & $0.259$ &$0.051$ \\ + \hline + \end{tabular} + \end{center} + On peut alors calculer l'espérance + \[ + E[X] = 0 \times 0.25 + 1 \times 0.441 + 2 \times 0.259 + 3 \times 0.051 = 1.11 + \] + On peut donc estimer qu'il y aura en moyenne $1.11$ personnes qui feront sonner le portique sur les 3 personnes. + \end{enumerate} +\end{solution} + +\begin{exercise}[subtitle={Équation puissance}] + Résoudre les équations et inéquations suivantes + \begin{multicols}{2} + \begin{enumerate} + \item $10^x = 44$ + \item $2^x = 33$ + \item $0.94^x \leq 43$ + \item $9 \times 0.17^x = 30$ + \end{enumerate} + \end{multicols} +\end{exercise} + +\begin{solution} + Les solutions ci-dessous ne sont pas justifiée car l'ordinateur ne sait pas faire. Par contre, vous vous devez savoir justifier vos réponses! + \begin{enumerate} + \item $x = \log(44)$ + \item $x = \frac{\log(33)}{\log(2)}$ + \item Il faut faire attention quand on divise par un log car ce dernier peut être négatif ce qui est le cas ici. Il faut donc pense à changer le sens de l'inégalité. + + $x \geq \frac{\log(43)}{\log(0.94)}$ + + \item Il faut penser à faire la division à par $9$ avant d'utiliser le log car sinon, on ne peut pas utiliser la formule $\log(a^n) = n\times \log(a)$. + + $x = \frac{\log(3.33)}{\log(0.17)}$ + \end{enumerate} +\end{solution} + +\begin{exercise}[subtitle={Étude de fonctions}] + Soit $f(x) = - 2x^3 + 99x^2 + 2580x + 46$ une fonction définie sur $\R$. + \begin{enumerate} + \item Calculer $f'(x)$ la dérivée de $f(x)$. + \item Calculer $f'(43)$ et $f'(-10)$. + \item En déduire une forme factorisée de $f'(x)$. + \item Étudier le signe de $f'(x)$ et en déduire les variations de $f(x)$. + \item Est-ce que la fonction $f(x)$ admet un maximum ou un minimum? Si oui, calculer sa valeur. + \end{enumerate} +\end{exercise} + +\begin{solution} + \begin{enumerate} + \item Dérivée de $f(x)$: $f'(x) = - 6x^2 + 198x + 2580$ + \item + \begin{align*} + f'(43) &= - 6 \times 43^{2} + 198 \times 43 + 2580\\&= - 6 \times 1849 + 8514 + 2580\\&= - 11094 + 11094\\&= 0 + \end{align*} + \begin{align*} + f'(-10) &= - 6 \times - 10^{2} + 198(- 10) + 2580\\&= - 6 \times 100 - 1980 + 2580\\&= - 600 + 600\\&= 0 + \end{align*} + Donc $x = 43$ et $x=-10$ sont des racines de $f'(x) = - 6x^2 + 198x + 2580$. + \item On en déduit la forme factorisée suivante + \[ + f'(x) = -6 (x - 43)(x--10) + \] + \item Pas de correction disponible + \item À causes des branches extérieurs, la fonction $f(x)$ n'a pas de maximum ou de minimum. + \end{enumerate} +\end{solution} + + + +%\printsolutionstype{exercise} + + + +\end{document} + +%%% Local Variables: +%%% mode: latex +%%% TeX-master: "master" +%%% End: diff --git a/TST/DM/2102_DM2/TST3/07_2102_DM2.tex b/TST/DM/2102_DM2/TST3/07_2102_DM2.tex new file mode 100644 index 0000000..829e96b --- /dev/null +++ b/TST/DM/2102_DM2/TST3/07_2102_DM2.tex @@ -0,0 +1,203 @@ +\documentclass[a5paper,10pt]{article} +\usepackage{myXsim} +\usepackage{tasks} + +% Title Page +\title{DM2 \hfill CAGLAR Rojin} +\tribe{TST} +\date{\hfillÀ render pour le Mercredi 24 février} + +\xsimsetup{ + solution/print = false +} + +\begin{document} +\maketitle + +\begin{exercise}[subtitle={Loi binomiale}] + Trois personnes s'apprêtent à passer le portique de sécurité. On suppose que pour chaque personne la probabilité que le portique sonne est égale à $0.86$. + + Soit $X$ la variable aléatoire donnant le nombre de personnes faisant sonner le portique, parmi les 3 personnes de ce groupe. + \begin{enumerate} + \item Tracer l'arbre représentant le situation. + \item Justifier que $X$ suit une loi binomiale dont on précisera les paramètres. + \item Quelle est la probabilité qu'une seule personne fasse sonner le portique? + \item Calculer puis interpréter les probabilités suivantes + \[ + P(X = 0) \qquad \qquad P(X \geq 2) + \] + \item Calculer l'espérance de $X$ et interpréter le résultat. + \end{enumerate} +\end{exercise} + +\begin{solution} + \begin{enumerate} + \item + \begin{tikzpicture}[sloped] + \node {.} + child {node {$0$} + child {node {$0$} + child {node {$0$} + edge from parent + node[above] {0.14} + } + child {node {$1$} + edge from parent + node[above] {0.86} + } + edge from parent + node[above] {0.14} + } + child[missing] {} + child {node {$1$} + child {node {$0$} + edge from parent + node[above] {0.14} + } + child {node {$1$} + edge from parent + node[above] {0.86} + } + edge from parent + node[above] {0.14} + } + edge from parent + node[above] {0.14} + } + child[missing] {} + child[missing] {} + child[missing] {} + child { node {$1$} + child {node {$0$} + child {node {$0$} + edge from parent + node[above] {0.14} + } + child {node {$1$} + edge from parent + node[above] {0.86} + } + edge from parent + node[above] {0.14} + } + child[missing] {} + child {node {$1$} + child {node {$0$} + edge from parent + node[above] {0.14} + } + child {node {$1$} + edge from parent + node[above] {0.86} + } + edge from parent + node[above] {0.14} + } + edge from parent + node[above] {0.86} + } ; + \end{tikzpicture} + \item Chaque personne a 2 possibilités (1: fait sonner ou 2: ne fait pas sonner) et l'on fait passer 3 personnes ce qui correspond à une répétition identique et aléatoire. On peut donc modéliser la situation par une loi binomiale. + \[ + X \sim \mathcal{B}(3; 0.76) + \] + \item Probabilité qu'une seule personne fasse sonner le portique. On voit qu'il y a 3 branches qui correspondent à cette situation dont + \[ + P(X = 1) = 3 \times 0.86^1 \times 0.14^2 \approx 0.051 + \] + \item + \[ + P(X = 0) = 0.14^3 \approx 0.003 + \] + \[ + P(X \geq 2) = P(X = 2) + P(X = 3) = 3 \times 0.86^2 \times 0.14^1 + 0.86^3 \approx 0.947 + \] + + \item Il faut d'abord tracer le tableau résumant la loi de probabilité: + \begin{center} + \begin{tabular}{|c|*{4}{c|}} + \hline + Valeur & 0 & 1 & 2 & 3 \\ + \hline + Probabilité & $0.003$ & $0.051$ & $0.311$ &$0.636$ \\ + \hline + \end{tabular} + \end{center} + On peut alors calculer l'espérance + \[ + E[X] = 0 \times 0.003 + 1 \times 0.051 + 2 \times 0.311 + 3 \times 0.636 = 2.58 + \] + On peut donc estimer qu'il y aura en moyenne $2.58$ personnes qui feront sonner le portique sur les 3 personnes. + \end{enumerate} +\end{solution} + +\begin{exercise}[subtitle={Équation puissance}] + Résoudre les équations et inéquations suivantes + \begin{multicols}{2} + \begin{enumerate} + \item $10^x = 14$ + \item $2^x = 38$ + \item $0.15^x \leq 40$ + \item $9 \times 0.56^x = 29$ + \end{enumerate} + \end{multicols} +\end{exercise} + +\begin{solution} + Les solutions ci-dessous ne sont pas justifiée car l'ordinateur ne sait pas faire. Par contre, vous vous devez savoir justifier vos réponses! + \begin{enumerate} + \item $x = \log(14)$ + \item $x = \frac{\log(38)}{\log(2)}$ + \item Il faut faire attention quand on divise par un log car ce dernier peut être négatif ce qui est le cas ici. Il faut donc pense à changer le sens de l'inégalité. + + $x \geq \frac{\log(40)}{\log(0.15)}$ + + \item Il faut penser à faire la division à par $9$ avant d'utiliser le log car sinon, on ne peut pas utiliser la formule $\log(a^n) = n\times \log(a)$. + + $x = \frac{\log(3.22)}{\log(0.56)}$ + \end{enumerate} +\end{solution} + +\begin{exercise}[subtitle={Étude de fonctions}] + Soit $f(x) = - x^3 + 67.5x^2 - 258x - 11$ une fonction définie sur $\R$. + \begin{enumerate} + \item Calculer $f'(x)$ la dérivée de $f(x)$. + \item Calculer $f'(43)$ et $f'(2)$. + \item En déduire une forme factorisée de $f'(x)$. + \item Étudier le signe de $f'(x)$ et en déduire les variations de $f(x)$. + \item Est-ce que la fonction $f(x)$ admet un maximum ou un minimum? Si oui, calculer sa valeur. + \end{enumerate} +\end{exercise} + +\begin{solution} + \begin{enumerate} + \item Dérivée de $f(x)$: $f'(x) = - 3x^2 + 135x - 258$ + \item + \begin{align*} + f'(43) &= - 3 \times 43^{2} + 135 \times 43 - 258\\&= - 3 \times 1849 + 5805 - 258\\&= - 5547 + 5547\\&= 0 + \end{align*} + \begin{align*} + f'(2) &= - 3 \times 2^{2} + 135 \times 2 - 258\\&= - 3 \times 4 + 270 - 258\\&= - 12 + 12\\&= 0 + \end{align*} + Donc $x = 43$ et $x=2$ sont des racines de $f'(x) = - 3x^2 + 135x - 258$. + \item On en déduit la forme factorisée suivante + \[ + f'(x) = -3 (x - 43)(x-2) + \] + \item Pas de correction disponible + \item À causes des branches extérieurs, la fonction $f(x)$ n'a pas de maximum ou de minimum. + \end{enumerate} +\end{solution} + + + +%\printsolutionstype{exercise} + + + +\end{document} + +%%% Local Variables: +%%% mode: latex +%%% TeX-master: "master" +%%% End: diff --git a/TST/DM/2102_DM2/TST3/08_2102_DM2.tex b/TST/DM/2102_DM2/TST3/08_2102_DM2.tex new file mode 100644 index 0000000..ba506c3 --- /dev/null +++ b/TST/DM/2102_DM2/TST3/08_2102_DM2.tex @@ -0,0 +1,203 @@ +\documentclass[a5paper,10pt]{article} +\usepackage{myXsim} +\usepackage{tasks} + +% Title Page +\title{DM2 \hfill DARICHE Kaïs} +\tribe{TST} +\date{\hfillÀ render pour le Mercredi 24 février} + +\xsimsetup{ + solution/print = false +} + +\begin{document} +\maketitle + +\begin{exercise}[subtitle={Loi binomiale}] + Trois personnes s'apprêtent à passer le portique de sécurité. On suppose que pour chaque personne la probabilité que le portique sonne est égale à $0.16$. + + Soit $X$ la variable aléatoire donnant le nombre de personnes faisant sonner le portique, parmi les 3 personnes de ce groupe. + \begin{enumerate} + \item Tracer l'arbre représentant le situation. + \item Justifier que $X$ suit une loi binomiale dont on précisera les paramètres. + \item Quelle est la probabilité qu'une seule personne fasse sonner le portique? + \item Calculer puis interpréter les probabilités suivantes + \[ + P(X = 0) \qquad \qquad P(X \geq 2) + \] + \item Calculer l'espérance de $X$ et interpréter le résultat. + \end{enumerate} +\end{exercise} + +\begin{solution} + \begin{enumerate} + \item + \begin{tikzpicture}[sloped] + \node {.} + child {node {$0$} + child {node {$0$} + child {node {$0$} + edge from parent + node[above] {0.84} + } + child {node {$1$} + edge from parent + node[above] {0.16} + } + edge from parent + node[above] {0.84} + } + child[missing] {} + child {node {$1$} + child {node {$0$} + edge from parent + node[above] {0.84} + } + child {node {$1$} + edge from parent + node[above] {0.16} + } + edge from parent + node[above] {0.84} + } + edge from parent + node[above] {0.84} + } + child[missing] {} + child[missing] {} + child[missing] {} + child { node {$1$} + child {node {$0$} + child {node {$0$} + edge from parent + node[above] {0.84} + } + child {node {$1$} + edge from parent + node[above] {0.16} + } + edge from parent + node[above] {0.84} + } + child[missing] {} + child {node {$1$} + child {node {$0$} + edge from parent + node[above] {0.84} + } + child {node {$1$} + edge from parent + node[above] {0.16} + } + edge from parent + node[above] {0.84} + } + edge from parent + node[above] {0.16} + } ; + \end{tikzpicture} + \item Chaque personne a 2 possibilités (1: fait sonner ou 2: ne fait pas sonner) et l'on fait passer 3 personnes ce qui correspond à une répétition identique et aléatoire. On peut donc modéliser la situation par une loi binomiale. + \[ + X \sim \mathcal{B}(3; 0.76) + \] + \item Probabilité qu'une seule personne fasse sonner le portique. On voit qu'il y a 3 branches qui correspondent à cette situation dont + \[ + P(X = 1) = 3 \times 0.16^1 \times 0.84^2 \approx 0.339 + \] + \item + \[ + P(X = 0) = 0.84^3 \approx 0.593 + \] + \[ + P(X \geq 2) = P(X = 2) + P(X = 3) = 3 \times 0.16^2 \times 0.84^1 + 0.16^3 \approx 0.069 + \] + + \item Il faut d'abord tracer le tableau résumant la loi de probabilité: + \begin{center} + \begin{tabular}{|c|*{4}{c|}} + \hline + Valeur & 0 & 1 & 2 & 3 \\ + \hline + Probabilité & $0.593$ & $0.339$ & $0.065$ &$0.004$ \\ + \hline + \end{tabular} + \end{center} + On peut alors calculer l'espérance + \[ + E[X] = 0 \times 0.593 + 1 \times 0.339 + 2 \times 0.065 + 3 \times 0.004 = 0.48 + \] + On peut donc estimer qu'il y aura en moyenne $0.48$ personnes qui feront sonner le portique sur les 3 personnes. + \end{enumerate} +\end{solution} + +\begin{exercise}[subtitle={Équation puissance}] + Résoudre les équations et inéquations suivantes + \begin{multicols}{2} + \begin{enumerate} + \item $10^x = 35$ + \item $14^x = 11$ + \item $0.39^x \leq 48$ + \item $3 \times 0.07^x = 4$ + \end{enumerate} + \end{multicols} +\end{exercise} + +\begin{solution} + Les solutions ci-dessous ne sont pas justifiée car l'ordinateur ne sait pas faire. Par contre, vous vous devez savoir justifier vos réponses! + \begin{enumerate} + \item $x = \log(35)$ + \item $x = \frac{\log(11)}{\log(14)}$ + \item Il faut faire attention quand on divise par un log car ce dernier peut être négatif ce qui est le cas ici. Il faut donc pense à changer le sens de l'inégalité. + + $x \geq \frac{\log(48)}{\log(0.39)}$ + + \item Il faut penser à faire la division à par $3$ avant d'utiliser le log car sinon, on ne peut pas utiliser la formule $\log(a^n) = n\times \log(a)$. + + $x = \frac{\log(1.33)}{\log(0.07)}$ + \end{enumerate} +\end{solution} + +\begin{exercise}[subtitle={Étude de fonctions}] + Soit $f(x) = x^3 - 54x^2 + 780x + 36$ une fonction définie sur $\R$. + \begin{enumerate} + \item Calculer $f'(x)$ la dérivée de $f(x)$. + \item Calculer $f'(26)$ et $f'(10)$. + \item En déduire une forme factorisée de $f'(x)$. + \item Étudier le signe de $f'(x)$ et en déduire les variations de $f(x)$. + \item Est-ce que la fonction $f(x)$ admet un maximum ou un minimum? Si oui, calculer sa valeur. + \end{enumerate} +\end{exercise} + +\begin{solution} + \begin{enumerate} + \item Dérivée de $f(x)$: $f'(x) = 3x^2 - 108x + 780$ + \item + \begin{align*} + f'(26) &= 3 \times 26^{2} - 108 \times 26 + 780\\&= 3 \times 676 - 2808 + 780\\&= 2028 - 2028\\&= 0 + \end{align*} + \begin{align*} + f'(10) &= 3 \times 10^{2} - 108 \times 10 + 780\\&= 3 \times 100 - 1080 + 780\\&= 300 - 300\\&= 0 + \end{align*} + Donc $x = 26$ et $x=10$ sont des racines de $f'(x) = 3x^2 - 108x + 780$. + \item On en déduit la forme factorisée suivante + \[ + f'(x) = 3 (x - 26)(x-10) + \] + \item Pas de correction disponible + \item À causes des branches extérieurs, la fonction $f(x)$ n'a pas de maximum ou de minimum. + \end{enumerate} +\end{solution} + + + +%\printsolutionstype{exercise} + + + +\end{document} + +%%% Local Variables: +%%% mode: latex +%%% TeX-master: "master" +%%% End: diff --git a/TST/DM/2102_DM2/TST3/09_2102_DM2.tex b/TST/DM/2102_DM2/TST3/09_2102_DM2.tex new file mode 100644 index 0000000..83f5e75 --- /dev/null +++ b/TST/DM/2102_DM2/TST3/09_2102_DM2.tex @@ -0,0 +1,203 @@ +\documentclass[a5paper,10pt]{article} +\usepackage{myXsim} +\usepackage{tasks} + +% Title Page +\title{DM2 \hfill DEBRAS Noémie} +\tribe{TST} +\date{\hfillÀ render pour le Mercredi 24 février} + +\xsimsetup{ + solution/print = false +} + +\begin{document} +\maketitle + +\begin{exercise}[subtitle={Loi binomiale}] + Trois personnes s'apprêtent à passer le portique de sécurité. On suppose que pour chaque personne la probabilité que le portique sonne est égale à $0.1$. + + Soit $X$ la variable aléatoire donnant le nombre de personnes faisant sonner le portique, parmi les 3 personnes de ce groupe. + \begin{enumerate} + \item Tracer l'arbre représentant le situation. + \item Justifier que $X$ suit une loi binomiale dont on précisera les paramètres. + \item Quelle est la probabilité qu'une seule personne fasse sonner le portique? + \item Calculer puis interpréter les probabilités suivantes + \[ + P(X = 0) \qquad \qquad P(X \geq 2) + \] + \item Calculer l'espérance de $X$ et interpréter le résultat. + \end{enumerate} +\end{exercise} + +\begin{solution} + \begin{enumerate} + \item + \begin{tikzpicture}[sloped] + \node {.} + child {node {$0$} + child {node {$0$} + child {node {$0$} + edge from parent + node[above] {0.9} + } + child {node {$1$} + edge from parent + node[above] {0.1} + } + edge from parent + node[above] {0.9} + } + child[missing] {} + child {node {$1$} + child {node {$0$} + edge from parent + node[above] {0.9} + } + child {node {$1$} + edge from parent + node[above] {0.1} + } + edge from parent + node[above] {0.9} + } + edge from parent + node[above] {0.9} + } + child[missing] {} + child[missing] {} + child[missing] {} + child { node {$1$} + child {node {$0$} + child {node {$0$} + edge from parent + node[above] {0.9} + } + child {node {$1$} + edge from parent + node[above] {0.1} + } + edge from parent + node[above] {0.9} + } + child[missing] {} + child {node {$1$} + child {node {$0$} + edge from parent + node[above] {0.9} + } + child {node {$1$} + edge from parent + node[above] {0.1} + } + edge from parent + node[above] {0.9} + } + edge from parent + node[above] {0.1} + } ; + \end{tikzpicture} + \item Chaque personne a 2 possibilités (1: fait sonner ou 2: ne fait pas sonner) et l'on fait passer 3 personnes ce qui correspond à une répétition identique et aléatoire. On peut donc modéliser la situation par une loi binomiale. + \[ + X \sim \mathcal{B}(3; 0.76) + \] + \item Probabilité qu'une seule personne fasse sonner le portique. On voit qu'il y a 3 branches qui correspondent à cette situation dont + \[ + P(X = 1) = 3 \times 0.1^1 \times 0.9^2 \approx 0.243 + \] + \item + \[ + P(X = 0) = 0.9^3 \approx 0.729 + \] + \[ + P(X \geq 2) = P(X = 2) + P(X = 3) = 3 \times 0.1^2 \times 0.9^1 + 0.1^3 \approx 0.028 + \] + + \item Il faut d'abord tracer le tableau résumant la loi de probabilité: + \begin{center} + \begin{tabular}{|c|*{4}{c|}} + \hline + Valeur & 0 & 1 & 2 & 3 \\ + \hline + Probabilité & $0.729$ & $0.243$ & $0.027$ &$0.001$ \\ + \hline + \end{tabular} + \end{center} + On peut alors calculer l'espérance + \[ + E[X] = 0 \times 0.729 + 1 \times 0.243 + 2 \times 0.027 + 3 \times 0.001 = 0.3 + \] + On peut donc estimer qu'il y aura en moyenne $0.3$ personnes qui feront sonner le portique sur les 3 personnes. + \end{enumerate} +\end{solution} + +\begin{exercise}[subtitle={Équation puissance}] + Résoudre les équations et inéquations suivantes + \begin{multicols}{2} + \begin{enumerate} + \item $10^x = 25$ + \item $12^x = 21$ + \item $0.77^x \leq 22$ + \item $6 \times 0.4^x = 40$ + \end{enumerate} + \end{multicols} +\end{exercise} + +\begin{solution} + Les solutions ci-dessous ne sont pas justifiée car l'ordinateur ne sait pas faire. Par contre, vous vous devez savoir justifier vos réponses! + \begin{enumerate} + \item $x = \log(25)$ + \item $x = \frac{\log(21)}{\log(12)}$ + \item Il faut faire attention quand on divise par un log car ce dernier peut être négatif ce qui est le cas ici. Il faut donc pense à changer le sens de l'inégalité. + + $x \geq \frac{\log(22)}{\log(0.77)}$ + + \item Il faut penser à faire la division à par $6$ avant d'utiliser le log car sinon, on ne peut pas utiliser la formule $\log(a^n) = n\times \log(a)$. + + $x = \frac{\log(6.67)}{\log(0.4)}$ + \end{enumerate} +\end{solution} + +\begin{exercise}[subtitle={Étude de fonctions}] + Soit $f(x) = 4x^3 - 132x^2 - 5460x + 39$ une fonction définie sur $\R$. + \begin{enumerate} + \item Calculer $f'(x)$ la dérivée de $f(x)$. + \item Calculer $f'(35)$ et $f'(-13)$. + \item En déduire une forme factorisée de $f'(x)$. + \item Étudier le signe de $f'(x)$ et en déduire les variations de $f(x)$. + \item Est-ce que la fonction $f(x)$ admet un maximum ou un minimum? Si oui, calculer sa valeur. + \end{enumerate} +\end{exercise} + +\begin{solution} + \begin{enumerate} + \item Dérivée de $f(x)$: $f'(x) = 12x^2 - 264x - 5460$ + \item + \begin{align*} + f'(35) &= 12 \times 35^{2} - 264 \times 35 - 5460\\&= 12 \times 1225 - 9240 - 5460\\&= 14700 - 14700\\&= 0 + \end{align*} + \begin{align*} + f'(-13) &= 12 \times - 13^{2} - 264(- 13) - 5460\\&= 12 \times 169 + 3432 - 5460\\&= 2028 - 2028\\&= 0 + \end{align*} + Donc $x = 35$ et $x=-13$ sont des racines de $f'(x) = 12x^2 - 264x - 5460$. + \item On en déduit la forme factorisée suivante + \[ + f'(x) = 12 (x - 35)(x--13) + \] + \item Pas de correction disponible + \item À causes des branches extérieurs, la fonction $f(x)$ n'a pas de maximum ou de minimum. + \end{enumerate} +\end{solution} + + + +%\printsolutionstype{exercise} + + + +\end{document} + +%%% Local Variables: +%%% mode: latex +%%% TeX-master: "master" +%%% End: diff --git a/TST/DM/2102_DM2/TST3/10_2102_DM2.tex b/TST/DM/2102_DM2/TST3/10_2102_DM2.tex new file mode 100644 index 0000000..24b2c26 --- /dev/null +++ b/TST/DM/2102_DM2/TST3/10_2102_DM2.tex @@ -0,0 +1,203 @@ +\documentclass[a5paper,10pt]{article} +\usepackage{myXsim} +\usepackage{tasks} + +% Title Page +\title{DM2 \hfill GERMAIN Anaïs} +\tribe{TST} +\date{\hfillÀ render pour le Mercredi 24 février} + +\xsimsetup{ + solution/print = false +} + +\begin{document} +\maketitle + +\begin{exercise}[subtitle={Loi binomiale}] + Trois personnes s'apprêtent à passer le portique de sécurité. On suppose que pour chaque personne la probabilité que le portique sonne est égale à $0.68$. + + Soit $X$ la variable aléatoire donnant le nombre de personnes faisant sonner le portique, parmi les 3 personnes de ce groupe. + \begin{enumerate} + \item Tracer l'arbre représentant le situation. + \item Justifier que $X$ suit une loi binomiale dont on précisera les paramètres. + \item Quelle est la probabilité qu'une seule personne fasse sonner le portique? + \item Calculer puis interpréter les probabilités suivantes + \[ + P(X = 0) \qquad \qquad P(X \geq 2) + \] + \item Calculer l'espérance de $X$ et interpréter le résultat. + \end{enumerate} +\end{exercise} + +\begin{solution} + \begin{enumerate} + \item + \begin{tikzpicture}[sloped] + \node {.} + child {node {$0$} + child {node {$0$} + child {node {$0$} + edge from parent + node[above] {0.32} + } + child {node {$1$} + edge from parent + node[above] {0.68} + } + edge from parent + node[above] {0.32} + } + child[missing] {} + child {node {$1$} + child {node {$0$} + edge from parent + node[above] {0.32} + } + child {node {$1$} + edge from parent + node[above] {0.68} + } + edge from parent + node[above] {0.32} + } + edge from parent + node[above] {0.32} + } + child[missing] {} + child[missing] {} + child[missing] {} + child { node {$1$} + child {node {$0$} + child {node {$0$} + edge from parent + node[above] {0.32} + } + child {node {$1$} + edge from parent + node[above] {0.68} + } + edge from parent + node[above] {0.32} + } + child[missing] {} + child {node {$1$} + child {node {$0$} + edge from parent + node[above] {0.32} + } + child {node {$1$} + edge from parent + node[above] {0.68} + } + edge from parent + node[above] {0.32} + } + edge from parent + node[above] {0.68} + } ; + \end{tikzpicture} + \item Chaque personne a 2 possibilités (1: fait sonner ou 2: ne fait pas sonner) et l'on fait passer 3 personnes ce qui correspond à une répétition identique et aléatoire. On peut donc modéliser la situation par une loi binomiale. + \[ + X \sim \mathcal{B}(3; 0.76) + \] + \item Probabilité qu'une seule personne fasse sonner le portique. On voit qu'il y a 3 branches qui correspondent à cette situation dont + \[ + P(X = 1) = 3 \times 0.68^1 \times 0.32^2 \approx 0.209 + \] + \item + \[ + P(X = 0) = 0.32^3 \approx 0.033 + \] + \[ + P(X \geq 2) = P(X = 2) + P(X = 3) = 3 \times 0.68^2 \times 0.32^1 + 0.68^3 \approx 0.758 + \] + + \item Il faut d'abord tracer le tableau résumant la loi de probabilité: + \begin{center} + \begin{tabular}{|c|*{4}{c|}} + \hline + Valeur & 0 & 1 & 2 & 3 \\ + \hline + Probabilité & $0.033$ & $0.209$ & $0.444$ &$0.314$ \\ + \hline + \end{tabular} + \end{center} + On peut alors calculer l'espérance + \[ + E[X] = 0 \times 0.033 + 1 \times 0.209 + 2 \times 0.444 + 3 \times 0.314 = 2.04 + \] + On peut donc estimer qu'il y aura en moyenne $2.04$ personnes qui feront sonner le portique sur les 3 personnes. + \end{enumerate} +\end{solution} + +\begin{exercise}[subtitle={Équation puissance}] + Résoudre les équations et inéquations suivantes + \begin{multicols}{2} + \begin{enumerate} + \item $10^x = 48$ + \item $2^x = 42$ + \item $0.47^x \leq 46$ + \item $7 \times 0.37^x = 45$ + \end{enumerate} + \end{multicols} +\end{exercise} + +\begin{solution} + Les solutions ci-dessous ne sont pas justifiée car l'ordinateur ne sait pas faire. Par contre, vous vous devez savoir justifier vos réponses! + \begin{enumerate} + \item $x = \log(48)$ + \item $x = \frac{\log(42)}{\log(2)}$ + \item Il faut faire attention quand on divise par un log car ce dernier peut être négatif ce qui est le cas ici. Il faut donc pense à changer le sens de l'inégalité. + + $x \geq \frac{\log(46)}{\log(0.47)}$ + + \item Il faut penser à faire la division à par $7$ avant d'utiliser le log car sinon, on ne peut pas utiliser la formule $\log(a^n) = n\times \log(a)$. + + $x = \frac{\log(6.43)}{\log(0.37)}$ + \end{enumerate} +\end{solution} + +\begin{exercise}[subtitle={Étude de fonctions}] + Soit $f(x) = - 7x^3 + 472.5x^2 + 4116x + 48$ une fonction définie sur $\R$. + \begin{enumerate} + \item Calculer $f'(x)$ la dérivée de $f(x)$. + \item Calculer $f'(49)$ et $f'(-4)$. + \item En déduire une forme factorisée de $f'(x)$. + \item Étudier le signe de $f'(x)$ et en déduire les variations de $f(x)$. + \item Est-ce que la fonction $f(x)$ admet un maximum ou un minimum? Si oui, calculer sa valeur. + \end{enumerate} +\end{exercise} + +\begin{solution} + \begin{enumerate} + \item Dérivée de $f(x)$: $f'(x) = - 21x^2 + 945x + 4116$ + \item + \begin{align*} + f'(49) &= - 21 \times 49^{2} + 945 \times 49 + 4116\\&= - 21 \times 2401 + 46305 + 4116\\&= - 50421 + 50421\\&= 0 + \end{align*} + \begin{align*} + f'(-4) &= - 21 \times - 4^{2} + 945(- 4) + 4116\\&= - 21 \times 16 - 3780 + 4116\\&= - 336 + 336\\&= 0 + \end{align*} + Donc $x = 49$ et $x=-4$ sont des racines de $f'(x) = - 21x^2 + 945x + 4116$. + \item On en déduit la forme factorisée suivante + \[ + f'(x) = -21 (x - 49)(x--4) + \] + \item Pas de correction disponible + \item À causes des branches extérieurs, la fonction $f(x)$ n'a pas de maximum ou de minimum. + \end{enumerate} +\end{solution} + + + +%\printsolutionstype{exercise} + + + +\end{document} + +%%% Local Variables: +%%% mode: latex +%%% TeX-master: "master" +%%% End: diff --git a/TST/DM/2102_DM2/TST3/11_2102_DM2.tex b/TST/DM/2102_DM2/TST3/11_2102_DM2.tex new file mode 100644 index 0000000..57465aa --- /dev/null +++ b/TST/DM/2102_DM2/TST3/11_2102_DM2.tex @@ -0,0 +1,203 @@ +\documentclass[a5paper,10pt]{article} +\usepackage{myXsim} +\usepackage{tasks} + +% Title Page +\title{DM2 \hfill HADJRAS Mohcine} +\tribe{TST} +\date{\hfillÀ render pour le Mercredi 24 février} + +\xsimsetup{ + solution/print = false +} + +\begin{document} +\maketitle + +\begin{exercise}[subtitle={Loi binomiale}] + Trois personnes s'apprêtent à passer le portique de sécurité. On suppose que pour chaque personne la probabilité que le portique sonne est égale à $0.31$. + + Soit $X$ la variable aléatoire donnant le nombre de personnes faisant sonner le portique, parmi les 3 personnes de ce groupe. + \begin{enumerate} + \item Tracer l'arbre représentant le situation. + \item Justifier que $X$ suit une loi binomiale dont on précisera les paramètres. + \item Quelle est la probabilité qu'une seule personne fasse sonner le portique? + \item Calculer puis interpréter les probabilités suivantes + \[ + P(X = 0) \qquad \qquad P(X \geq 2) + \] + \item Calculer l'espérance de $X$ et interpréter le résultat. + \end{enumerate} +\end{exercise} + +\begin{solution} + \begin{enumerate} + \item + \begin{tikzpicture}[sloped] + \node {.} + child {node {$0$} + child {node {$0$} + child {node {$0$} + edge from parent + node[above] {0.69} + } + child {node {$1$} + edge from parent + node[above] {0.31} + } + edge from parent + node[above] {0.69} + } + child[missing] {} + child {node {$1$} + child {node {$0$} + edge from parent + node[above] {0.69} + } + child {node {$1$} + edge from parent + node[above] {0.31} + } + edge from parent + node[above] {0.69} + } + edge from parent + node[above] {0.69} + } + child[missing] {} + child[missing] {} + child[missing] {} + child { node {$1$} + child {node {$0$} + child {node {$0$} + edge from parent + node[above] {0.69} + } + child {node {$1$} + edge from parent + node[above] {0.31} + } + edge from parent + node[above] {0.69} + } + child[missing] {} + child {node {$1$} + child {node {$0$} + edge from parent + node[above] {0.69} + } + child {node {$1$} + edge from parent + node[above] {0.31} + } + edge from parent + node[above] {0.69} + } + edge from parent + node[above] {0.31} + } ; + \end{tikzpicture} + \item Chaque personne a 2 possibilités (1: fait sonner ou 2: ne fait pas sonner) et l'on fait passer 3 personnes ce qui correspond à une répétition identique et aléatoire. On peut donc modéliser la situation par une loi binomiale. + \[ + X \sim \mathcal{B}(3; 0.76) + \] + \item Probabilité qu'une seule personne fasse sonner le portique. On voit qu'il y a 3 branches qui correspondent à cette situation dont + \[ + P(X = 1) = 3 \times 0.31^1 \times 0.69^2 \approx 0.443 + \] + \item + \[ + P(X = 0) = 0.69^3 \approx 0.329 + \] + \[ + P(X \geq 2) = P(X = 2) + P(X = 3) = 3 \times 0.31^2 \times 0.69^1 + 0.31^3 \approx 0.229 + \] + + \item Il faut d'abord tracer le tableau résumant la loi de probabilité: + \begin{center} + \begin{tabular}{|c|*{4}{c|}} + \hline + Valeur & 0 & 1 & 2 & 3 \\ + \hline + Probabilité & $0.329$ & $0.443$ & $0.199$ &$0.03$ \\ + \hline + \end{tabular} + \end{center} + On peut alors calculer l'espérance + \[ + E[X] = 0 \times 0.329 + 1 \times 0.443 + 2 \times 0.199 + 3 \times 0.03 = 0.93 + \] + On peut donc estimer qu'il y aura en moyenne $0.93$ personnes qui feront sonner le portique sur les 3 personnes. + \end{enumerate} +\end{solution} + +\begin{exercise}[subtitle={Équation puissance}] + Résoudre les équations et inéquations suivantes + \begin{multicols}{2} + \begin{enumerate} + \item $10^x = 10$ + \item $11^x = 20$ + \item $0.09^x \leq 22$ + \item $6 \times 0.16^x = 45$ + \end{enumerate} + \end{multicols} +\end{exercise} + +\begin{solution} + Les solutions ci-dessous ne sont pas justifiée car l'ordinateur ne sait pas faire. Par contre, vous vous devez savoir justifier vos réponses! + \begin{enumerate} + \item $x = \log(10)$ + \item $x = \frac{\log(20)}{\log(11)}$ + \item Il faut faire attention quand on divise par un log car ce dernier peut être négatif ce qui est le cas ici. Il faut donc pense à changer le sens de l'inégalité. + + $x \geq \frac{\log(22)}{\log(0.09)}$ + + \item Il faut penser à faire la division à par $6$ avant d'utiliser le log car sinon, on ne peut pas utiliser la formule $\log(a^n) = n\times \log(a)$. + + $x = \frac{\log(7.5)}{\log(0.16)}$ + \end{enumerate} +\end{solution} + +\begin{exercise}[subtitle={Étude de fonctions}] + Soit $f(x) = 3x^3 - 225x^2 + 4896x + 16$ une fonction définie sur $\R$. + \begin{enumerate} + \item Calculer $f'(x)$ la dérivée de $f(x)$. + \item Calculer $f'(34)$ et $f'(16)$. + \item En déduire une forme factorisée de $f'(x)$. + \item Étudier le signe de $f'(x)$ et en déduire les variations de $f(x)$. + \item Est-ce que la fonction $f(x)$ admet un maximum ou un minimum? Si oui, calculer sa valeur. + \end{enumerate} +\end{exercise} + +\begin{solution} + \begin{enumerate} + \item Dérivée de $f(x)$: $f'(x) = 9x^2 - 450x + 4896$ + \item + \begin{align*} + f'(34) &= 9 \times 34^{2} - 450 \times 34 + 4896\\&= 9 \times 1156 - 15300 + 4896\\&= 10404 - 10404\\&= 0 + \end{align*} + \begin{align*} + f'(16) &= 9 \times 16^{2} - 450 \times 16 + 4896\\&= 9 \times 256 - 7200 + 4896\\&= 2304 - 2304\\&= 0 + \end{align*} + Donc $x = 34$ et $x=16$ sont des racines de $f'(x) = 9x^2 - 450x + 4896$. + \item On en déduit la forme factorisée suivante + \[ + f'(x) = 9 (x - 34)(x-16) + \] + \item Pas de correction disponible + \item À causes des branches extérieurs, la fonction $f(x)$ n'a pas de maximum ou de minimum. + \end{enumerate} +\end{solution} + + + +%\printsolutionstype{exercise} + + + +\end{document} + +%%% Local Variables: +%%% mode: latex +%%% TeX-master: "master" +%%% End: diff --git a/TST/DM/2102_DM2/TST3/12_2102_DM2.tex b/TST/DM/2102_DM2/TST3/12_2102_DM2.tex new file mode 100644 index 0000000..ce5eb1a --- /dev/null +++ b/TST/DM/2102_DM2/TST3/12_2102_DM2.tex @@ -0,0 +1,203 @@ +\documentclass[a5paper,10pt]{article} +\usepackage{myXsim} +\usepackage{tasks} + +% Title Page +\title{DM2 \hfill HENRIST Maxime} +\tribe{TST} +\date{\hfillÀ render pour le Mercredi 24 février} + +\xsimsetup{ + solution/print = false +} + +\begin{document} +\maketitle + +\begin{exercise}[subtitle={Loi binomiale}] + Trois personnes s'apprêtent à passer le portique de sécurité. On suppose que pour chaque personne la probabilité que le portique sonne est égale à $0.41$. + + Soit $X$ la variable aléatoire donnant le nombre de personnes faisant sonner le portique, parmi les 3 personnes de ce groupe. + \begin{enumerate} + \item Tracer l'arbre représentant le situation. + \item Justifier que $X$ suit une loi binomiale dont on précisera les paramètres. + \item Quelle est la probabilité qu'une seule personne fasse sonner le portique? + \item Calculer puis interpréter les probabilités suivantes + \[ + P(X = 0) \qquad \qquad P(X \geq 2) + \] + \item Calculer l'espérance de $X$ et interpréter le résultat. + \end{enumerate} +\end{exercise} + +\begin{solution} + \begin{enumerate} + \item + \begin{tikzpicture}[sloped] + \node {.} + child {node {$0$} + child {node {$0$} + child {node {$0$} + edge from parent + node[above] {0.59} + } + child {node {$1$} + edge from parent + node[above] {0.41} + } + edge from parent + node[above] {0.59} + } + child[missing] {} + child {node {$1$} + child {node {$0$} + edge from parent + node[above] {0.59} + } + child {node {$1$} + edge from parent + node[above] {0.41} + } + edge from parent + node[above] {0.59} + } + edge from parent + node[above] {0.59} + } + child[missing] {} + child[missing] {} + child[missing] {} + child { node {$1$} + child {node {$0$} + child {node {$0$} + edge from parent + node[above] {0.59} + } + child {node {$1$} + edge from parent + node[above] {0.41} + } + edge from parent + node[above] {0.59} + } + child[missing] {} + child {node {$1$} + child {node {$0$} + edge from parent + node[above] {0.59} + } + child {node {$1$} + edge from parent + node[above] {0.41} + } + edge from parent + node[above] {0.59} + } + edge from parent + node[above] {0.41} + } ; + \end{tikzpicture} + \item Chaque personne a 2 possibilités (1: fait sonner ou 2: ne fait pas sonner) et l'on fait passer 3 personnes ce qui correspond à une répétition identique et aléatoire. On peut donc modéliser la situation par une loi binomiale. + \[ + X \sim \mathcal{B}(3; 0.76) + \] + \item Probabilité qu'une seule personne fasse sonner le portique. On voit qu'il y a 3 branches qui correspondent à cette situation dont + \[ + P(X = 1) = 3 \times 0.41^1 \times 0.59^2 \approx 0.428 + \] + \item + \[ + P(X = 0) = 0.59^3 \approx 0.205 + \] + \[ + P(X \geq 2) = P(X = 2) + P(X = 3) = 3 \times 0.41^2 \times 0.59^1 + 0.41^3 \approx 0.367 + \] + + \item Il faut d'abord tracer le tableau résumant la loi de probabilité: + \begin{center} + \begin{tabular}{|c|*{4}{c|}} + \hline + Valeur & 0 & 1 & 2 & 3 \\ + \hline + Probabilité & $0.205$ & $0.428$ & $0.298$ &$0.069$ \\ + \hline + \end{tabular} + \end{center} + On peut alors calculer l'espérance + \[ + E[X] = 0 \times 0.205 + 1 \times 0.428 + 2 \times 0.298 + 3 \times 0.069 = 1.23 + \] + On peut donc estimer qu'il y aura en moyenne $1.23$ personnes qui feront sonner le portique sur les 3 personnes. + \end{enumerate} +\end{solution} + +\begin{exercise}[subtitle={Équation puissance}] + Résoudre les équations et inéquations suivantes + \begin{multicols}{2} + \begin{enumerate} + \item $10^x = 24$ + \item $14^x = 16$ + \item $0.35^x \leq 34$ + \item $8 \times 0.25^x = 36$ + \end{enumerate} + \end{multicols} +\end{exercise} + +\begin{solution} + Les solutions ci-dessous ne sont pas justifiée car l'ordinateur ne sait pas faire. Par contre, vous vous devez savoir justifier vos réponses! + \begin{enumerate} + \item $x = \log(24)$ + \item $x = \frac{\log(16)}{\log(14)}$ + \item Il faut faire attention quand on divise par un log car ce dernier peut être négatif ce qui est le cas ici. Il faut donc pense à changer le sens de l'inégalité. + + $x \geq \frac{\log(34)}{\log(0.35)}$ + + \item Il faut penser à faire la division à par $8$ avant d'utiliser le log car sinon, on ne peut pas utiliser la formule $\log(a^n) = n\times \log(a)$. + + $x = \frac{\log(4.5)}{\log(0.25)}$ + \end{enumerate} +\end{solution} + +\begin{exercise}[subtitle={Étude de fonctions}] + Soit $f(x) = 7x^3 - 346.5x^2 - 4914x + 30$ une fonction définie sur $\R$. + \begin{enumerate} + \item Calculer $f'(x)$ la dérivée de $f(x)$. + \item Calculer $f'(39)$ et $f'(-6)$. + \item En déduire une forme factorisée de $f'(x)$. + \item Étudier le signe de $f'(x)$ et en déduire les variations de $f(x)$. + \item Est-ce que la fonction $f(x)$ admet un maximum ou un minimum? Si oui, calculer sa valeur. + \end{enumerate} +\end{exercise} + +\begin{solution} + \begin{enumerate} + \item Dérivée de $f(x)$: $f'(x) = 21x^2 - 693x - 4914$ + \item + \begin{align*} + f'(39) &= 21 \times 39^{2} - 693 \times 39 - 4914\\&= 21 \times 1521 - 27027 - 4914\\&= 31941 - 31941\\&= 0 + \end{align*} + \begin{align*} + f'(-6) &= 21 \times - 6^{2} - 693(- 6) - 4914\\&= 21 \times 36 + 4158 - 4914\\&= 756 - 756\\&= 0 + \end{align*} + Donc $x = 39$ et $x=-6$ sont des racines de $f'(x) = 21x^2 - 693x - 4914$. + \item On en déduit la forme factorisée suivante + \[ + f'(x) = 21 (x - 39)(x--6) + \] + \item Pas de correction disponible + \item À causes des branches extérieurs, la fonction $f(x)$ n'a pas de maximum ou de minimum. + \end{enumerate} +\end{solution} + + + +%\printsolutionstype{exercise} + + + +\end{document} + +%%% Local Variables: +%%% mode: latex +%%% TeX-master: "master" +%%% End: diff --git a/TST/DM/2102_DM2/TST3/13_2102_DM2.tex b/TST/DM/2102_DM2/TST3/13_2102_DM2.tex new file mode 100644 index 0000000..b89aa68 --- /dev/null +++ b/TST/DM/2102_DM2/TST3/13_2102_DM2.tex @@ -0,0 +1,203 @@ +\documentclass[a5paper,10pt]{article} +\usepackage{myXsim} +\usepackage{tasks} + +% Title Page +\title{DM2 \hfill INFANTES Antoine} +\tribe{TST} +\date{\hfillÀ render pour le Mercredi 24 février} + +\xsimsetup{ + solution/print = false +} + +\begin{document} +\maketitle + +\begin{exercise}[subtitle={Loi binomiale}] + Trois personnes s'apprêtent à passer le portique de sécurité. On suppose que pour chaque personne la probabilité que le portique sonne est égale à $0.58$. + + Soit $X$ la variable aléatoire donnant le nombre de personnes faisant sonner le portique, parmi les 3 personnes de ce groupe. + \begin{enumerate} + \item Tracer l'arbre représentant le situation. + \item Justifier que $X$ suit une loi binomiale dont on précisera les paramètres. + \item Quelle est la probabilité qu'une seule personne fasse sonner le portique? + \item Calculer puis interpréter les probabilités suivantes + \[ + P(X = 0) \qquad \qquad P(X \geq 2) + \] + \item Calculer l'espérance de $X$ et interpréter le résultat. + \end{enumerate} +\end{exercise} + +\begin{solution} + \begin{enumerate} + \item + \begin{tikzpicture}[sloped] + \node {.} + child {node {$0$} + child {node {$0$} + child {node {$0$} + edge from parent + node[above] {0.42} + } + child {node {$1$} + edge from parent + node[above] {0.58} + } + edge from parent + node[above] {0.42} + } + child[missing] {} + child {node {$1$} + child {node {$0$} + edge from parent + node[above] {0.42} + } + child {node {$1$} + edge from parent + node[above] {0.58} + } + edge from parent + node[above] {0.42} + } + edge from parent + node[above] {0.42} + } + child[missing] {} + child[missing] {} + child[missing] {} + child { node {$1$} + child {node {$0$} + child {node {$0$} + edge from parent + node[above] {0.42} + } + child {node {$1$} + edge from parent + node[above] {0.58} + } + edge from parent + node[above] {0.42} + } + child[missing] {} + child {node {$1$} + child {node {$0$} + edge from parent + node[above] {0.42} + } + child {node {$1$} + edge from parent + node[above] {0.58} + } + edge from parent + node[above] {0.42} + } + edge from parent + node[above] {0.58} + } ; + \end{tikzpicture} + \item Chaque personne a 2 possibilités (1: fait sonner ou 2: ne fait pas sonner) et l'on fait passer 3 personnes ce qui correspond à une répétition identique et aléatoire. On peut donc modéliser la situation par une loi binomiale. + \[ + X \sim \mathcal{B}(3; 0.76) + \] + \item Probabilité qu'une seule personne fasse sonner le portique. On voit qu'il y a 3 branches qui correspondent à cette situation dont + \[ + P(X = 1) = 3 \times 0.58^1 \times 0.42^2 \approx 0.307 + \] + \item + \[ + P(X = 0) = 0.42^3 \approx 0.074 + \] + \[ + P(X \geq 2) = P(X = 2) + P(X = 3) = 3 \times 0.58^2 \times 0.42^1 + 0.58^3 \approx 0.619 + \] + + \item Il faut d'abord tracer le tableau résumant la loi de probabilité: + \begin{center} + \begin{tabular}{|c|*{4}{c|}} + \hline + Valeur & 0 & 1 & 2 & 3 \\ + \hline + Probabilité & $0.074$ & $0.307$ & $0.424$ &$0.195$ \\ + \hline + \end{tabular} + \end{center} + On peut alors calculer l'espérance + \[ + E[X] = 0 \times 0.074 + 1 \times 0.307 + 2 \times 0.424 + 3 \times 0.195 = 1.74 + \] + On peut donc estimer qu'il y aura en moyenne $1.74$ personnes qui feront sonner le portique sur les 3 personnes. + \end{enumerate} +\end{solution} + +\begin{exercise}[subtitle={Équation puissance}] + Résoudre les équations et inéquations suivantes + \begin{multicols}{2} + \begin{enumerate} + \item $10^x = 1$ + \item $11^x = 10$ + \item $0.52^x \leq 14$ + \item $5 \times 0.49^x = 45$ + \end{enumerate} + \end{multicols} +\end{exercise} + +\begin{solution} + Les solutions ci-dessous ne sont pas justifiée car l'ordinateur ne sait pas faire. Par contre, vous vous devez savoir justifier vos réponses! + \begin{enumerate} + \item $x = \log(1)$ + \item $x = \frac{\log(10)}{\log(11)}$ + \item Il faut faire attention quand on divise par un log car ce dernier peut être négatif ce qui est le cas ici. Il faut donc pense à changer le sens de l'inégalité. + + $x \geq \frac{\log(14)}{\log(0.52)}$ + + \item Il faut penser à faire la division à par $5$ avant d'utiliser le log car sinon, on ne peut pas utiliser la formule $\log(a^n) = n\times \log(a)$. + + $x = \frac{\log(9.0)}{\log(0.49)}$ + \end{enumerate} +\end{solution} + +\begin{exercise}[subtitle={Étude de fonctions}] + Soit $f(x) = - 9x^3 + 364.5x^2 + 756x - 21$ une fonction définie sur $\R$. + \begin{enumerate} + \item Calculer $f'(x)$ la dérivée de $f(x)$. + \item Calculer $f'(28)$ et $f'(-1)$. + \item En déduire une forme factorisée de $f'(x)$. + \item Étudier le signe de $f'(x)$ et en déduire les variations de $f(x)$. + \item Est-ce que la fonction $f(x)$ admet un maximum ou un minimum? Si oui, calculer sa valeur. + \end{enumerate} +\end{exercise} + +\begin{solution} + \begin{enumerate} + \item Dérivée de $f(x)$: $f'(x) = - 27x^2 + 729x + 756$ + \item + \begin{align*} + f'(28) &= - 27 \times 28^{2} + 729 \times 28 + 756\\&= - 27 \times 784 + 20412 + 756\\&= - 21168 + 21168\\&= 0 + \end{align*} + \begin{align*} + f'(-1) &= - 27 \times - 1^{2} + 729(- 1) + 756\\&= - 27 \times 1 - 729 + 756\\&= - 27 + 27\\&= 0 + \end{align*} + Donc $x = 28$ et $x=-1$ sont des racines de $f'(x) = - 27x^2 + 729x + 756$. + \item On en déduit la forme factorisée suivante + \[ + f'(x) = -27 (x - 28)(x--1) + \] + \item Pas de correction disponible + \item À causes des branches extérieurs, la fonction $f(x)$ n'a pas de maximum ou de minimum. + \end{enumerate} +\end{solution} + + + +%\printsolutionstype{exercise} + + + +\end{document} + +%%% Local Variables: +%%% mode: latex +%%% TeX-master: "master" +%%% End: diff --git a/TST/DM/2102_DM2/TST3/14_2102_DM2.tex b/TST/DM/2102_DM2/TST3/14_2102_DM2.tex new file mode 100644 index 0000000..ea3256f --- /dev/null +++ b/TST/DM/2102_DM2/TST3/14_2102_DM2.tex @@ -0,0 +1,203 @@ +\documentclass[a5paper,10pt]{article} +\usepackage{myXsim} +\usepackage{tasks} + +% Title Page +\title{DM2 \hfill MAGRO Robin} +\tribe{TST} +\date{\hfillÀ render pour le Mercredi 24 février} + +\xsimsetup{ + solution/print = false +} + +\begin{document} +\maketitle + +\begin{exercise}[subtitle={Loi binomiale}] + Trois personnes s'apprêtent à passer le portique de sécurité. On suppose que pour chaque personne la probabilité que le portique sonne est égale à $0.59$. + + Soit $X$ la variable aléatoire donnant le nombre de personnes faisant sonner le portique, parmi les 3 personnes de ce groupe. + \begin{enumerate} + \item Tracer l'arbre représentant le situation. + \item Justifier que $X$ suit une loi binomiale dont on précisera les paramètres. + \item Quelle est la probabilité qu'une seule personne fasse sonner le portique? + \item Calculer puis interpréter les probabilités suivantes + \[ + P(X = 0) \qquad \qquad P(X \geq 2) + \] + \item Calculer l'espérance de $X$ et interpréter le résultat. + \end{enumerate} +\end{exercise} + +\begin{solution} + \begin{enumerate} + \item + \begin{tikzpicture}[sloped] + \node {.} + child {node {$0$} + child {node {$0$} + child {node {$0$} + edge from parent + node[above] {0.41} + } + child {node {$1$} + edge from parent + node[above] {0.59} + } + edge from parent + node[above] {0.41} + } + child[missing] {} + child {node {$1$} + child {node {$0$} + edge from parent + node[above] {0.41} + } + child {node {$1$} + edge from parent + node[above] {0.59} + } + edge from parent + node[above] {0.41} + } + edge from parent + node[above] {0.41} + } + child[missing] {} + child[missing] {} + child[missing] {} + child { node {$1$} + child {node {$0$} + child {node {$0$} + edge from parent + node[above] {0.41} + } + child {node {$1$} + edge from parent + node[above] {0.59} + } + edge from parent + node[above] {0.41} + } + child[missing] {} + child {node {$1$} + child {node {$0$} + edge from parent + node[above] {0.41} + } + child {node {$1$} + edge from parent + node[above] {0.59} + } + edge from parent + node[above] {0.41} + } + edge from parent + node[above] {0.59} + } ; + \end{tikzpicture} + \item Chaque personne a 2 possibilités (1: fait sonner ou 2: ne fait pas sonner) et l'on fait passer 3 personnes ce qui correspond à une répétition identique et aléatoire. On peut donc modéliser la situation par une loi binomiale. + \[ + X \sim \mathcal{B}(3; 0.76) + \] + \item Probabilité qu'une seule personne fasse sonner le portique. On voit qu'il y a 3 branches qui correspondent à cette situation dont + \[ + P(X = 1) = 3 \times 0.59^1 \times 0.41^2 \approx 0.298 + \] + \item + \[ + P(X = 0) = 0.41^3 \approx 0.069 + \] + \[ + P(X \geq 2) = P(X = 2) + P(X = 3) = 3 \times 0.59^2 \times 0.41^1 + 0.59^3 \approx 0.633 + \] + + \item Il faut d'abord tracer le tableau résumant la loi de probabilité: + \begin{center} + \begin{tabular}{|c|*{4}{c|}} + \hline + Valeur & 0 & 1 & 2 & 3 \\ + \hline + Probabilité & $0.069$ & $0.298$ & $0.428$ &$0.205$ \\ + \hline + \end{tabular} + \end{center} + On peut alors calculer l'espérance + \[ + E[X] = 0 \times 0.069 + 1 \times 0.298 + 2 \times 0.428 + 3 \times 0.205 = 1.77 + \] + On peut donc estimer qu'il y aura en moyenne $1.77$ personnes qui feront sonner le portique sur les 3 personnes. + \end{enumerate} +\end{solution} + +\begin{exercise}[subtitle={Équation puissance}] + Résoudre les équations et inéquations suivantes + \begin{multicols}{2} + \begin{enumerate} + \item $10^x = 5$ + \item $15^x = 1$ + \item $0.57^x \leq 45$ + \item $3 \times 0.51^x = 21$ + \end{enumerate} + \end{multicols} +\end{exercise} + +\begin{solution} + Les solutions ci-dessous ne sont pas justifiée car l'ordinateur ne sait pas faire. Par contre, vous vous devez savoir justifier vos réponses! + \begin{enumerate} + \item $x = \log(5)$ + \item $x = \frac{\log(1)}{\log(15)}$ + \item Il faut faire attention quand on divise par un log car ce dernier peut être négatif ce qui est le cas ici. Il faut donc pense à changer le sens de l'inégalité. + + $x \geq \frac{\log(45)}{\log(0.57)}$ + + \item Il faut penser à faire la division à par $3$ avant d'utiliser le log car sinon, on ne peut pas utiliser la formule $\log(a^n) = n\times \log(a)$. + + $x = \frac{\log(7.0)}{\log(0.51)}$ + \end{enumerate} +\end{solution} + +\begin{exercise}[subtitle={Étude de fonctions}] + Soit $f(x) = - x^3 + 43.5x^2 + 510x - 37$ une fonction définie sur $\R$. + \begin{enumerate} + \item Calculer $f'(x)$ la dérivée de $f(x)$. + \item Calculer $f'(34)$ et $f'(-5)$. + \item En déduire une forme factorisée de $f'(x)$. + \item Étudier le signe de $f'(x)$ et en déduire les variations de $f(x)$. + \item Est-ce que la fonction $f(x)$ admet un maximum ou un minimum? Si oui, calculer sa valeur. + \end{enumerate} +\end{exercise} + +\begin{solution} + \begin{enumerate} + \item Dérivée de $f(x)$: $f'(x) = - 3x^2 + 87x + 510$ + \item + \begin{align*} + f'(34) &= - 3 \times 34^{2} + 87 \times 34 + 510\\&= - 3 \times 1156 + 2958 + 510\\&= - 3468 + 3468\\&= 0 + \end{align*} + \begin{align*} + f'(-5) &= - 3 \times - 5^{2} + 87(- 5) + 510\\&= - 3 \times 25 - 435 + 510\\&= - 75 + 75\\&= 0 + \end{align*} + Donc $x = 34$ et $x=-5$ sont des racines de $f'(x) = - 3x^2 + 87x + 510$. + \item On en déduit la forme factorisée suivante + \[ + f'(x) = -3 (x - 34)(x--5) + \] + \item Pas de correction disponible + \item À causes des branches extérieurs, la fonction $f(x)$ n'a pas de maximum ou de minimum. + \end{enumerate} +\end{solution} + + + +%\printsolutionstype{exercise} + + + +\end{document} + +%%% Local Variables: +%%% mode: latex +%%% TeX-master: "master" +%%% End: diff --git a/TST/DM/2102_DM2/TST3/15_2102_DM2.tex b/TST/DM/2102_DM2/TST3/15_2102_DM2.tex new file mode 100644 index 0000000..e12353a --- /dev/null +++ b/TST/DM/2102_DM2/TST3/15_2102_DM2.tex @@ -0,0 +1,203 @@ +\documentclass[a5paper,10pt]{article} +\usepackage{myXsim} +\usepackage{tasks} + +% Title Page +\title{DM2 \hfill MORFIN Chloé} +\tribe{TST} +\date{\hfillÀ render pour le Mercredi 24 février} + +\xsimsetup{ + solution/print = false +} + +\begin{document} +\maketitle + +\begin{exercise}[subtitle={Loi binomiale}] + Trois personnes s'apprêtent à passer le portique de sécurité. On suppose que pour chaque personne la probabilité que le portique sonne est égale à $0.96$. + + Soit $X$ la variable aléatoire donnant le nombre de personnes faisant sonner le portique, parmi les 3 personnes de ce groupe. + \begin{enumerate} + \item Tracer l'arbre représentant le situation. + \item Justifier que $X$ suit une loi binomiale dont on précisera les paramètres. + \item Quelle est la probabilité qu'une seule personne fasse sonner le portique? + \item Calculer puis interpréter les probabilités suivantes + \[ + P(X = 0) \qquad \qquad P(X \geq 2) + \] + \item Calculer l'espérance de $X$ et interpréter le résultat. + \end{enumerate} +\end{exercise} + +\begin{solution} + \begin{enumerate} + \item + \begin{tikzpicture}[sloped] + \node {.} + child {node {$0$} + child {node {$0$} + child {node {$0$} + edge from parent + node[above] {0.04} + } + child {node {$1$} + edge from parent + node[above] {0.96} + } + edge from parent + node[above] {0.04} + } + child[missing] {} + child {node {$1$} + child {node {$0$} + edge from parent + node[above] {0.04} + } + child {node {$1$} + edge from parent + node[above] {0.96} + } + edge from parent + node[above] {0.04} + } + edge from parent + node[above] {0.04} + } + child[missing] {} + child[missing] {} + child[missing] {} + child { node {$1$} + child {node {$0$} + child {node {$0$} + edge from parent + node[above] {0.04} + } + child {node {$1$} + edge from parent + node[above] {0.96} + } + edge from parent + node[above] {0.04} + } + child[missing] {} + child {node {$1$} + child {node {$0$} + edge from parent + node[above] {0.04} + } + child {node {$1$} + edge from parent + node[above] {0.96} + } + edge from parent + node[above] {0.04} + } + edge from parent + node[above] {0.96} + } ; + \end{tikzpicture} + \item Chaque personne a 2 possibilités (1: fait sonner ou 2: ne fait pas sonner) et l'on fait passer 3 personnes ce qui correspond à une répétition identique et aléatoire. On peut donc modéliser la situation par une loi binomiale. + \[ + X \sim \mathcal{B}(3; 0.76) + \] + \item Probabilité qu'une seule personne fasse sonner le portique. On voit qu'il y a 3 branches qui correspondent à cette situation dont + \[ + P(X = 1) = 3 \times 0.96^1 \times 0.04^2 \approx 0.005 + \] + \item + \[ + P(X = 0) = 0.04^3 \approx 0.0 + \] + \[ + P(X \geq 2) = P(X = 2) + P(X = 3) = 3 \times 0.96^2 \times 0.04^1 + 0.96^3 \approx 0.996 + \] + + \item Il faut d'abord tracer le tableau résumant la loi de probabilité: + \begin{center} + \begin{tabular}{|c|*{4}{c|}} + \hline + Valeur & 0 & 1 & 2 & 3 \\ + \hline + Probabilité & $0.0$ & $0.005$ & $0.111$ &$0.885$ \\ + \hline + \end{tabular} + \end{center} + On peut alors calculer l'espérance + \[ + E[X] = 0 \times 0.0 + 1 \times 0.005 + 2 \times 0.111 + 3 \times 0.885 = 2.88 + \] + On peut donc estimer qu'il y aura en moyenne $2.88$ personnes qui feront sonner le portique sur les 3 personnes. + \end{enumerate} +\end{solution} + +\begin{exercise}[subtitle={Équation puissance}] + Résoudre les équations et inéquations suivantes + \begin{multicols}{2} + \begin{enumerate} + \item $10^x = 17$ + \item $3^x = 31$ + \item $0.44^x \leq 45$ + \item $9 \times 1.0^x = 17$ + \end{enumerate} + \end{multicols} +\end{exercise} + +\begin{solution} + Les solutions ci-dessous ne sont pas justifiée car l'ordinateur ne sait pas faire. Par contre, vous vous devez savoir justifier vos réponses! + \begin{enumerate} + \item $x = \log(17)$ + \item $x = \frac{\log(31)}{\log(3)}$ + \item Il faut faire attention quand on divise par un log car ce dernier peut être négatif ce qui est le cas ici. Il faut donc pense à changer le sens de l'inégalité. + + $x \geq \frac{\log(45)}{\log(0.44)}$ + + \item Il faut penser à faire la division à par $9$ avant d'utiliser le log car sinon, on ne peut pas utiliser la formule $\log(a^n) = n\times \log(a)$. + + $x = \frac{\log(1.89)}{\log(1.0)}$ + \end{enumerate} +\end{solution} + +\begin{exercise}[subtitle={Étude de fonctions}] + Soit $f(x) = - 8x^3 + 324x^2 + 1392x - 9$ une fonction définie sur $\R$. + \begin{enumerate} + \item Calculer $f'(x)$ la dérivée de $f(x)$. + \item Calculer $f'(29)$ et $f'(-2)$. + \item En déduire une forme factorisée de $f'(x)$. + \item Étudier le signe de $f'(x)$ et en déduire les variations de $f(x)$. + \item Est-ce que la fonction $f(x)$ admet un maximum ou un minimum? Si oui, calculer sa valeur. + \end{enumerate} +\end{exercise} + +\begin{solution} + \begin{enumerate} + \item Dérivée de $f(x)$: $f'(x) = - 24x^2 + 648x + 1392$ + \item + \begin{align*} + f'(29) &= - 24 \times 29^{2} + 648 \times 29 + 1392\\&= - 24 \times 841 + 18792 + 1392\\&= - 20184 + 20184\\&= 0 + \end{align*} + \begin{align*} + f'(-2) &= - 24 \times - 2^{2} + 648(- 2) + 1392\\&= - 24 \times 4 - 1296 + 1392\\&= - 96 + 96\\&= 0 + \end{align*} + Donc $x = 29$ et $x=-2$ sont des racines de $f'(x) = - 24x^2 + 648x + 1392$. + \item On en déduit la forme factorisée suivante + \[ + f'(x) = -24 (x - 29)(x--2) + \] + \item Pas de correction disponible + \item À causes des branches extérieurs, la fonction $f(x)$ n'a pas de maximum ou de minimum. + \end{enumerate} +\end{solution} + + + +%\printsolutionstype{exercise} + + + +\end{document} + +%%% Local Variables: +%%% mode: latex +%%% TeX-master: "master" +%%% End: diff --git a/TST/DM/2102_DM2/TST3/16_2102_DM2.tex b/TST/DM/2102_DM2/TST3/16_2102_DM2.tex new file mode 100644 index 0000000..995b894 --- /dev/null +++ b/TST/DM/2102_DM2/TST3/16_2102_DM2.tex @@ -0,0 +1,203 @@ +\documentclass[a5paper,10pt]{article} +\usepackage{myXsim} +\usepackage{tasks} + +% Title Page +\title{DM2 \hfill PERES RAMALHO Emeric} +\tribe{TST} +\date{\hfillÀ render pour le Mercredi 24 février} + +\xsimsetup{ + solution/print = false +} + +\begin{document} +\maketitle + +\begin{exercise}[subtitle={Loi binomiale}] + Trois personnes s'apprêtent à passer le portique de sécurité. On suppose que pour chaque personne la probabilité que le portique sonne est égale à $0.19$. + + Soit $X$ la variable aléatoire donnant le nombre de personnes faisant sonner le portique, parmi les 3 personnes de ce groupe. + \begin{enumerate} + \item Tracer l'arbre représentant le situation. + \item Justifier que $X$ suit une loi binomiale dont on précisera les paramètres. + \item Quelle est la probabilité qu'une seule personne fasse sonner le portique? + \item Calculer puis interpréter les probabilités suivantes + \[ + P(X = 0) \qquad \qquad P(X \geq 2) + \] + \item Calculer l'espérance de $X$ et interpréter le résultat. + \end{enumerate} +\end{exercise} + +\begin{solution} + \begin{enumerate} + \item + \begin{tikzpicture}[sloped] + \node {.} + child {node {$0$} + child {node {$0$} + child {node {$0$} + edge from parent + node[above] {0.81} + } + child {node {$1$} + edge from parent + node[above] {0.19} + } + edge from parent + node[above] {0.81} + } + child[missing] {} + child {node {$1$} + child {node {$0$} + edge from parent + node[above] {0.81} + } + child {node {$1$} + edge from parent + node[above] {0.19} + } + edge from parent + node[above] {0.81} + } + edge from parent + node[above] {0.81} + } + child[missing] {} + child[missing] {} + child[missing] {} + child { node {$1$} + child {node {$0$} + child {node {$0$} + edge from parent + node[above] {0.81} + } + child {node {$1$} + edge from parent + node[above] {0.19} + } + edge from parent + node[above] {0.81} + } + child[missing] {} + child {node {$1$} + child {node {$0$} + edge from parent + node[above] {0.81} + } + child {node {$1$} + edge from parent + node[above] {0.19} + } + edge from parent + node[above] {0.81} + } + edge from parent + node[above] {0.19} + } ; + \end{tikzpicture} + \item Chaque personne a 2 possibilités (1: fait sonner ou 2: ne fait pas sonner) et l'on fait passer 3 personnes ce qui correspond à une répétition identique et aléatoire. On peut donc modéliser la situation par une loi binomiale. + \[ + X \sim \mathcal{B}(3; 0.76) + \] + \item Probabilité qu'une seule personne fasse sonner le portique. On voit qu'il y a 3 branches qui correspondent à cette situation dont + \[ + P(X = 1) = 3 \times 0.19^1 \times 0.81^2 \approx 0.374 + \] + \item + \[ + P(X = 0) = 0.81^3 \approx 0.531 + \] + \[ + P(X \geq 2) = P(X = 2) + P(X = 3) = 3 \times 0.19^2 \times 0.81^1 + 0.19^3 \approx 0.095 + \] + + \item Il faut d'abord tracer le tableau résumant la loi de probabilité: + \begin{center} + \begin{tabular}{|c|*{4}{c|}} + \hline + Valeur & 0 & 1 & 2 & 3 \\ + \hline + Probabilité & $0.531$ & $0.374$ & $0.088$ &$0.007$ \\ + \hline + \end{tabular} + \end{center} + On peut alors calculer l'espérance + \[ + E[X] = 0 \times 0.531 + 1 \times 0.374 + 2 \times 0.088 + 3 \times 0.007 = 0.57 + \] + On peut donc estimer qu'il y aura en moyenne $0.57$ personnes qui feront sonner le portique sur les 3 personnes. + \end{enumerate} +\end{solution} + +\begin{exercise}[subtitle={Équation puissance}] + Résoudre les équations et inéquations suivantes + \begin{multicols}{2} + \begin{enumerate} + \item $10^x = 5$ + \item $4^x = 39$ + \item $0.95^x \leq 21$ + \item $10 \times 0.74^x = 16$ + \end{enumerate} + \end{multicols} +\end{exercise} + +\begin{solution} + Les solutions ci-dessous ne sont pas justifiée car l'ordinateur ne sait pas faire. Par contre, vous vous devez savoir justifier vos réponses! + \begin{enumerate} + \item $x = \log(5)$ + \item $x = \frac{\log(39)}{\log(4)}$ + \item Il faut faire attention quand on divise par un log car ce dernier peut être négatif ce qui est le cas ici. Il faut donc pense à changer le sens de l'inégalité. + + $x \geq \frac{\log(21)}{\log(0.95)}$ + + \item Il faut penser à faire la division à par $10$ avant d'utiliser le log car sinon, on ne peut pas utiliser la formule $\log(a^n) = n\times \log(a)$. + + $x = \frac{\log(1.6)}{\log(0.74)}$ + \end{enumerate} +\end{solution} + +\begin{exercise}[subtitle={Étude de fonctions}] + Soit $f(x) = - 3x^3 + 108x^2 - 396x - 26$ une fonction définie sur $\R$. + \begin{enumerate} + \item Calculer $f'(x)$ la dérivée de $f(x)$. + \item Calculer $f'(22)$ et $f'(2)$. + \item En déduire une forme factorisée de $f'(x)$. + \item Étudier le signe de $f'(x)$ et en déduire les variations de $f(x)$. + \item Est-ce que la fonction $f(x)$ admet un maximum ou un minimum? Si oui, calculer sa valeur. + \end{enumerate} +\end{exercise} + +\begin{solution} + \begin{enumerate} + \item Dérivée de $f(x)$: $f'(x) = - 9x^2 + 216x - 396$ + \item + \begin{align*} + f'(22) &= - 9 \times 22^{2} + 216 \times 22 - 396\\&= - 9 \times 484 + 4752 - 396\\&= - 4356 + 4356\\&= 0 + \end{align*} + \begin{align*} + f'(2) &= - 9 \times 2^{2} + 216 \times 2 - 396\\&= - 9 \times 4 + 432 - 396\\&= - 36 + 36\\&= 0 + \end{align*} + Donc $x = 22$ et $x=2$ sont des racines de $f'(x) = - 9x^2 + 216x - 396$. + \item On en déduit la forme factorisée suivante + \[ + f'(x) = -9 (x - 22)(x-2) + \] + \item Pas de correction disponible + \item À causes des branches extérieurs, la fonction $f(x)$ n'a pas de maximum ou de minimum. + \end{enumerate} +\end{solution} + + + +%\printsolutionstype{exercise} + + + +\end{document} + +%%% Local Variables: +%%% mode: latex +%%% TeX-master: "master" +%%% End: diff --git a/TST/DM/2102_DM2/TST3/17_2102_DM2.tex b/TST/DM/2102_DM2/TST3/17_2102_DM2.tex new file mode 100644 index 0000000..2a9ece1 --- /dev/null +++ b/TST/DM/2102_DM2/TST3/17_2102_DM2.tex @@ -0,0 +1,203 @@ +\documentclass[a5paper,10pt]{article} +\usepackage{myXsim} +\usepackage{tasks} + +% Title Page +\title{DM2 \hfill RADOUAA Saleh} +\tribe{TST} +\date{\hfillÀ render pour le Mercredi 24 février} + +\xsimsetup{ + solution/print = false +} + +\begin{document} +\maketitle + +\begin{exercise}[subtitle={Loi binomiale}] + Trois personnes s'apprêtent à passer le portique de sécurité. On suppose que pour chaque personne la probabilité que le portique sonne est égale à $0.08$. + + Soit $X$ la variable aléatoire donnant le nombre de personnes faisant sonner le portique, parmi les 3 personnes de ce groupe. + \begin{enumerate} + \item Tracer l'arbre représentant le situation. + \item Justifier que $X$ suit une loi binomiale dont on précisera les paramètres. + \item Quelle est la probabilité qu'une seule personne fasse sonner le portique? + \item Calculer puis interpréter les probabilités suivantes + \[ + P(X = 0) \qquad \qquad P(X \geq 2) + \] + \item Calculer l'espérance de $X$ et interpréter le résultat. + \end{enumerate} +\end{exercise} + +\begin{solution} + \begin{enumerate} + \item + \begin{tikzpicture}[sloped] + \node {.} + child {node {$0$} + child {node {$0$} + child {node {$0$} + edge from parent + node[above] {0.92} + } + child {node {$1$} + edge from parent + node[above] {0.08} + } + edge from parent + node[above] {0.92} + } + child[missing] {} + child {node {$1$} + child {node {$0$} + edge from parent + node[above] {0.92} + } + child {node {$1$} + edge from parent + node[above] {0.08} + } + edge from parent + node[above] {0.92} + } + edge from parent + node[above] {0.92} + } + child[missing] {} + child[missing] {} + child[missing] {} + child { node {$1$} + child {node {$0$} + child {node {$0$} + edge from parent + node[above] {0.92} + } + child {node {$1$} + edge from parent + node[above] {0.08} + } + edge from parent + node[above] {0.92} + } + child[missing] {} + child {node {$1$} + child {node {$0$} + edge from parent + node[above] {0.92} + } + child {node {$1$} + edge from parent + node[above] {0.08} + } + edge from parent + node[above] {0.92} + } + edge from parent + node[above] {0.08} + } ; + \end{tikzpicture} + \item Chaque personne a 2 possibilités (1: fait sonner ou 2: ne fait pas sonner) et l'on fait passer 3 personnes ce qui correspond à une répétition identique et aléatoire. On peut donc modéliser la situation par une loi binomiale. + \[ + X \sim \mathcal{B}(3; 0.76) + \] + \item Probabilité qu'une seule personne fasse sonner le portique. On voit qu'il y a 3 branches qui correspondent à cette situation dont + \[ + P(X = 1) = 3 \times 0.08^1 \times 0.92^2 \approx 0.203 + \] + \item + \[ + P(X = 0) = 0.92^3 \approx 0.779 + \] + \[ + P(X \geq 2) = P(X = 2) + P(X = 3) = 3 \times 0.08^2 \times 0.92^1 + 0.08^3 \approx 0.019 + \] + + \item Il faut d'abord tracer le tableau résumant la loi de probabilité: + \begin{center} + \begin{tabular}{|c|*{4}{c|}} + \hline + Valeur & 0 & 1 & 2 & 3 \\ + \hline + Probabilité & $0.779$ & $0.203$ & $0.018$ &$0.001$ \\ + \hline + \end{tabular} + \end{center} + On peut alors calculer l'espérance + \[ + E[X] = 0 \times 0.779 + 1 \times 0.203 + 2 \times 0.018 + 3 \times 0.001 = 0.24 + \] + On peut donc estimer qu'il y aura en moyenne $0.24$ personnes qui feront sonner le portique sur les 3 personnes. + \end{enumerate} +\end{solution} + +\begin{exercise}[subtitle={Équation puissance}] + Résoudre les équations et inéquations suivantes + \begin{multicols}{2} + \begin{enumerate} + \item $10^x = 8$ + \item $17^x = 11$ + \item $0.84^x \leq 28$ + \item $8 \times 0.96^x = 22$ + \end{enumerate} + \end{multicols} +\end{exercise} + +\begin{solution} + Les solutions ci-dessous ne sont pas justifiée car l'ordinateur ne sait pas faire. Par contre, vous vous devez savoir justifier vos réponses! + \begin{enumerate} + \item $x = \log(8)$ + \item $x = \frac{\log(11)}{\log(17)}$ + \item Il faut faire attention quand on divise par un log car ce dernier peut être négatif ce qui est le cas ici. Il faut donc pense à changer le sens de l'inégalité. + + $x \geq \frac{\log(28)}{\log(0.84)}$ + + \item Il faut penser à faire la division à par $8$ avant d'utiliser le log car sinon, on ne peut pas utiliser la formule $\log(a^n) = n\times \log(a)$. + + $x = \frac{\log(2.75)}{\log(0.96)}$ + \end{enumerate} +\end{solution} + +\begin{exercise}[subtitle={Étude de fonctions}] + Soit $f(x) = 3x^3 - 207x^2 + 1161x + 46$ une fonction définie sur $\R$. + \begin{enumerate} + \item Calculer $f'(x)$ la dérivée de $f(x)$. + \item Calculer $f'(43)$ et $f'(3)$. + \item En déduire une forme factorisée de $f'(x)$. + \item Étudier le signe de $f'(x)$ et en déduire les variations de $f(x)$. + \item Est-ce que la fonction $f(x)$ admet un maximum ou un minimum? Si oui, calculer sa valeur. + \end{enumerate} +\end{exercise} + +\begin{solution} + \begin{enumerate} + \item Dérivée de $f(x)$: $f'(x) = 9x^2 - 414x + 1161$ + \item + \begin{align*} + f'(43) &= 9 \times 43^{2} - 414 \times 43 + 1161\\&= 9 \times 1849 - 17802 + 1161\\&= 16641 - 16641\\&= 0 + \end{align*} + \begin{align*} + f'(3) &= 9 \times 3^{2} - 414 \times 3 + 1161\\&= 9 \times 9 - 1242 + 1161\\&= 81 - 81\\&= 0 + \end{align*} + Donc $x = 43$ et $x=3$ sont des racines de $f'(x) = 9x^2 - 414x + 1161$. + \item On en déduit la forme factorisée suivante + \[ + f'(x) = 9 (x - 43)(x-3) + \] + \item Pas de correction disponible + \item À causes des branches extérieurs, la fonction $f(x)$ n'a pas de maximum ou de minimum. + \end{enumerate} +\end{solution} + + + +%\printsolutionstype{exercise} + + + +\end{document} + +%%% Local Variables: +%%% mode: latex +%%% TeX-master: "master" +%%% End: diff --git a/TST/DM/2102_DM2/TST3/18_2102_DM2.tex b/TST/DM/2102_DM2/TST3/18_2102_DM2.tex new file mode 100644 index 0000000..17ef934 --- /dev/null +++ b/TST/DM/2102_DM2/TST3/18_2102_DM2.tex @@ -0,0 +1,203 @@ +\documentclass[a5paper,10pt]{article} +\usepackage{myXsim} +\usepackage{tasks} + +% Title Page +\title{DM2 \hfill TAY Ummuhan} +\tribe{TST} +\date{\hfillÀ render pour le Mercredi 24 février} + +\xsimsetup{ + solution/print = false +} + +\begin{document} +\maketitle + +\begin{exercise}[subtitle={Loi binomiale}] + Trois personnes s'apprêtent à passer le portique de sécurité. On suppose que pour chaque personne la probabilité que le portique sonne est égale à $0.72$. + + Soit $X$ la variable aléatoire donnant le nombre de personnes faisant sonner le portique, parmi les 3 personnes de ce groupe. + \begin{enumerate} + \item Tracer l'arbre représentant le situation. + \item Justifier que $X$ suit une loi binomiale dont on précisera les paramètres. + \item Quelle est la probabilité qu'une seule personne fasse sonner le portique? + \item Calculer puis interpréter les probabilités suivantes + \[ + P(X = 0) \qquad \qquad P(X \geq 2) + \] + \item Calculer l'espérance de $X$ et interpréter le résultat. + \end{enumerate} +\end{exercise} + +\begin{solution} + \begin{enumerate} + \item + \begin{tikzpicture}[sloped] + \node {.} + child {node {$0$} + child {node {$0$} + child {node {$0$} + edge from parent + node[above] {0.28} + } + child {node {$1$} + edge from parent + node[above] {0.72} + } + edge from parent + node[above] {0.28} + } + child[missing] {} + child {node {$1$} + child {node {$0$} + edge from parent + node[above] {0.28} + } + child {node {$1$} + edge from parent + node[above] {0.72} + } + edge from parent + node[above] {0.28} + } + edge from parent + node[above] {0.28} + } + child[missing] {} + child[missing] {} + child[missing] {} + child { node {$1$} + child {node {$0$} + child {node {$0$} + edge from parent + node[above] {0.28} + } + child {node {$1$} + edge from parent + node[above] {0.72} + } + edge from parent + node[above] {0.28} + } + child[missing] {} + child {node {$1$} + child {node {$0$} + edge from parent + node[above] {0.28} + } + child {node {$1$} + edge from parent + node[above] {0.72} + } + edge from parent + node[above] {0.28} + } + edge from parent + node[above] {0.72} + } ; + \end{tikzpicture} + \item Chaque personne a 2 possibilités (1: fait sonner ou 2: ne fait pas sonner) et l'on fait passer 3 personnes ce qui correspond à une répétition identique et aléatoire. On peut donc modéliser la situation par une loi binomiale. + \[ + X \sim \mathcal{B}(3; 0.76) + \] + \item Probabilité qu'une seule personne fasse sonner le portique. On voit qu'il y a 3 branches qui correspondent à cette situation dont + \[ + P(X = 1) = 3 \times 0.72^1 \times 0.28^2 \approx 0.169 + \] + \item + \[ + P(X = 0) = 0.28^3 \approx 0.022 + \] + \[ + P(X \geq 2) = P(X = 2) + P(X = 3) = 3 \times 0.72^2 \times 0.28^1 + 0.72^3 \approx 0.808 + \] + + \item Il faut d'abord tracer le tableau résumant la loi de probabilité: + \begin{center} + \begin{tabular}{|c|*{4}{c|}} + \hline + Valeur & 0 & 1 & 2 & 3 \\ + \hline + Probabilité & $0.022$ & $0.169$ & $0.435$ &$0.373$ \\ + \hline + \end{tabular} + \end{center} + On peut alors calculer l'espérance + \[ + E[X] = 0 \times 0.022 + 1 \times 0.169 + 2 \times 0.435 + 3 \times 0.373 = 2.16 + \] + On peut donc estimer qu'il y aura en moyenne $2.16$ personnes qui feront sonner le portique sur les 3 personnes. + \end{enumerate} +\end{solution} + +\begin{exercise}[subtitle={Équation puissance}] + Résoudre les équations et inéquations suivantes + \begin{multicols}{2} + \begin{enumerate} + \item $10^x = 41$ + \item $2^x = 7$ + \item $0.44^x \leq 20$ + \item $3 \times 0.08^x = 24$ + \end{enumerate} + \end{multicols} +\end{exercise} + +\begin{solution} + Les solutions ci-dessous ne sont pas justifiée car l'ordinateur ne sait pas faire. Par contre, vous vous devez savoir justifier vos réponses! + \begin{enumerate} + \item $x = \log(41)$ + \item $x = \frac{\log(7)}{\log(2)}$ + \item Il faut faire attention quand on divise par un log car ce dernier peut être négatif ce qui est le cas ici. Il faut donc pense à changer le sens de l'inégalité. + + $x \geq \frac{\log(20)}{\log(0.44)}$ + + \item Il faut penser à faire la division à par $3$ avant d'utiliser le log car sinon, on ne peut pas utiliser la formule $\log(a^n) = n\times \log(a)$. + + $x = \frac{\log(8.0)}{\log(0.08)}$ + \end{enumerate} +\end{solution} + +\begin{exercise}[subtitle={Étude de fonctions}] + Soit $f(x) = 7x^3 - 472.5x^2 + 10374x + 15$ une fonction définie sur $\R$. + \begin{enumerate} + \item Calculer $f'(x)$ la dérivée de $f(x)$. + \item Calculer $f'(26)$ et $f'(19)$. + \item En déduire une forme factorisée de $f'(x)$. + \item Étudier le signe de $f'(x)$ et en déduire les variations de $f(x)$. + \item Est-ce que la fonction $f(x)$ admet un maximum ou un minimum? Si oui, calculer sa valeur. + \end{enumerate} +\end{exercise} + +\begin{solution} + \begin{enumerate} + \item Dérivée de $f(x)$: $f'(x) = 21x^2 - 945x + 10374$ + \item + \begin{align*} + f'(26) &= 21 \times 26^{2} - 945 \times 26 + 10374\\&= 21 \times 676 - 24570 + 10374\\&= 14196 - 14196\\&= 0 + \end{align*} + \begin{align*} + f'(19) &= 21 \times 19^{2} - 945 \times 19 + 10374\\&= 21 \times 361 - 17955 + 10374\\&= 7581 - 7581\\&= 0 + \end{align*} + Donc $x = 26$ et $x=19$ sont des racines de $f'(x) = 21x^2 - 945x + 10374$. + \item On en déduit la forme factorisée suivante + \[ + f'(x) = 21 (x - 26)(x-19) + \] + \item Pas de correction disponible + \item À causes des branches extérieurs, la fonction $f(x)$ n'a pas de maximum ou de minimum. + \end{enumerate} +\end{solution} + + + +%\printsolutionstype{exercise} + + + +\end{document} + +%%% Local Variables: +%%% mode: latex +%%% TeX-master: "master" +%%% End: diff --git a/TST/DM/2102_DM2/TST3/19_2102_DM2.tex b/TST/DM/2102_DM2/TST3/19_2102_DM2.tex new file mode 100644 index 0000000..93837bc --- /dev/null +++ b/TST/DM/2102_DM2/TST3/19_2102_DM2.tex @@ -0,0 +1,203 @@ +\documentclass[a5paper,10pt]{article} +\usepackage{myXsim} +\usepackage{tasks} + +% Title Page +\title{DM2 \hfill VIALON-DUPERRON Victorien} +\tribe{TST} +\date{\hfillÀ render pour le Mercredi 24 février} + +\xsimsetup{ + solution/print = false +} + +\begin{document} +\maketitle + +\begin{exercise}[subtitle={Loi binomiale}] + Trois personnes s'apprêtent à passer le portique de sécurité. On suppose que pour chaque personne la probabilité que le portique sonne est égale à $0.18$. + + Soit $X$ la variable aléatoire donnant le nombre de personnes faisant sonner le portique, parmi les 3 personnes de ce groupe. + \begin{enumerate} + \item Tracer l'arbre représentant le situation. + \item Justifier que $X$ suit une loi binomiale dont on précisera les paramètres. + \item Quelle est la probabilité qu'une seule personne fasse sonner le portique? + \item Calculer puis interpréter les probabilités suivantes + \[ + P(X = 0) \qquad \qquad P(X \geq 2) + \] + \item Calculer l'espérance de $X$ et interpréter le résultat. + \end{enumerate} +\end{exercise} + +\begin{solution} + \begin{enumerate} + \item + \begin{tikzpicture}[sloped] + \node {.} + child {node {$0$} + child {node {$0$} + child {node {$0$} + edge from parent + node[above] {0.82} + } + child {node {$1$} + edge from parent + node[above] {0.18} + } + edge from parent + node[above] {0.82} + } + child[missing] {} + child {node {$1$} + child {node {$0$} + edge from parent + node[above] {0.82} + } + child {node {$1$} + edge from parent + node[above] {0.18} + } + edge from parent + node[above] {0.82} + } + edge from parent + node[above] {0.82} + } + child[missing] {} + child[missing] {} + child[missing] {} + child { node {$1$} + child {node {$0$} + child {node {$0$} + edge from parent + node[above] {0.82} + } + child {node {$1$} + edge from parent + node[above] {0.18} + } + edge from parent + node[above] {0.82} + } + child[missing] {} + child {node {$1$} + child {node {$0$} + edge from parent + node[above] {0.82} + } + child {node {$1$} + edge from parent + node[above] {0.18} + } + edge from parent + node[above] {0.82} + } + edge from parent + node[above] {0.18} + } ; + \end{tikzpicture} + \item Chaque personne a 2 possibilités (1: fait sonner ou 2: ne fait pas sonner) et l'on fait passer 3 personnes ce qui correspond à une répétition identique et aléatoire. On peut donc modéliser la situation par une loi binomiale. + \[ + X \sim \mathcal{B}(3; 0.76) + \] + \item Probabilité qu'une seule personne fasse sonner le portique. On voit qu'il y a 3 branches qui correspondent à cette situation dont + \[ + P(X = 1) = 3 \times 0.18^1 \times 0.82^2 \approx 0.363 + \] + \item + \[ + P(X = 0) = 0.82^3 \approx 0.551 + \] + \[ + P(X \geq 2) = P(X = 2) + P(X = 3) = 3 \times 0.18^2 \times 0.82^1 + 0.18^3 \approx 0.086 + \] + + \item Il faut d'abord tracer le tableau résumant la loi de probabilité: + \begin{center} + \begin{tabular}{|c|*{4}{c|}} + \hline + Valeur & 0 & 1 & 2 & 3 \\ + \hline + Probabilité & $0.551$ & $0.363$ & $0.08$ &$0.006$ \\ + \hline + \end{tabular} + \end{center} + On peut alors calculer l'espérance + \[ + E[X] = 0 \times 0.551 + 1 \times 0.363 + 2 \times 0.08 + 3 \times 0.006 = 0.54 + \] + On peut donc estimer qu'il y aura en moyenne $0.54$ personnes qui feront sonner le portique sur les 3 personnes. + \end{enumerate} +\end{solution} + +\begin{exercise}[subtitle={Équation puissance}] + Résoudre les équations et inéquations suivantes + \begin{multicols}{2} + \begin{enumerate} + \item $10^x = 22$ + \item $4^x = 6$ + \item $0.01^x \leq 35$ + \item $10 \times 0.36^x = 19$ + \end{enumerate} + \end{multicols} +\end{exercise} + +\begin{solution} + Les solutions ci-dessous ne sont pas justifiée car l'ordinateur ne sait pas faire. Par contre, vous vous devez savoir justifier vos réponses! + \begin{enumerate} + \item $x = \log(22)$ + \item $x = \frac{\log(6)}{\log(4)}$ + \item Il faut faire attention quand on divise par un log car ce dernier peut être négatif ce qui est le cas ici. Il faut donc pense à changer le sens de l'inégalité. + + $x \geq \frac{\log(35)}{\log(0.01)}$ + + \item Il faut penser à faire la division à par $10$ avant d'utiliser le log car sinon, on ne peut pas utiliser la formule $\log(a^n) = n\times \log(a)$. + + $x = \frac{\log(1.9)}{\log(0.36)}$ + \end{enumerate} +\end{solution} + +\begin{exercise}[subtitle={Étude de fonctions}] + Soit $f(x) = - 4x^3 + 72x^2 + 1296x - 22$ une fonction définie sur $\R$. + \begin{enumerate} + \item Calculer $f'(x)$ la dérivée de $f(x)$. + \item Calculer $f'(18)$ et $f'(-6)$. + \item En déduire une forme factorisée de $f'(x)$. + \item Étudier le signe de $f'(x)$ et en déduire les variations de $f(x)$. + \item Est-ce que la fonction $f(x)$ admet un maximum ou un minimum? Si oui, calculer sa valeur. + \end{enumerate} +\end{exercise} + +\begin{solution} + \begin{enumerate} + \item Dérivée de $f(x)$: $f'(x) = - 12x^2 + 144x + 1296$ + \item + \begin{align*} + f'(18) &= - 12 \times 18^{2} + 144 \times 18 + 1296\\&= - 12 \times 324 + 2592 + 1296\\&= - 3888 + 3888\\&= 0 + \end{align*} + \begin{align*} + f'(-6) &= - 12 \times - 6^{2} + 144(- 6) + 1296\\&= - 12 \times 36 - 864 + 1296\\&= - 432 + 432\\&= 0 + \end{align*} + Donc $x = 18$ et $x=-6$ sont des racines de $f'(x) = - 12x^2 + 144x + 1296$. + \item On en déduit la forme factorisée suivante + \[ + f'(x) = -12 (x - 18)(x--6) + \] + \item Pas de correction disponible + \item À causes des branches extérieurs, la fonction $f(x)$ n'a pas de maximum ou de minimum. + \end{enumerate} +\end{solution} + + + +%\printsolutionstype{exercise} + + + +\end{document} + +%%% Local Variables: +%%% mode: latex +%%% TeX-master: "master" +%%% End: diff --git a/TST/DM/2102_DM2/TST3/20_2102_DM2.tex b/TST/DM/2102_DM2/TST3/20_2102_DM2.tex new file mode 100644 index 0000000..593217f --- /dev/null +++ b/TST/DM/2102_DM2/TST3/20_2102_DM2.tex @@ -0,0 +1,203 @@ +\documentclass[a5paper,10pt]{article} +\usepackage{myXsim} +\usepackage{tasks} + +% Title Page +\title{DM2 \hfill ZENAGUI Yanis} +\tribe{TST} +\date{\hfillÀ render pour le Mercredi 24 février} + +\xsimsetup{ + solution/print = false +} + +\begin{document} +\maketitle + +\begin{exercise}[subtitle={Loi binomiale}] + Trois personnes s'apprêtent à passer le portique de sécurité. On suppose que pour chaque personne la probabilité que le portique sonne est égale à $0.23$. + + Soit $X$ la variable aléatoire donnant le nombre de personnes faisant sonner le portique, parmi les 3 personnes de ce groupe. + \begin{enumerate} + \item Tracer l'arbre représentant le situation. + \item Justifier que $X$ suit une loi binomiale dont on précisera les paramètres. + \item Quelle est la probabilité qu'une seule personne fasse sonner le portique? + \item Calculer puis interpréter les probabilités suivantes + \[ + P(X = 0) \qquad \qquad P(X \geq 2) + \] + \item Calculer l'espérance de $X$ et interpréter le résultat. + \end{enumerate} +\end{exercise} + +\begin{solution} + \begin{enumerate} + \item + \begin{tikzpicture}[sloped] + \node {.} + child {node {$0$} + child {node {$0$} + child {node {$0$} + edge from parent + node[above] {0.77} + } + child {node {$1$} + edge from parent + node[above] {0.23} + } + edge from parent + node[above] {0.77} + } + child[missing] {} + child {node {$1$} + child {node {$0$} + edge from parent + node[above] {0.77} + } + child {node {$1$} + edge from parent + node[above] {0.23} + } + edge from parent + node[above] {0.77} + } + edge from parent + node[above] {0.77} + } + child[missing] {} + child[missing] {} + child[missing] {} + child { node {$1$} + child {node {$0$} + child {node {$0$} + edge from parent + node[above] {0.77} + } + child {node {$1$} + edge from parent + node[above] {0.23} + } + edge from parent + node[above] {0.77} + } + child[missing] {} + child {node {$1$} + child {node {$0$} + edge from parent + node[above] {0.77} + } + child {node {$1$} + edge from parent + node[above] {0.23} + } + edge from parent + node[above] {0.77} + } + edge from parent + node[above] {0.23} + } ; + \end{tikzpicture} + \item Chaque personne a 2 possibilités (1: fait sonner ou 2: ne fait pas sonner) et l'on fait passer 3 personnes ce qui correspond à une répétition identique et aléatoire. On peut donc modéliser la situation par une loi binomiale. + \[ + X \sim \mathcal{B}(3; 0.76) + \] + \item Probabilité qu'une seule personne fasse sonner le portique. On voit qu'il y a 3 branches qui correspondent à cette situation dont + \[ + P(X = 1) = 3 \times 0.23^1 \times 0.77^2 \approx 0.409 + \] + \item + \[ + P(X = 0) = 0.77^3 \approx 0.457 + \] + \[ + P(X \geq 2) = P(X = 2) + P(X = 3) = 3 \times 0.23^2 \times 0.77^1 + 0.23^3 \approx 0.134 + \] + + \item Il faut d'abord tracer le tableau résumant la loi de probabilité: + \begin{center} + \begin{tabular}{|c|*{4}{c|}} + \hline + Valeur & 0 & 1 & 2 & 3 \\ + \hline + Probabilité & $0.457$ & $0.409$ & $0.122$ &$0.012$ \\ + \hline + \end{tabular} + \end{center} + On peut alors calculer l'espérance + \[ + E[X] = 0 \times 0.457 + 1 \times 0.409 + 2 \times 0.122 + 3 \times 0.012 = 0.69 + \] + On peut donc estimer qu'il y aura en moyenne $0.69$ personnes qui feront sonner le portique sur les 3 personnes. + \end{enumerate} +\end{solution} + +\begin{exercise}[subtitle={Équation puissance}] + Résoudre les équations et inéquations suivantes + \begin{multicols}{2} + \begin{enumerate} + \item $10^x = 15$ + \item $7^x = 38$ + \item $0.66^x \leq 14$ + \item $9 \times 0.36^x = 21$ + \end{enumerate} + \end{multicols} +\end{exercise} + +\begin{solution} + Les solutions ci-dessous ne sont pas justifiée car l'ordinateur ne sait pas faire. Par contre, vous vous devez savoir justifier vos réponses! + \begin{enumerate} + \item $x = \log(15)$ + \item $x = \frac{\log(38)}{\log(7)}$ + \item Il faut faire attention quand on divise par un log car ce dernier peut être négatif ce qui est le cas ici. Il faut donc pense à changer le sens de l'inégalité. + + $x \geq \frac{\log(14)}{\log(0.66)}$ + + \item Il faut penser à faire la division à par $9$ avant d'utiliser le log car sinon, on ne peut pas utiliser la formule $\log(a^n) = n\times \log(a)$. + + $x = \frac{\log(2.33)}{\log(0.36)}$ + \end{enumerate} +\end{solution} + +\begin{exercise}[subtitle={Étude de fonctions}] + Soit $f(x) = - 5x^3 + 375x^2 - 4515x + 9$ une fonction définie sur $\R$. + \begin{enumerate} + \item Calculer $f'(x)$ la dérivée de $f(x)$. + \item Calculer $f'(43)$ et $f'(7)$. + \item En déduire une forme factorisée de $f'(x)$. + \item Étudier le signe de $f'(x)$ et en déduire les variations de $f(x)$. + \item Est-ce que la fonction $f(x)$ admet un maximum ou un minimum? Si oui, calculer sa valeur. + \end{enumerate} +\end{exercise} + +\begin{solution} + \begin{enumerate} + \item Dérivée de $f(x)$: $f'(x) = - 15x^2 + 750x - 4515$ + \item + \begin{align*} + f'(43) &= - 15 \times 43^{2} + 750 \times 43 - 4515\\&= - 15 \times 1849 + 32250 - 4515\\&= - 27735 + 27735\\&= 0 + \end{align*} + \begin{align*} + f'(7) &= - 15 \times 7^{2} + 750 \times 7 - 4515\\&= - 15 \times 49 + 5250 - 4515\\&= - 735 + 735\\&= 0 + \end{align*} + Donc $x = 43$ et $x=7$ sont des racines de $f'(x) = - 15x^2 + 750x - 4515$. + \item On en déduit la forme factorisée suivante + \[ + f'(x) = -15 (x - 43)(x-7) + \] + \item Pas de correction disponible + \item À causes des branches extérieurs, la fonction $f(x)$ n'a pas de maximum ou de minimum. + \end{enumerate} +\end{solution} + + + +%\printsolutionstype{exercise} + + + +\end{document} + +%%% Local Variables: +%%% mode: latex +%%% TeX-master: "master" +%%% End: diff --git a/TST/DM/2102_DM2/TST3/all_2102_DM2.pdf b/TST/DM/2102_DM2/TST3/all_2102_DM2.pdf new file mode 100644 index 0000000..99789d7 Binary files /dev/null and b/TST/DM/2102_DM2/TST3/all_2102_DM2.pdf differ diff --git a/TST/DM/2102_DM2/TST3/corr_01_2102_DM2.tex b/TST/DM/2102_DM2/TST3/corr_01_2102_DM2.tex new file mode 100644 index 0000000..58a1904 --- /dev/null +++ b/TST/DM/2102_DM2/TST3/corr_01_2102_DM2.tex @@ -0,0 +1,203 @@ +\documentclass[a5paper,10pt]{article} +\usepackage{myXsim} +\usepackage{tasks} + +% Title Page +\title{DM2 \hfill ASAIDI Sophian} +\tribe{TST} +\date{\hfillÀ render pour le Mercredi 24 février} + +\xsimsetup{ + solution/print = true +} + +\begin{document} +\maketitle + +\begin{exercise}[subtitle={Loi binomiale}] + Trois personnes s'apprêtent à passer le portique de sécurité. On suppose que pour chaque personne la probabilité que le portique sonne est égale à $0.19$. + + Soit $X$ la variable aléatoire donnant le nombre de personnes faisant sonner le portique, parmi les 3 personnes de ce groupe. + \begin{enumerate} + \item Tracer l'arbre représentant le situation. + \item Justifier que $X$ suit une loi binomiale dont on précisera les paramètres. + \item Quelle est la probabilité qu'une seule personne fasse sonner le portique? + \item Calculer puis interpréter les probabilités suivantes + \[ + P(X = 0) \qquad \qquad P(X \geq 2) + \] + \item Calculer l'espérance de $X$ et interpréter le résultat. + \end{enumerate} +\end{exercise} + +\begin{solution} + \begin{enumerate} + \item + \begin{tikzpicture}[sloped] + \node {.} + child {node {$0$} + child {node {$0$} + child {node {$0$} + edge from parent + node[above] {0.81} + } + child {node {$1$} + edge from parent + node[above] {0.19} + } + edge from parent + node[above] {0.81} + } + child[missing] {} + child {node {$1$} + child {node {$0$} + edge from parent + node[above] {0.81} + } + child {node {$1$} + edge from parent + node[above] {0.19} + } + edge from parent + node[above] {0.81} + } + edge from parent + node[above] {0.81} + } + child[missing] {} + child[missing] {} + child[missing] {} + child { node {$1$} + child {node {$0$} + child {node {$0$} + edge from parent + node[above] {0.81} + } + child {node {$1$} + edge from parent + node[above] {0.19} + } + edge from parent + node[above] {0.81} + } + child[missing] {} + child {node {$1$} + child {node {$0$} + edge from parent + node[above] {0.81} + } + child {node {$1$} + edge from parent + node[above] {0.19} + } + edge from parent + node[above] {0.81} + } + edge from parent + node[above] {0.19} + } ; + \end{tikzpicture} + \item Chaque personne a 2 possibilités (1: fait sonner ou 2: ne fait pas sonner) et l'on fait passer 3 personnes ce qui correspond à une répétition identique et aléatoire. On peut donc modéliser la situation par une loi binomiale. + \[ + X \sim \mathcal{B}(3; 0.76) + \] + \item Probabilité qu'une seule personne fasse sonner le portique. On voit qu'il y a 3 branches qui correspondent à cette situation dont + \[ + P(X = 1) = 3 \times 0.19^1 \times 0.81^2 \approx 0.374 + \] + \item + \[ + P(X = 0) = 0.81^3 \approx 0.531 + \] + \[ + P(X \geq 2) = P(X = 2) + P(X = 3) = 3 \times 0.19^2 \times 0.81^1 + 0.19^3 \approx 0.095 + \] + + \item Il faut d'abord tracer le tableau résumant la loi de probabilité: + \begin{center} + \begin{tabular}{|c|*{4}{c|}} + \hline + Valeur & 0 & 1 & 2 & 3 \\ + \hline + Probabilité & $0.531$ & $0.374$ & $0.088$ &$0.007$ \\ + \hline + \end{tabular} + \end{center} + On peut alors calculer l'espérance + \[ + E[X] = 0 \times 0.531 + 1 \times 0.374 + 2 \times 0.088 + 3 \times 0.007 = 0.57 + \] + On peut donc estimer qu'il y aura en moyenne $0.57$ personnes qui feront sonner le portique sur les 3 personnes. + \end{enumerate} +\end{solution} + +\begin{exercise}[subtitle={Équation puissance}] + Résoudre les équations et inéquations suivantes + \begin{multicols}{2} + \begin{enumerate} + \item $10^x = 7$ + \item $3^x = 35$ + \item $0.8^x \leq 2$ + \item $4 \times 0.06^x = 49$ + \end{enumerate} + \end{multicols} +\end{exercise} + +\begin{solution} + Les solutions ci-dessous ne sont pas justifiée car l'ordinateur ne sait pas faire. Par contre, vous vous devez savoir justifier vos réponses! + \begin{enumerate} + \item $x = \log(7)$ + \item $x = \frac{\log(35)}{\log(3)}$ + \item Il faut faire attention quand on divise par un log car ce dernier peut être négatif ce qui est le cas ici. Il faut donc pense à changer le sens de l'inégalité. + + $x \geq \frac{\log(2)}{\log(0.8)}$ + + \item Il faut penser à faire la division à par $4$ avant d'utiliser le log car sinon, on ne peut pas utiliser la formule $\log(a^n) = n\times \log(a)$. + + $x = \frac{\log(12.25)}{\log(0.06)}$ + \end{enumerate} +\end{solution} + +\begin{exercise}[subtitle={Étude de fonctions}] + Soit $f(x) = 5x^3 - 202.5x^2 - 2970x + 24$ une fonction définie sur $\R$. + \begin{enumerate} + \item Calculer $f'(x)$ la dérivée de $f(x)$. + \item Calculer $f'(33)$ et $f'(-6)$. + \item En déduire une forme factorisée de $f'(x)$. + \item Étudier le signe de $f'(x)$ et en déduire les variations de $f(x)$. + \item Est-ce que la fonction $f(x)$ admet un maximum ou un minimum? Si oui, calculer sa valeur. + \end{enumerate} +\end{exercise} + +\begin{solution} + \begin{enumerate} + \item Dérivée de $f(x)$: $f'(x) = 15x^2 - 405x - 2970$ + \item + \begin{align*} + f'(33) &= 15 \times 33^{2} - 405 \times 33 - 2970\\&= 15 \times 1089 - 13365 - 2970\\&= 16335 - 16335\\&= 0 + \end{align*} + \begin{align*} + f'(-6) &= 15 \times - 6^{2} - 405(- 6) - 2970\\&= 15 \times 36 + 2430 - 2970\\&= 540 - 540\\&= 0 + \end{align*} + Donc $x = 33$ et $x=-6$ sont des racines de $f'(x) = 15x^2 - 405x - 2970$. + \item On en déduit la forme factorisée suivante + \[ + f'(x) = 15 (x - 33)(x--6) + \] + \item Pas de correction disponible + \item À causes des branches extérieurs, la fonction $f(x)$ n'a pas de maximum ou de minimum. + \end{enumerate} +\end{solution} + + + +%\printsolutionstype{exercise} + + + +\end{document} + +%%% Local Variables: +%%% mode: latex +%%% TeX-master: "master" +%%% End: diff --git a/TST/DM/2102_DM2/TST3/corr_02_2102_DM2.tex b/TST/DM/2102_DM2/TST3/corr_02_2102_DM2.tex new file mode 100644 index 0000000..eb0b508 --- /dev/null +++ b/TST/DM/2102_DM2/TST3/corr_02_2102_DM2.tex @@ -0,0 +1,203 @@ +\documentclass[a5paper,10pt]{article} +\usepackage{myXsim} +\usepackage{tasks} + +% Title Page +\title{DM2 \hfill BELARBI Samira} +\tribe{TST} +\date{\hfillÀ render pour le Mercredi 24 février} + +\xsimsetup{ + solution/print = true +} + +\begin{document} +\maketitle + +\begin{exercise}[subtitle={Loi binomiale}] + Trois personnes s'apprêtent à passer le portique de sécurité. On suppose que pour chaque personne la probabilité que le portique sonne est égale à $0.58$. + + Soit $X$ la variable aléatoire donnant le nombre de personnes faisant sonner le portique, parmi les 3 personnes de ce groupe. + \begin{enumerate} + \item Tracer l'arbre représentant le situation. + \item Justifier que $X$ suit une loi binomiale dont on précisera les paramètres. + \item Quelle est la probabilité qu'une seule personne fasse sonner le portique? + \item Calculer puis interpréter les probabilités suivantes + \[ + P(X = 0) \qquad \qquad P(X \geq 2) + \] + \item Calculer l'espérance de $X$ et interpréter le résultat. + \end{enumerate} +\end{exercise} + +\begin{solution} + \begin{enumerate} + \item + \begin{tikzpicture}[sloped] + \node {.} + child {node {$0$} + child {node {$0$} + child {node {$0$} + edge from parent + node[above] {0.42} + } + child {node {$1$} + edge from parent + node[above] {0.58} + } + edge from parent + node[above] {0.42} + } + child[missing] {} + child {node {$1$} + child {node {$0$} + edge from parent + node[above] {0.42} + } + child {node {$1$} + edge from parent + node[above] {0.58} + } + edge from parent + node[above] {0.42} + } + edge from parent + node[above] {0.42} + } + child[missing] {} + child[missing] {} + child[missing] {} + child { node {$1$} + child {node {$0$} + child {node {$0$} + edge from parent + node[above] {0.42} + } + child {node {$1$} + edge from parent + node[above] {0.58} + } + edge from parent + node[above] {0.42} + } + child[missing] {} + child {node {$1$} + child {node {$0$} + edge from parent + node[above] {0.42} + } + child {node {$1$} + edge from parent + node[above] {0.58} + } + edge from parent + node[above] {0.42} + } + edge from parent + node[above] {0.58} + } ; + \end{tikzpicture} + \item Chaque personne a 2 possibilités (1: fait sonner ou 2: ne fait pas sonner) et l'on fait passer 3 personnes ce qui correspond à une répétition identique et aléatoire. On peut donc modéliser la situation par une loi binomiale. + \[ + X \sim \mathcal{B}(3; 0.76) + \] + \item Probabilité qu'une seule personne fasse sonner le portique. On voit qu'il y a 3 branches qui correspondent à cette situation dont + \[ + P(X = 1) = 3 \times 0.58^1 \times 0.42^2 \approx 0.307 + \] + \item + \[ + P(X = 0) = 0.42^3 \approx 0.074 + \] + \[ + P(X \geq 2) = P(X = 2) + P(X = 3) = 3 \times 0.58^2 \times 0.42^1 + 0.58^3 \approx 0.619 + \] + + \item Il faut d'abord tracer le tableau résumant la loi de probabilité: + \begin{center} + \begin{tabular}{|c|*{4}{c|}} + \hline + Valeur & 0 & 1 & 2 & 3 \\ + \hline + Probabilité & $0.074$ & $0.307$ & $0.424$ &$0.195$ \\ + \hline + \end{tabular} + \end{center} + On peut alors calculer l'espérance + \[ + E[X] = 0 \times 0.074 + 1 \times 0.307 + 2 \times 0.424 + 3 \times 0.195 = 1.74 + \] + On peut donc estimer qu'il y aura en moyenne $1.74$ personnes qui feront sonner le portique sur les 3 personnes. + \end{enumerate} +\end{solution} + +\begin{exercise}[subtitle={Équation puissance}] + Résoudre les équations et inéquations suivantes + \begin{multicols}{2} + \begin{enumerate} + \item $10^x = 6$ + \item $10^x = 31$ + \item $0.32^x \leq 15$ + \item $5 \times 0.06^x = 9$ + \end{enumerate} + \end{multicols} +\end{exercise} + +\begin{solution} + Les solutions ci-dessous ne sont pas justifiée car l'ordinateur ne sait pas faire. Par contre, vous vous devez savoir justifier vos réponses! + \begin{enumerate} + \item $x = \log(6)$ + \item $x = \frac{\log(31)}{\log(10)}$ + \item Il faut faire attention quand on divise par un log car ce dernier peut être négatif ce qui est le cas ici. Il faut donc pense à changer le sens de l'inégalité. + + $x \geq \frac{\log(15)}{\log(0.32)}$ + + \item Il faut penser à faire la division à par $5$ avant d'utiliser le log car sinon, on ne peut pas utiliser la formule $\log(a^n) = n\times \log(a)$. + + $x = \frac{\log(1.8)}{\log(0.06)}$ + \end{enumerate} +\end{solution} + +\begin{exercise}[subtitle={Étude de fonctions}] + Soit $f(x) = - 4x^3 + 126x^2 + 2784x - 17$ une fonction définie sur $\R$. + \begin{enumerate} + \item Calculer $f'(x)$ la dérivée de $f(x)$. + \item Calculer $f'(29)$ et $f'(-8)$. + \item En déduire une forme factorisée de $f'(x)$. + \item Étudier le signe de $f'(x)$ et en déduire les variations de $f(x)$. + \item Est-ce que la fonction $f(x)$ admet un maximum ou un minimum? Si oui, calculer sa valeur. + \end{enumerate} +\end{exercise} + +\begin{solution} + \begin{enumerate} + \item Dérivée de $f(x)$: $f'(x) = - 12x^2 + 252x + 2784$ + \item + \begin{align*} + f'(29) &= - 12 \times 29^{2} + 252 \times 29 + 2784\\&= - 12 \times 841 + 7308 + 2784\\&= - 10092 + 10092\\&= 0 + \end{align*} + \begin{align*} + f'(-8) &= - 12 \times - 8^{2} + 252(- 8) + 2784\\&= - 12 \times 64 - 2016 + 2784\\&= - 768 + 768\\&= 0 + \end{align*} + Donc $x = 29$ et $x=-8$ sont des racines de $f'(x) = - 12x^2 + 252x + 2784$. + \item On en déduit la forme factorisée suivante + \[ + f'(x) = -12 (x - 29)(x--8) + \] + \item Pas de correction disponible + \item À causes des branches extérieurs, la fonction $f(x)$ n'a pas de maximum ou de minimum. + \end{enumerate} +\end{solution} + + + +%\printsolutionstype{exercise} + + + +\end{document} + +%%% Local Variables: +%%% mode: latex +%%% TeX-master: "master" +%%% End: diff --git a/TST/DM/2102_DM2/TST3/corr_03_2102_DM2.tex b/TST/DM/2102_DM2/TST3/corr_03_2102_DM2.tex new file mode 100644 index 0000000..fe1abff --- /dev/null +++ b/TST/DM/2102_DM2/TST3/corr_03_2102_DM2.tex @@ -0,0 +1,203 @@ +\documentclass[a5paper,10pt]{article} +\usepackage{myXsim} +\usepackage{tasks} + +% Title Page +\title{DM2 \hfill BERTAN Ufuk} +\tribe{TST} +\date{\hfillÀ render pour le Mercredi 24 février} + +\xsimsetup{ + solution/print = true +} + +\begin{document} +\maketitle + +\begin{exercise}[subtitle={Loi binomiale}] + Trois personnes s'apprêtent à passer le portique de sécurité. On suppose que pour chaque personne la probabilité que le portique sonne est égale à $0.04$. + + Soit $X$ la variable aléatoire donnant le nombre de personnes faisant sonner le portique, parmi les 3 personnes de ce groupe. + \begin{enumerate} + \item Tracer l'arbre représentant le situation. + \item Justifier que $X$ suit une loi binomiale dont on précisera les paramètres. + \item Quelle est la probabilité qu'une seule personne fasse sonner le portique? + \item Calculer puis interpréter les probabilités suivantes + \[ + P(X = 0) \qquad \qquad P(X \geq 2) + \] + \item Calculer l'espérance de $X$ et interpréter le résultat. + \end{enumerate} +\end{exercise} + +\begin{solution} + \begin{enumerate} + \item + \begin{tikzpicture}[sloped] + \node {.} + child {node {$0$} + child {node {$0$} + child {node {$0$} + edge from parent + node[above] {0.96} + } + child {node {$1$} + edge from parent + node[above] {0.04} + } + edge from parent + node[above] {0.96} + } + child[missing] {} + child {node {$1$} + child {node {$0$} + edge from parent + node[above] {0.96} + } + child {node {$1$} + edge from parent + node[above] {0.04} + } + edge from parent + node[above] {0.96} + } + edge from parent + node[above] {0.96} + } + child[missing] {} + child[missing] {} + child[missing] {} + child { node {$1$} + child {node {$0$} + child {node {$0$} + edge from parent + node[above] {0.96} + } + child {node {$1$} + edge from parent + node[above] {0.04} + } + edge from parent + node[above] {0.96} + } + child[missing] {} + child {node {$1$} + child {node {$0$} + edge from parent + node[above] {0.96} + } + child {node {$1$} + edge from parent + node[above] {0.04} + } + edge from parent + node[above] {0.96} + } + edge from parent + node[above] {0.04} + } ; + \end{tikzpicture} + \item Chaque personne a 2 possibilités (1: fait sonner ou 2: ne fait pas sonner) et l'on fait passer 3 personnes ce qui correspond à une répétition identique et aléatoire. On peut donc modéliser la situation par une loi binomiale. + \[ + X \sim \mathcal{B}(3; 0.76) + \] + \item Probabilité qu'une seule personne fasse sonner le portique. On voit qu'il y a 3 branches qui correspondent à cette situation dont + \[ + P(X = 1) = 3 \times 0.04^1 \times 0.96^2 \approx 0.111 + \] + \item + \[ + P(X = 0) = 0.96^3 \approx 0.885 + \] + \[ + P(X \geq 2) = P(X = 2) + P(X = 3) = 3 \times 0.04^2 \times 0.96^1 + 0.04^3 \approx 0.005 + \] + + \item Il faut d'abord tracer le tableau résumant la loi de probabilité: + \begin{center} + \begin{tabular}{|c|*{4}{c|}} + \hline + Valeur & 0 & 1 & 2 & 3 \\ + \hline + Probabilité & $0.885$ & $0.111$ & $0.005$ &$0.0$ \\ + \hline + \end{tabular} + \end{center} + On peut alors calculer l'espérance + \[ + E[X] = 0 \times 0.885 + 1 \times 0.111 + 2 \times 0.005 + 3 \times 0.0 = 0.12 + \] + On peut donc estimer qu'il y aura en moyenne $0.12$ personnes qui feront sonner le portique sur les 3 personnes. + \end{enumerate} +\end{solution} + +\begin{exercise}[subtitle={Équation puissance}] + Résoudre les équations et inéquations suivantes + \begin{multicols}{2} + \begin{enumerate} + \item $10^x = 14$ + \item $11^x = 35$ + \item $0.05^x \leq 24$ + \item $4 \times 0.92^x = 47$ + \end{enumerate} + \end{multicols} +\end{exercise} + +\begin{solution} + Les solutions ci-dessous ne sont pas justifiée car l'ordinateur ne sait pas faire. Par contre, vous vous devez savoir justifier vos réponses! + \begin{enumerate} + \item $x = \log(14)$ + \item $x = \frac{\log(35)}{\log(11)}$ + \item Il faut faire attention quand on divise par un log car ce dernier peut être négatif ce qui est le cas ici. Il faut donc pense à changer le sens de l'inégalité. + + $x \geq \frac{\log(24)}{\log(0.05)}$ + + \item Il faut penser à faire la division à par $4$ avant d'utiliser le log car sinon, on ne peut pas utiliser la formule $\log(a^n) = n\times \log(a)$. + + $x = \frac{\log(11.75)}{\log(0.92)}$ + \end{enumerate} +\end{solution} + +\begin{exercise}[subtitle={Étude de fonctions}] + Soit $f(x) = 4x^3 - 306x^2 + 4128x + 39$ une fonction définie sur $\R$. + \begin{enumerate} + \item Calculer $f'(x)$ la dérivée de $f(x)$. + \item Calculer $f'(43)$ et $f'(8)$. + \item En déduire une forme factorisée de $f'(x)$. + \item Étudier le signe de $f'(x)$ et en déduire les variations de $f(x)$. + \item Est-ce que la fonction $f(x)$ admet un maximum ou un minimum? Si oui, calculer sa valeur. + \end{enumerate} +\end{exercise} + +\begin{solution} + \begin{enumerate} + \item Dérivée de $f(x)$: $f'(x) = 12x^2 - 612x + 4128$ + \item + \begin{align*} + f'(43) &= 12 \times 43^{2} - 612 \times 43 + 4128\\&= 12 \times 1849 - 26316 + 4128\\&= 22188 - 22188\\&= 0 + \end{align*} + \begin{align*} + f'(8) &= 12 \times 8^{2} - 612 \times 8 + 4128\\&= 12 \times 64 - 4896 + 4128\\&= 768 - 768\\&= 0 + \end{align*} + Donc $x = 43$ et $x=8$ sont des racines de $f'(x) = 12x^2 - 612x + 4128$. + \item On en déduit la forme factorisée suivante + \[ + f'(x) = 12 (x - 43)(x-8) + \] + \item Pas de correction disponible + \item À causes des branches extérieurs, la fonction $f(x)$ n'a pas de maximum ou de minimum. + \end{enumerate} +\end{solution} + + + +%\printsolutionstype{exercise} + + + +\end{document} + +%%% Local Variables: +%%% mode: latex +%%% TeX-master: "master" +%%% End: diff --git a/TST/DM/2102_DM2/TST3/corr_04_2102_DM2.tex b/TST/DM/2102_DM2/TST3/corr_04_2102_DM2.tex new file mode 100644 index 0000000..1a758b6 --- /dev/null +++ b/TST/DM/2102_DM2/TST3/corr_04_2102_DM2.tex @@ -0,0 +1,203 @@ +\documentclass[a5paper,10pt]{article} +\usepackage{myXsim} +\usepackage{tasks} + +% Title Page +\title{DM2 \hfill BOUALIA Bilel} +\tribe{TST} +\date{\hfillÀ render pour le Mercredi 24 février} + +\xsimsetup{ + solution/print = true +} + +\begin{document} +\maketitle + +\begin{exercise}[subtitle={Loi binomiale}] + Trois personnes s'apprêtent à passer le portique de sécurité. On suppose que pour chaque personne la probabilité que le portique sonne est égale à $0.68$. + + Soit $X$ la variable aléatoire donnant le nombre de personnes faisant sonner le portique, parmi les 3 personnes de ce groupe. + \begin{enumerate} + \item Tracer l'arbre représentant le situation. + \item Justifier que $X$ suit une loi binomiale dont on précisera les paramètres. + \item Quelle est la probabilité qu'une seule personne fasse sonner le portique? + \item Calculer puis interpréter les probabilités suivantes + \[ + P(X = 0) \qquad \qquad P(X \geq 2) + \] + \item Calculer l'espérance de $X$ et interpréter le résultat. + \end{enumerate} +\end{exercise} + +\begin{solution} + \begin{enumerate} + \item + \begin{tikzpicture}[sloped] + \node {.} + child {node {$0$} + child {node {$0$} + child {node {$0$} + edge from parent + node[above] {0.32} + } + child {node {$1$} + edge from parent + node[above] {0.68} + } + edge from parent + node[above] {0.32} + } + child[missing] {} + child {node {$1$} + child {node {$0$} + edge from parent + node[above] {0.32} + } + child {node {$1$} + edge from parent + node[above] {0.68} + } + edge from parent + node[above] {0.32} + } + edge from parent + node[above] {0.32} + } + child[missing] {} + child[missing] {} + child[missing] {} + child { node {$1$} + child {node {$0$} + child {node {$0$} + edge from parent + node[above] {0.32} + } + child {node {$1$} + edge from parent + node[above] {0.68} + } + edge from parent + node[above] {0.32} + } + child[missing] {} + child {node {$1$} + child {node {$0$} + edge from parent + node[above] {0.32} + } + child {node {$1$} + edge from parent + node[above] {0.68} + } + edge from parent + node[above] {0.32} + } + edge from parent + node[above] {0.68} + } ; + \end{tikzpicture} + \item Chaque personne a 2 possibilités (1: fait sonner ou 2: ne fait pas sonner) et l'on fait passer 3 personnes ce qui correspond à une répétition identique et aléatoire. On peut donc modéliser la situation par une loi binomiale. + \[ + X \sim \mathcal{B}(3; 0.76) + \] + \item Probabilité qu'une seule personne fasse sonner le portique. On voit qu'il y a 3 branches qui correspondent à cette situation dont + \[ + P(X = 1) = 3 \times 0.68^1 \times 0.32^2 \approx 0.209 + \] + \item + \[ + P(X = 0) = 0.32^3 \approx 0.033 + \] + \[ + P(X \geq 2) = P(X = 2) + P(X = 3) = 3 \times 0.68^2 \times 0.32^1 + 0.68^3 \approx 0.758 + \] + + \item Il faut d'abord tracer le tableau résumant la loi de probabilité: + \begin{center} + \begin{tabular}{|c|*{4}{c|}} + \hline + Valeur & 0 & 1 & 2 & 3 \\ + \hline + Probabilité & $0.033$ & $0.209$ & $0.444$ &$0.314$ \\ + \hline + \end{tabular} + \end{center} + On peut alors calculer l'espérance + \[ + E[X] = 0 \times 0.033 + 1 \times 0.209 + 2 \times 0.444 + 3 \times 0.314 = 2.04 + \] + On peut donc estimer qu'il y aura en moyenne $2.04$ personnes qui feront sonner le portique sur les 3 personnes. + \end{enumerate} +\end{solution} + +\begin{exercise}[subtitle={Équation puissance}] + Résoudre les équations et inéquations suivantes + \begin{multicols}{2} + \begin{enumerate} + \item $10^x = 45$ + \item $10^x = 5$ + \item $0.69^x \leq 42$ + \item $4 \times 0.04^x = 21$ + \end{enumerate} + \end{multicols} +\end{exercise} + +\begin{solution} + Les solutions ci-dessous ne sont pas justifiée car l'ordinateur ne sait pas faire. Par contre, vous vous devez savoir justifier vos réponses! + \begin{enumerate} + \item $x = \log(45)$ + \item $x = \frac{\log(5)}{\log(10)}$ + \item Il faut faire attention quand on divise par un log car ce dernier peut être négatif ce qui est le cas ici. Il faut donc pense à changer le sens de l'inégalité. + + $x \geq \frac{\log(42)}{\log(0.69)}$ + + \item Il faut penser à faire la division à par $4$ avant d'utiliser le log car sinon, on ne peut pas utiliser la formule $\log(a^n) = n\times \log(a)$. + + $x = \frac{\log(5.25)}{\log(0.04)}$ + \end{enumerate} +\end{solution} + +\begin{exercise}[subtitle={Étude de fonctions}] + Soit $f(x) = 10x^3 - 645x^2 - 7200x - 16$ une fonction définie sur $\R$. + \begin{enumerate} + \item Calculer $f'(x)$ la dérivée de $f(x)$. + \item Calculer $f'(48)$ et $f'(-5)$. + \item En déduire une forme factorisée de $f'(x)$. + \item Étudier le signe de $f'(x)$ et en déduire les variations de $f(x)$. + \item Est-ce que la fonction $f(x)$ admet un maximum ou un minimum? Si oui, calculer sa valeur. + \end{enumerate} +\end{exercise} + +\begin{solution} + \begin{enumerate} + \item Dérivée de $f(x)$: $f'(x) = 30x^2 - 1290x - 7200$ + \item + \begin{align*} + f'(48) &= 30 \times 48^{2} - 1290 \times 48 - 7200\\&= 30 \times 2304 - 61920 - 7200\\&= 69120 - 69120\\&= 0 + \end{align*} + \begin{align*} + f'(-5) &= 30 \times - 5^{2} - 1290(- 5) - 7200\\&= 30 \times 25 + 6450 - 7200\\&= 750 - 750\\&= 0 + \end{align*} + Donc $x = 48$ et $x=-5$ sont des racines de $f'(x) = 30x^2 - 1290x - 7200$. + \item On en déduit la forme factorisée suivante + \[ + f'(x) = 30 (x - 48)(x--5) + \] + \item Pas de correction disponible + \item À causes des branches extérieurs, la fonction $f(x)$ n'a pas de maximum ou de minimum. + \end{enumerate} +\end{solution} + + + +%\printsolutionstype{exercise} + + + +\end{document} + +%%% Local Variables: +%%% mode: latex +%%% TeX-master: "master" +%%% End: diff --git a/TST/DM/2102_DM2/TST3/corr_05_2102_DM2.tex b/TST/DM/2102_DM2/TST3/corr_05_2102_DM2.tex new file mode 100644 index 0000000..96545d2 --- /dev/null +++ b/TST/DM/2102_DM2/TST3/corr_05_2102_DM2.tex @@ -0,0 +1,203 @@ +\documentclass[a5paper,10pt]{article} +\usepackage{myXsim} +\usepackage{tasks} + +% Title Page +\title{DM2 \hfill BOUCHOUX Kevin} +\tribe{TST} +\date{\hfillÀ render pour le Mercredi 24 février} + +\xsimsetup{ + solution/print = true +} + +\begin{document} +\maketitle + +\begin{exercise}[subtitle={Loi binomiale}] + Trois personnes s'apprêtent à passer le portique de sécurité. On suppose que pour chaque personne la probabilité que le portique sonne est égale à $0.7$. + + Soit $X$ la variable aléatoire donnant le nombre de personnes faisant sonner le portique, parmi les 3 personnes de ce groupe. + \begin{enumerate} + \item Tracer l'arbre représentant le situation. + \item Justifier que $X$ suit une loi binomiale dont on précisera les paramètres. + \item Quelle est la probabilité qu'une seule personne fasse sonner le portique? + \item Calculer puis interpréter les probabilités suivantes + \[ + P(X = 0) \qquad \qquad P(X \geq 2) + \] + \item Calculer l'espérance de $X$ et interpréter le résultat. + \end{enumerate} +\end{exercise} + +\begin{solution} + \begin{enumerate} + \item + \begin{tikzpicture}[sloped] + \node {.} + child {node {$0$} + child {node {$0$} + child {node {$0$} + edge from parent + node[above] {0.3} + } + child {node {$1$} + edge from parent + node[above] {0.7} + } + edge from parent + node[above] {0.3} + } + child[missing] {} + child {node {$1$} + child {node {$0$} + edge from parent + node[above] {0.3} + } + child {node {$1$} + edge from parent + node[above] {0.7} + } + edge from parent + node[above] {0.3} + } + edge from parent + node[above] {0.3} + } + child[missing] {} + child[missing] {} + child[missing] {} + child { node {$1$} + child {node {$0$} + child {node {$0$} + edge from parent + node[above] {0.3} + } + child {node {$1$} + edge from parent + node[above] {0.7} + } + edge from parent + node[above] {0.3} + } + child[missing] {} + child {node {$1$} + child {node {$0$} + edge from parent + node[above] {0.3} + } + child {node {$1$} + edge from parent + node[above] {0.7} + } + edge from parent + node[above] {0.3} + } + edge from parent + node[above] {0.7} + } ; + \end{tikzpicture} + \item Chaque personne a 2 possibilités (1: fait sonner ou 2: ne fait pas sonner) et l'on fait passer 3 personnes ce qui correspond à une répétition identique et aléatoire. On peut donc modéliser la situation par une loi binomiale. + \[ + X \sim \mathcal{B}(3; 0.76) + \] + \item Probabilité qu'une seule personne fasse sonner le portique. On voit qu'il y a 3 branches qui correspondent à cette situation dont + \[ + P(X = 1) = 3 \times 0.7^1 \times 0.3^2 \approx 0.189 + \] + \item + \[ + P(X = 0) = 0.3^3 \approx 0.027 + \] + \[ + P(X \geq 2) = P(X = 2) + P(X = 3) = 3 \times 0.7^2 \times 0.3^1 + 0.7^3 \approx 0.784 + \] + + \item Il faut d'abord tracer le tableau résumant la loi de probabilité: + \begin{center} + \begin{tabular}{|c|*{4}{c|}} + \hline + Valeur & 0 & 1 & 2 & 3 \\ + \hline + Probabilité & $0.027$ & $0.189$ & $0.441$ &$0.343$ \\ + \hline + \end{tabular} + \end{center} + On peut alors calculer l'espérance + \[ + E[X] = 0 \times 0.027 + 1 \times 0.189 + 2 \times 0.441 + 3 \times 0.343 = 2.1 + \] + On peut donc estimer qu'il y aura en moyenne $2.1$ personnes qui feront sonner le portique sur les 3 personnes. + \end{enumerate} +\end{solution} + +\begin{exercise}[subtitle={Équation puissance}] + Résoudre les équations et inéquations suivantes + \begin{multicols}{2} + \begin{enumerate} + \item $10^x = 4$ + \item $7^x = 14$ + \item $0.44^x \leq 29$ + \item $6 \times 0.27^x = 10$ + \end{enumerate} + \end{multicols} +\end{exercise} + +\begin{solution} + Les solutions ci-dessous ne sont pas justifiée car l'ordinateur ne sait pas faire. Par contre, vous vous devez savoir justifier vos réponses! + \begin{enumerate} + \item $x = \log(4)$ + \item $x = \frac{\log(14)}{\log(7)}$ + \item Il faut faire attention quand on divise par un log car ce dernier peut être négatif ce qui est le cas ici. Il faut donc pense à changer le sens de l'inégalité. + + $x \geq \frac{\log(29)}{\log(0.44)}$ + + \item Il faut penser à faire la division à par $6$ avant d'utiliser le log car sinon, on ne peut pas utiliser la formule $\log(a^n) = n\times \log(a)$. + + $x = \frac{\log(1.67)}{\log(0.27)}$ + \end{enumerate} +\end{solution} + +\begin{exercise}[subtitle={Étude de fonctions}] + Soit $f(x) = 10x^3 - 840x^2 + 18450x - 1$ une fonction définie sur $\R$. + \begin{enumerate} + \item Calculer $f'(x)$ la dérivée de $f(x)$. + \item Calculer $f'(41)$ et $f'(15)$. + \item En déduire une forme factorisée de $f'(x)$. + \item Étudier le signe de $f'(x)$ et en déduire les variations de $f(x)$. + \item Est-ce que la fonction $f(x)$ admet un maximum ou un minimum? Si oui, calculer sa valeur. + \end{enumerate} +\end{exercise} + +\begin{solution} + \begin{enumerate} + \item Dérivée de $f(x)$: $f'(x) = 30x^2 - 1680x + 18450$ + \item + \begin{align*} + f'(41) &= 30 \times 41^{2} - 1680 \times 41 + 18450\\&= 30 \times 1681 - 68880 + 18450\\&= 50430 - 50430\\&= 0 + \end{align*} + \begin{align*} + f'(15) &= 30 \times 15^{2} - 1680 \times 15 + 18450\\&= 30 \times 225 - 25200 + 18450\\&= 6750 - 6750\\&= 0 + \end{align*} + Donc $x = 41$ et $x=15$ sont des racines de $f'(x) = 30x^2 - 1680x + 18450$. + \item On en déduit la forme factorisée suivante + \[ + f'(x) = 30 (x - 41)(x-15) + \] + \item Pas de correction disponible + \item À causes des branches extérieurs, la fonction $f(x)$ n'a pas de maximum ou de minimum. + \end{enumerate} +\end{solution} + + + +%\printsolutionstype{exercise} + + + +\end{document} + +%%% Local Variables: +%%% mode: latex +%%% TeX-master: "master" +%%% End: diff --git a/TST/DM/2102_DM2/TST3/corr_06_2102_DM2.tex b/TST/DM/2102_DM2/TST3/corr_06_2102_DM2.tex new file mode 100644 index 0000000..4bff570 --- /dev/null +++ b/TST/DM/2102_DM2/TST3/corr_06_2102_DM2.tex @@ -0,0 +1,203 @@ +\documentclass[a5paper,10pt]{article} +\usepackage{myXsim} +\usepackage{tasks} + +% Title Page +\title{DM2 \hfill BUDIN Nathan} +\tribe{TST} +\date{\hfillÀ render pour le Mercredi 24 février} + +\xsimsetup{ + solution/print = true +} + +\begin{document} +\maketitle + +\begin{exercise}[subtitle={Loi binomiale}] + Trois personnes s'apprêtent à passer le portique de sécurité. On suppose que pour chaque personne la probabilité que le portique sonne est égale à $0.37$. + + Soit $X$ la variable aléatoire donnant le nombre de personnes faisant sonner le portique, parmi les 3 personnes de ce groupe. + \begin{enumerate} + \item Tracer l'arbre représentant le situation. + \item Justifier que $X$ suit une loi binomiale dont on précisera les paramètres. + \item Quelle est la probabilité qu'une seule personne fasse sonner le portique? + \item Calculer puis interpréter les probabilités suivantes + \[ + P(X = 0) \qquad \qquad P(X \geq 2) + \] + \item Calculer l'espérance de $X$ et interpréter le résultat. + \end{enumerate} +\end{exercise} + +\begin{solution} + \begin{enumerate} + \item + \begin{tikzpicture}[sloped] + \node {.} + child {node {$0$} + child {node {$0$} + child {node {$0$} + edge from parent + node[above] {0.63} + } + child {node {$1$} + edge from parent + node[above] {0.37} + } + edge from parent + node[above] {0.63} + } + child[missing] {} + child {node {$1$} + child {node {$0$} + edge from parent + node[above] {0.63} + } + child {node {$1$} + edge from parent + node[above] {0.37} + } + edge from parent + node[above] {0.63} + } + edge from parent + node[above] {0.63} + } + child[missing] {} + child[missing] {} + child[missing] {} + child { node {$1$} + child {node {$0$} + child {node {$0$} + edge from parent + node[above] {0.63} + } + child {node {$1$} + edge from parent + node[above] {0.37} + } + edge from parent + node[above] {0.63} + } + child[missing] {} + child {node {$1$} + child {node {$0$} + edge from parent + node[above] {0.63} + } + child {node {$1$} + edge from parent + node[above] {0.37} + } + edge from parent + node[above] {0.63} + } + edge from parent + node[above] {0.37} + } ; + \end{tikzpicture} + \item Chaque personne a 2 possibilités (1: fait sonner ou 2: ne fait pas sonner) et l'on fait passer 3 personnes ce qui correspond à une répétition identique et aléatoire. On peut donc modéliser la situation par une loi binomiale. + \[ + X \sim \mathcal{B}(3; 0.76) + \] + \item Probabilité qu'une seule personne fasse sonner le portique. On voit qu'il y a 3 branches qui correspondent à cette situation dont + \[ + P(X = 1) = 3 \times 0.37^1 \times 0.63^2 \approx 0.441 + \] + \item + \[ + P(X = 0) = 0.63^3 \approx 0.25 + \] + \[ + P(X \geq 2) = P(X = 2) + P(X = 3) = 3 \times 0.37^2 \times 0.63^1 + 0.37^3 \approx 0.31 + \] + + \item Il faut d'abord tracer le tableau résumant la loi de probabilité: + \begin{center} + \begin{tabular}{|c|*{4}{c|}} + \hline + Valeur & 0 & 1 & 2 & 3 \\ + \hline + Probabilité & $0.25$ & $0.441$ & $0.259$ &$0.051$ \\ + \hline + \end{tabular} + \end{center} + On peut alors calculer l'espérance + \[ + E[X] = 0 \times 0.25 + 1 \times 0.441 + 2 \times 0.259 + 3 \times 0.051 = 1.11 + \] + On peut donc estimer qu'il y aura en moyenne $1.11$ personnes qui feront sonner le portique sur les 3 personnes. + \end{enumerate} +\end{solution} + +\begin{exercise}[subtitle={Équation puissance}] + Résoudre les équations et inéquations suivantes + \begin{multicols}{2} + \begin{enumerate} + \item $10^x = 44$ + \item $2^x = 33$ + \item $0.94^x \leq 43$ + \item $9 \times 0.17^x = 30$ + \end{enumerate} + \end{multicols} +\end{exercise} + +\begin{solution} + Les solutions ci-dessous ne sont pas justifiée car l'ordinateur ne sait pas faire. Par contre, vous vous devez savoir justifier vos réponses! + \begin{enumerate} + \item $x = \log(44)$ + \item $x = \frac{\log(33)}{\log(2)}$ + \item Il faut faire attention quand on divise par un log car ce dernier peut être négatif ce qui est le cas ici. Il faut donc pense à changer le sens de l'inégalité. + + $x \geq \frac{\log(43)}{\log(0.94)}$ + + \item Il faut penser à faire la division à par $9$ avant d'utiliser le log car sinon, on ne peut pas utiliser la formule $\log(a^n) = n\times \log(a)$. + + $x = \frac{\log(3.33)}{\log(0.17)}$ + \end{enumerate} +\end{solution} + +\begin{exercise}[subtitle={Étude de fonctions}] + Soit $f(x) = - 2x^3 + 99x^2 + 2580x + 46$ une fonction définie sur $\R$. + \begin{enumerate} + \item Calculer $f'(x)$ la dérivée de $f(x)$. + \item Calculer $f'(43)$ et $f'(-10)$. + \item En déduire une forme factorisée de $f'(x)$. + \item Étudier le signe de $f'(x)$ et en déduire les variations de $f(x)$. + \item Est-ce que la fonction $f(x)$ admet un maximum ou un minimum? Si oui, calculer sa valeur. + \end{enumerate} +\end{exercise} + +\begin{solution} + \begin{enumerate} + \item Dérivée de $f(x)$: $f'(x) = - 6x^2 + 198x + 2580$ + \item + \begin{align*} + f'(43) &= - 6 \times 43^{2} + 198 \times 43 + 2580\\&= - 6 \times 1849 + 8514 + 2580\\&= - 11094 + 11094\\&= 0 + \end{align*} + \begin{align*} + f'(-10) &= - 6 \times - 10^{2} + 198(- 10) + 2580\\&= - 6 \times 100 - 1980 + 2580\\&= - 600 + 600\\&= 0 + \end{align*} + Donc $x = 43$ et $x=-10$ sont des racines de $f'(x) = - 6x^2 + 198x + 2580$. + \item On en déduit la forme factorisée suivante + \[ + f'(x) = -6 (x - 43)(x--10) + \] + \item Pas de correction disponible + \item À causes des branches extérieurs, la fonction $f(x)$ n'a pas de maximum ou de minimum. + \end{enumerate} +\end{solution} + + + +%\printsolutionstype{exercise} + + + +\end{document} + +%%% Local Variables: +%%% mode: latex +%%% TeX-master: "master" +%%% End: diff --git a/TST/DM/2102_DM2/TST3/corr_07_2102_DM2.tex b/TST/DM/2102_DM2/TST3/corr_07_2102_DM2.tex new file mode 100644 index 0000000..f3c9fc7 --- /dev/null +++ b/TST/DM/2102_DM2/TST3/corr_07_2102_DM2.tex @@ -0,0 +1,203 @@ +\documentclass[a5paper,10pt]{article} +\usepackage{myXsim} +\usepackage{tasks} + +% Title Page +\title{DM2 \hfill CAGLAR Rojin} +\tribe{TST} +\date{\hfillÀ render pour le Mercredi 24 février} + +\xsimsetup{ + solution/print = true +} + +\begin{document} +\maketitle + +\begin{exercise}[subtitle={Loi binomiale}] + Trois personnes s'apprêtent à passer le portique de sécurité. On suppose que pour chaque personne la probabilité que le portique sonne est égale à $0.86$. + + Soit $X$ la variable aléatoire donnant le nombre de personnes faisant sonner le portique, parmi les 3 personnes de ce groupe. + \begin{enumerate} + \item Tracer l'arbre représentant le situation. + \item Justifier que $X$ suit une loi binomiale dont on précisera les paramètres. + \item Quelle est la probabilité qu'une seule personne fasse sonner le portique? + \item Calculer puis interpréter les probabilités suivantes + \[ + P(X = 0) \qquad \qquad P(X \geq 2) + \] + \item Calculer l'espérance de $X$ et interpréter le résultat. + \end{enumerate} +\end{exercise} + +\begin{solution} + \begin{enumerate} + \item + \begin{tikzpicture}[sloped] + \node {.} + child {node {$0$} + child {node {$0$} + child {node {$0$} + edge from parent + node[above] {0.14} + } + child {node {$1$} + edge from parent + node[above] {0.86} + } + edge from parent + node[above] {0.14} + } + child[missing] {} + child {node {$1$} + child {node {$0$} + edge from parent + node[above] {0.14} + } + child {node {$1$} + edge from parent + node[above] {0.86} + } + edge from parent + node[above] {0.14} + } + edge from parent + node[above] {0.14} + } + child[missing] {} + child[missing] {} + child[missing] {} + child { node {$1$} + child {node {$0$} + child {node {$0$} + edge from parent + node[above] {0.14} + } + child {node {$1$} + edge from parent + node[above] {0.86} + } + edge from parent + node[above] {0.14} + } + child[missing] {} + child {node {$1$} + child {node {$0$} + edge from parent + node[above] {0.14} + } + child {node {$1$} + edge from parent + node[above] {0.86} + } + edge from parent + node[above] {0.14} + } + edge from parent + node[above] {0.86} + } ; + \end{tikzpicture} + \item Chaque personne a 2 possibilités (1: fait sonner ou 2: ne fait pas sonner) et l'on fait passer 3 personnes ce qui correspond à une répétition identique et aléatoire. On peut donc modéliser la situation par une loi binomiale. + \[ + X \sim \mathcal{B}(3; 0.76) + \] + \item Probabilité qu'une seule personne fasse sonner le portique. On voit qu'il y a 3 branches qui correspondent à cette situation dont + \[ + P(X = 1) = 3 \times 0.86^1 \times 0.14^2 \approx 0.051 + \] + \item + \[ + P(X = 0) = 0.14^3 \approx 0.003 + \] + \[ + P(X \geq 2) = P(X = 2) + P(X = 3) = 3 \times 0.86^2 \times 0.14^1 + 0.86^3 \approx 0.947 + \] + + \item Il faut d'abord tracer le tableau résumant la loi de probabilité: + \begin{center} + \begin{tabular}{|c|*{4}{c|}} + \hline + Valeur & 0 & 1 & 2 & 3 \\ + \hline + Probabilité & $0.003$ & $0.051$ & $0.311$ &$0.636$ \\ + \hline + \end{tabular} + \end{center} + On peut alors calculer l'espérance + \[ + E[X] = 0 \times 0.003 + 1 \times 0.051 + 2 \times 0.311 + 3 \times 0.636 = 2.58 + \] + On peut donc estimer qu'il y aura en moyenne $2.58$ personnes qui feront sonner le portique sur les 3 personnes. + \end{enumerate} +\end{solution} + +\begin{exercise}[subtitle={Équation puissance}] + Résoudre les équations et inéquations suivantes + \begin{multicols}{2} + \begin{enumerate} + \item $10^x = 14$ + \item $2^x = 38$ + \item $0.15^x \leq 40$ + \item $9 \times 0.56^x = 29$ + \end{enumerate} + \end{multicols} +\end{exercise} + +\begin{solution} + Les solutions ci-dessous ne sont pas justifiée car l'ordinateur ne sait pas faire. Par contre, vous vous devez savoir justifier vos réponses! + \begin{enumerate} + \item $x = \log(14)$ + \item $x = \frac{\log(38)}{\log(2)}$ + \item Il faut faire attention quand on divise par un log car ce dernier peut être négatif ce qui est le cas ici. Il faut donc pense à changer le sens de l'inégalité. + + $x \geq \frac{\log(40)}{\log(0.15)}$ + + \item Il faut penser à faire la division à par $9$ avant d'utiliser le log car sinon, on ne peut pas utiliser la formule $\log(a^n) = n\times \log(a)$. + + $x = \frac{\log(3.22)}{\log(0.56)}$ + \end{enumerate} +\end{solution} + +\begin{exercise}[subtitle={Étude de fonctions}] + Soit $f(x) = - x^3 + 67.5x^2 - 258x - 11$ une fonction définie sur $\R$. + \begin{enumerate} + \item Calculer $f'(x)$ la dérivée de $f(x)$. + \item Calculer $f'(43)$ et $f'(2)$. + \item En déduire une forme factorisée de $f'(x)$. + \item Étudier le signe de $f'(x)$ et en déduire les variations de $f(x)$. + \item Est-ce que la fonction $f(x)$ admet un maximum ou un minimum? Si oui, calculer sa valeur. + \end{enumerate} +\end{exercise} + +\begin{solution} + \begin{enumerate} + \item Dérivée de $f(x)$: $f'(x) = - 3x^2 + 135x - 258$ + \item + \begin{align*} + f'(43) &= - 3 \times 43^{2} + 135 \times 43 - 258\\&= - 3 \times 1849 + 5805 - 258\\&= - 5547 + 5547\\&= 0 + \end{align*} + \begin{align*} + f'(2) &= - 3 \times 2^{2} + 135 \times 2 - 258\\&= - 3 \times 4 + 270 - 258\\&= - 12 + 12\\&= 0 + \end{align*} + Donc $x = 43$ et $x=2$ sont des racines de $f'(x) = - 3x^2 + 135x - 258$. + \item On en déduit la forme factorisée suivante + \[ + f'(x) = -3 (x - 43)(x-2) + \] + \item Pas de correction disponible + \item À causes des branches extérieurs, la fonction $f(x)$ n'a pas de maximum ou de minimum. + \end{enumerate} +\end{solution} + + + +%\printsolutionstype{exercise} + + + +\end{document} + +%%% Local Variables: +%%% mode: latex +%%% TeX-master: "master" +%%% End: diff --git a/TST/DM/2102_DM2/TST3/corr_08_2102_DM2.tex b/TST/DM/2102_DM2/TST3/corr_08_2102_DM2.tex new file mode 100644 index 0000000..86b4b27 --- /dev/null +++ b/TST/DM/2102_DM2/TST3/corr_08_2102_DM2.tex @@ -0,0 +1,203 @@ +\documentclass[a5paper,10pt]{article} +\usepackage{myXsim} +\usepackage{tasks} + +% Title Page +\title{DM2 \hfill DARICHE Kaïs} +\tribe{TST} +\date{\hfillÀ render pour le Mercredi 24 février} + +\xsimsetup{ + solution/print = true +} + +\begin{document} +\maketitle + +\begin{exercise}[subtitle={Loi binomiale}] + Trois personnes s'apprêtent à passer le portique de sécurité. On suppose que pour chaque personne la probabilité que le portique sonne est égale à $0.16$. + + Soit $X$ la variable aléatoire donnant le nombre de personnes faisant sonner le portique, parmi les 3 personnes de ce groupe. + \begin{enumerate} + \item Tracer l'arbre représentant le situation. + \item Justifier que $X$ suit une loi binomiale dont on précisera les paramètres. + \item Quelle est la probabilité qu'une seule personne fasse sonner le portique? + \item Calculer puis interpréter les probabilités suivantes + \[ + P(X = 0) \qquad \qquad P(X \geq 2) + \] + \item Calculer l'espérance de $X$ et interpréter le résultat. + \end{enumerate} +\end{exercise} + +\begin{solution} + \begin{enumerate} + \item + \begin{tikzpicture}[sloped] + \node {.} + child {node {$0$} + child {node {$0$} + child {node {$0$} + edge from parent + node[above] {0.84} + } + child {node {$1$} + edge from parent + node[above] {0.16} + } + edge from parent + node[above] {0.84} + } + child[missing] {} + child {node {$1$} + child {node {$0$} + edge from parent + node[above] {0.84} + } + child {node {$1$} + edge from parent + node[above] {0.16} + } + edge from parent + node[above] {0.84} + } + edge from parent + node[above] {0.84} + } + child[missing] {} + child[missing] {} + child[missing] {} + child { node {$1$} + child {node {$0$} + child {node {$0$} + edge from parent + node[above] {0.84} + } + child {node {$1$} + edge from parent + node[above] {0.16} + } + edge from parent + node[above] {0.84} + } + child[missing] {} + child {node {$1$} + child {node {$0$} + edge from parent + node[above] {0.84} + } + child {node {$1$} + edge from parent + node[above] {0.16} + } + edge from parent + node[above] {0.84} + } + edge from parent + node[above] {0.16} + } ; + \end{tikzpicture} + \item Chaque personne a 2 possibilités (1: fait sonner ou 2: ne fait pas sonner) et l'on fait passer 3 personnes ce qui correspond à une répétition identique et aléatoire. On peut donc modéliser la situation par une loi binomiale. + \[ + X \sim \mathcal{B}(3; 0.76) + \] + \item Probabilité qu'une seule personne fasse sonner le portique. On voit qu'il y a 3 branches qui correspondent à cette situation dont + \[ + P(X = 1) = 3 \times 0.16^1 \times 0.84^2 \approx 0.339 + \] + \item + \[ + P(X = 0) = 0.84^3 \approx 0.593 + \] + \[ + P(X \geq 2) = P(X = 2) + P(X = 3) = 3 \times 0.16^2 \times 0.84^1 + 0.16^3 \approx 0.069 + \] + + \item Il faut d'abord tracer le tableau résumant la loi de probabilité: + \begin{center} + \begin{tabular}{|c|*{4}{c|}} + \hline + Valeur & 0 & 1 & 2 & 3 \\ + \hline + Probabilité & $0.593$ & $0.339$ & $0.065$ &$0.004$ \\ + \hline + \end{tabular} + \end{center} + On peut alors calculer l'espérance + \[ + E[X] = 0 \times 0.593 + 1 \times 0.339 + 2 \times 0.065 + 3 \times 0.004 = 0.48 + \] + On peut donc estimer qu'il y aura en moyenne $0.48$ personnes qui feront sonner le portique sur les 3 personnes. + \end{enumerate} +\end{solution} + +\begin{exercise}[subtitle={Équation puissance}] + Résoudre les équations et inéquations suivantes + \begin{multicols}{2} + \begin{enumerate} + \item $10^x = 35$ + \item $14^x = 11$ + \item $0.39^x \leq 48$ + \item $3 \times 0.07^x = 4$ + \end{enumerate} + \end{multicols} +\end{exercise} + +\begin{solution} + Les solutions ci-dessous ne sont pas justifiée car l'ordinateur ne sait pas faire. Par contre, vous vous devez savoir justifier vos réponses! + \begin{enumerate} + \item $x = \log(35)$ + \item $x = \frac{\log(11)}{\log(14)}$ + \item Il faut faire attention quand on divise par un log car ce dernier peut être négatif ce qui est le cas ici. Il faut donc pense à changer le sens de l'inégalité. + + $x \geq \frac{\log(48)}{\log(0.39)}$ + + \item Il faut penser à faire la division à par $3$ avant d'utiliser le log car sinon, on ne peut pas utiliser la formule $\log(a^n) = n\times \log(a)$. + + $x = \frac{\log(1.33)}{\log(0.07)}$ + \end{enumerate} +\end{solution} + +\begin{exercise}[subtitle={Étude de fonctions}] + Soit $f(x) = x^3 - 54x^2 + 780x + 36$ une fonction définie sur $\R$. + \begin{enumerate} + \item Calculer $f'(x)$ la dérivée de $f(x)$. + \item Calculer $f'(26)$ et $f'(10)$. + \item En déduire une forme factorisée de $f'(x)$. + \item Étudier le signe de $f'(x)$ et en déduire les variations de $f(x)$. + \item Est-ce que la fonction $f(x)$ admet un maximum ou un minimum? Si oui, calculer sa valeur. + \end{enumerate} +\end{exercise} + +\begin{solution} + \begin{enumerate} + \item Dérivée de $f(x)$: $f'(x) = 3x^2 - 108x + 780$ + \item + \begin{align*} + f'(26) &= 3 \times 26^{2} - 108 \times 26 + 780\\&= 3 \times 676 - 2808 + 780\\&= 2028 - 2028\\&= 0 + \end{align*} + \begin{align*} + f'(10) &= 3 \times 10^{2} - 108 \times 10 + 780\\&= 3 \times 100 - 1080 + 780\\&= 300 - 300\\&= 0 + \end{align*} + Donc $x = 26$ et $x=10$ sont des racines de $f'(x) = 3x^2 - 108x + 780$. + \item On en déduit la forme factorisée suivante + \[ + f'(x) = 3 (x - 26)(x-10) + \] + \item Pas de correction disponible + \item À causes des branches extérieurs, la fonction $f(x)$ n'a pas de maximum ou de minimum. + \end{enumerate} +\end{solution} + + + +%\printsolutionstype{exercise} + + + +\end{document} + +%%% Local Variables: +%%% mode: latex +%%% TeX-master: "master" +%%% End: diff --git a/TST/DM/2102_DM2/TST3/corr_09_2102_DM2.tex b/TST/DM/2102_DM2/TST3/corr_09_2102_DM2.tex new file mode 100644 index 0000000..9047dbb --- /dev/null +++ b/TST/DM/2102_DM2/TST3/corr_09_2102_DM2.tex @@ -0,0 +1,203 @@ +\documentclass[a5paper,10pt]{article} +\usepackage{myXsim} +\usepackage{tasks} + +% Title Page +\title{DM2 \hfill DEBRAS Noémie} +\tribe{TST} +\date{\hfillÀ render pour le Mercredi 24 février} + +\xsimsetup{ + solution/print = true +} + +\begin{document} +\maketitle + +\begin{exercise}[subtitle={Loi binomiale}] + Trois personnes s'apprêtent à passer le portique de sécurité. On suppose que pour chaque personne la probabilité que le portique sonne est égale à $0.1$. + + Soit $X$ la variable aléatoire donnant le nombre de personnes faisant sonner le portique, parmi les 3 personnes de ce groupe. + \begin{enumerate} + \item Tracer l'arbre représentant le situation. + \item Justifier que $X$ suit une loi binomiale dont on précisera les paramètres. + \item Quelle est la probabilité qu'une seule personne fasse sonner le portique? + \item Calculer puis interpréter les probabilités suivantes + \[ + P(X = 0) \qquad \qquad P(X \geq 2) + \] + \item Calculer l'espérance de $X$ et interpréter le résultat. + \end{enumerate} +\end{exercise} + +\begin{solution} + \begin{enumerate} + \item + \begin{tikzpicture}[sloped] + \node {.} + child {node {$0$} + child {node {$0$} + child {node {$0$} + edge from parent + node[above] {0.9} + } + child {node {$1$} + edge from parent + node[above] {0.1} + } + edge from parent + node[above] {0.9} + } + child[missing] {} + child {node {$1$} + child {node {$0$} + edge from parent + node[above] {0.9} + } + child {node {$1$} + edge from parent + node[above] {0.1} + } + edge from parent + node[above] {0.9} + } + edge from parent + node[above] {0.9} + } + child[missing] {} + child[missing] {} + child[missing] {} + child { node {$1$} + child {node {$0$} + child {node {$0$} + edge from parent + node[above] {0.9} + } + child {node {$1$} + edge from parent + node[above] {0.1} + } + edge from parent + node[above] {0.9} + } + child[missing] {} + child {node {$1$} + child {node {$0$} + edge from parent + node[above] {0.9} + } + child {node {$1$} + edge from parent + node[above] {0.1} + } + edge from parent + node[above] {0.9} + } + edge from parent + node[above] {0.1} + } ; + \end{tikzpicture} + \item Chaque personne a 2 possibilités (1: fait sonner ou 2: ne fait pas sonner) et l'on fait passer 3 personnes ce qui correspond à une répétition identique et aléatoire. On peut donc modéliser la situation par une loi binomiale. + \[ + X \sim \mathcal{B}(3; 0.76) + \] + \item Probabilité qu'une seule personne fasse sonner le portique. On voit qu'il y a 3 branches qui correspondent à cette situation dont + \[ + P(X = 1) = 3 \times 0.1^1 \times 0.9^2 \approx 0.243 + \] + \item + \[ + P(X = 0) = 0.9^3 \approx 0.729 + \] + \[ + P(X \geq 2) = P(X = 2) + P(X = 3) = 3 \times 0.1^2 \times 0.9^1 + 0.1^3 \approx 0.028 + \] + + \item Il faut d'abord tracer le tableau résumant la loi de probabilité: + \begin{center} + \begin{tabular}{|c|*{4}{c|}} + \hline + Valeur & 0 & 1 & 2 & 3 \\ + \hline + Probabilité & $0.729$ & $0.243$ & $0.027$ &$0.001$ \\ + \hline + \end{tabular} + \end{center} + On peut alors calculer l'espérance + \[ + E[X] = 0 \times 0.729 + 1 \times 0.243 + 2 \times 0.027 + 3 \times 0.001 = 0.3 + \] + On peut donc estimer qu'il y aura en moyenne $0.3$ personnes qui feront sonner le portique sur les 3 personnes. + \end{enumerate} +\end{solution} + +\begin{exercise}[subtitle={Équation puissance}] + Résoudre les équations et inéquations suivantes + \begin{multicols}{2} + \begin{enumerate} + \item $10^x = 25$ + \item $12^x = 21$ + \item $0.77^x \leq 22$ + \item $6 \times 0.4^x = 40$ + \end{enumerate} + \end{multicols} +\end{exercise} + +\begin{solution} + Les solutions ci-dessous ne sont pas justifiée car l'ordinateur ne sait pas faire. Par contre, vous vous devez savoir justifier vos réponses! + \begin{enumerate} + \item $x = \log(25)$ + \item $x = \frac{\log(21)}{\log(12)}$ + \item Il faut faire attention quand on divise par un log car ce dernier peut être négatif ce qui est le cas ici. Il faut donc pense à changer le sens de l'inégalité. + + $x \geq \frac{\log(22)}{\log(0.77)}$ + + \item Il faut penser à faire la division à par $6$ avant d'utiliser le log car sinon, on ne peut pas utiliser la formule $\log(a^n) = n\times \log(a)$. + + $x = \frac{\log(6.67)}{\log(0.4)}$ + \end{enumerate} +\end{solution} + +\begin{exercise}[subtitle={Étude de fonctions}] + Soit $f(x) = 4x^3 - 132x^2 - 5460x + 39$ une fonction définie sur $\R$. + \begin{enumerate} + \item Calculer $f'(x)$ la dérivée de $f(x)$. + \item Calculer $f'(35)$ et $f'(-13)$. + \item En déduire une forme factorisée de $f'(x)$. + \item Étudier le signe de $f'(x)$ et en déduire les variations de $f(x)$. + \item Est-ce que la fonction $f(x)$ admet un maximum ou un minimum? Si oui, calculer sa valeur. + \end{enumerate} +\end{exercise} + +\begin{solution} + \begin{enumerate} + \item Dérivée de $f(x)$: $f'(x) = 12x^2 - 264x - 5460$ + \item + \begin{align*} + f'(35) &= 12 \times 35^{2} - 264 \times 35 - 5460\\&= 12 \times 1225 - 9240 - 5460\\&= 14700 - 14700\\&= 0 + \end{align*} + \begin{align*} + f'(-13) &= 12 \times - 13^{2} - 264(- 13) - 5460\\&= 12 \times 169 + 3432 - 5460\\&= 2028 - 2028\\&= 0 + \end{align*} + Donc $x = 35$ et $x=-13$ sont des racines de $f'(x) = 12x^2 - 264x - 5460$. + \item On en déduit la forme factorisée suivante + \[ + f'(x) = 12 (x - 35)(x--13) + \] + \item Pas de correction disponible + \item À causes des branches extérieurs, la fonction $f(x)$ n'a pas de maximum ou de minimum. + \end{enumerate} +\end{solution} + + + +%\printsolutionstype{exercise} + + + +\end{document} + +%%% Local Variables: +%%% mode: latex +%%% TeX-master: "master" +%%% End: diff --git a/TST/DM/2102_DM2/TST3/corr_10_2102_DM2.tex b/TST/DM/2102_DM2/TST3/corr_10_2102_DM2.tex new file mode 100644 index 0000000..c1f9b8b --- /dev/null +++ b/TST/DM/2102_DM2/TST3/corr_10_2102_DM2.tex @@ -0,0 +1,203 @@ +\documentclass[a5paper,10pt]{article} +\usepackage{myXsim} +\usepackage{tasks} + +% Title Page +\title{DM2 \hfill GERMAIN Anaïs} +\tribe{TST} +\date{\hfillÀ render pour le Mercredi 24 février} + +\xsimsetup{ + solution/print = true +} + +\begin{document} +\maketitle + +\begin{exercise}[subtitle={Loi binomiale}] + Trois personnes s'apprêtent à passer le portique de sécurité. On suppose que pour chaque personne la probabilité que le portique sonne est égale à $0.68$. + + Soit $X$ la variable aléatoire donnant le nombre de personnes faisant sonner le portique, parmi les 3 personnes de ce groupe. + \begin{enumerate} + \item Tracer l'arbre représentant le situation. + \item Justifier que $X$ suit une loi binomiale dont on précisera les paramètres. + \item Quelle est la probabilité qu'une seule personne fasse sonner le portique? + \item Calculer puis interpréter les probabilités suivantes + \[ + P(X = 0) \qquad \qquad P(X \geq 2) + \] + \item Calculer l'espérance de $X$ et interpréter le résultat. + \end{enumerate} +\end{exercise} + +\begin{solution} + \begin{enumerate} + \item + \begin{tikzpicture}[sloped] + \node {.} + child {node {$0$} + child {node {$0$} + child {node {$0$} + edge from parent + node[above] {0.32} + } + child {node {$1$} + edge from parent + node[above] {0.68} + } + edge from parent + node[above] {0.32} + } + child[missing] {} + child {node {$1$} + child {node {$0$} + edge from parent + node[above] {0.32} + } + child {node {$1$} + edge from parent + node[above] {0.68} + } + edge from parent + node[above] {0.32} + } + edge from parent + node[above] {0.32} + } + child[missing] {} + child[missing] {} + child[missing] {} + child { node {$1$} + child {node {$0$} + child {node {$0$} + edge from parent + node[above] {0.32} + } + child {node {$1$} + edge from parent + node[above] {0.68} + } + edge from parent + node[above] {0.32} + } + child[missing] {} + child {node {$1$} + child {node {$0$} + edge from parent + node[above] {0.32} + } + child {node {$1$} + edge from parent + node[above] {0.68} + } + edge from parent + node[above] {0.32} + } + edge from parent + node[above] {0.68} + } ; + \end{tikzpicture} + \item Chaque personne a 2 possibilités (1: fait sonner ou 2: ne fait pas sonner) et l'on fait passer 3 personnes ce qui correspond à une répétition identique et aléatoire. On peut donc modéliser la situation par une loi binomiale. + \[ + X \sim \mathcal{B}(3; 0.76) + \] + \item Probabilité qu'une seule personne fasse sonner le portique. On voit qu'il y a 3 branches qui correspondent à cette situation dont + \[ + P(X = 1) = 3 \times 0.68^1 \times 0.32^2 \approx 0.209 + \] + \item + \[ + P(X = 0) = 0.32^3 \approx 0.033 + \] + \[ + P(X \geq 2) = P(X = 2) + P(X = 3) = 3 \times 0.68^2 \times 0.32^1 + 0.68^3 \approx 0.758 + \] + + \item Il faut d'abord tracer le tableau résumant la loi de probabilité: + \begin{center} + \begin{tabular}{|c|*{4}{c|}} + \hline + Valeur & 0 & 1 & 2 & 3 \\ + \hline + Probabilité & $0.033$ & $0.209$ & $0.444$ &$0.314$ \\ + \hline + \end{tabular} + \end{center} + On peut alors calculer l'espérance + \[ + E[X] = 0 \times 0.033 + 1 \times 0.209 + 2 \times 0.444 + 3 \times 0.314 = 2.04 + \] + On peut donc estimer qu'il y aura en moyenne $2.04$ personnes qui feront sonner le portique sur les 3 personnes. + \end{enumerate} +\end{solution} + +\begin{exercise}[subtitle={Équation puissance}] + Résoudre les équations et inéquations suivantes + \begin{multicols}{2} + \begin{enumerate} + \item $10^x = 48$ + \item $2^x = 42$ + \item $0.47^x \leq 46$ + \item $7 \times 0.37^x = 45$ + \end{enumerate} + \end{multicols} +\end{exercise} + +\begin{solution} + Les solutions ci-dessous ne sont pas justifiée car l'ordinateur ne sait pas faire. Par contre, vous vous devez savoir justifier vos réponses! + \begin{enumerate} + \item $x = \log(48)$ + \item $x = \frac{\log(42)}{\log(2)}$ + \item Il faut faire attention quand on divise par un log car ce dernier peut être négatif ce qui est le cas ici. Il faut donc pense à changer le sens de l'inégalité. + + $x \geq \frac{\log(46)}{\log(0.47)}$ + + \item Il faut penser à faire la division à par $7$ avant d'utiliser le log car sinon, on ne peut pas utiliser la formule $\log(a^n) = n\times \log(a)$. + + $x = \frac{\log(6.43)}{\log(0.37)}$ + \end{enumerate} +\end{solution} + +\begin{exercise}[subtitle={Étude de fonctions}] + Soit $f(x) = - 7x^3 + 472.5x^2 + 4116x + 48$ une fonction définie sur $\R$. + \begin{enumerate} + \item Calculer $f'(x)$ la dérivée de $f(x)$. + \item Calculer $f'(49)$ et $f'(-4)$. + \item En déduire une forme factorisée de $f'(x)$. + \item Étudier le signe de $f'(x)$ et en déduire les variations de $f(x)$. + \item Est-ce que la fonction $f(x)$ admet un maximum ou un minimum? Si oui, calculer sa valeur. + \end{enumerate} +\end{exercise} + +\begin{solution} + \begin{enumerate} + \item Dérivée de $f(x)$: $f'(x) = - 21x^2 + 945x + 4116$ + \item + \begin{align*} + f'(49) &= - 21 \times 49^{2} + 945 \times 49 + 4116\\&= - 21 \times 2401 + 46305 + 4116\\&= - 50421 + 50421\\&= 0 + \end{align*} + \begin{align*} + f'(-4) &= - 21 \times - 4^{2} + 945(- 4) + 4116\\&= - 21 \times 16 - 3780 + 4116\\&= - 336 + 336\\&= 0 + \end{align*} + Donc $x = 49$ et $x=-4$ sont des racines de $f'(x) = - 21x^2 + 945x + 4116$. + \item On en déduit la forme factorisée suivante + \[ + f'(x) = -21 (x - 49)(x--4) + \] + \item Pas de correction disponible + \item À causes des branches extérieurs, la fonction $f(x)$ n'a pas de maximum ou de minimum. + \end{enumerate} +\end{solution} + + + +%\printsolutionstype{exercise} + + + +\end{document} + +%%% Local Variables: +%%% mode: latex +%%% TeX-master: "master" +%%% End: diff --git a/TST/DM/2102_DM2/TST3/corr_11_2102_DM2.tex b/TST/DM/2102_DM2/TST3/corr_11_2102_DM2.tex new file mode 100644 index 0000000..f378033 --- /dev/null +++ b/TST/DM/2102_DM2/TST3/corr_11_2102_DM2.tex @@ -0,0 +1,203 @@ +\documentclass[a5paper,10pt]{article} +\usepackage{myXsim} +\usepackage{tasks} + +% Title Page +\title{DM2 \hfill HADJRAS Mohcine} +\tribe{TST} +\date{\hfillÀ render pour le Mercredi 24 février} + +\xsimsetup{ + solution/print = true +} + +\begin{document} +\maketitle + +\begin{exercise}[subtitle={Loi binomiale}] + Trois personnes s'apprêtent à passer le portique de sécurité. On suppose que pour chaque personne la probabilité que le portique sonne est égale à $0.31$. + + Soit $X$ la variable aléatoire donnant le nombre de personnes faisant sonner le portique, parmi les 3 personnes de ce groupe. + \begin{enumerate} + \item Tracer l'arbre représentant le situation. + \item Justifier que $X$ suit une loi binomiale dont on précisera les paramètres. + \item Quelle est la probabilité qu'une seule personne fasse sonner le portique? + \item Calculer puis interpréter les probabilités suivantes + \[ + P(X = 0) \qquad \qquad P(X \geq 2) + \] + \item Calculer l'espérance de $X$ et interpréter le résultat. + \end{enumerate} +\end{exercise} + +\begin{solution} + \begin{enumerate} + \item + \begin{tikzpicture}[sloped] + \node {.} + child {node {$0$} + child {node {$0$} + child {node {$0$} + edge from parent + node[above] {0.69} + } + child {node {$1$} + edge from parent + node[above] {0.31} + } + edge from parent + node[above] {0.69} + } + child[missing] {} + child {node {$1$} + child {node {$0$} + edge from parent + node[above] {0.69} + } + child {node {$1$} + edge from parent + node[above] {0.31} + } + edge from parent + node[above] {0.69} + } + edge from parent + node[above] {0.69} + } + child[missing] {} + child[missing] {} + child[missing] {} + child { node {$1$} + child {node {$0$} + child {node {$0$} + edge from parent + node[above] {0.69} + } + child {node {$1$} + edge from parent + node[above] {0.31} + } + edge from parent + node[above] {0.69} + } + child[missing] {} + child {node {$1$} + child {node {$0$} + edge from parent + node[above] {0.69} + } + child {node {$1$} + edge from parent + node[above] {0.31} + } + edge from parent + node[above] {0.69} + } + edge from parent + node[above] {0.31} + } ; + \end{tikzpicture} + \item Chaque personne a 2 possibilités (1: fait sonner ou 2: ne fait pas sonner) et l'on fait passer 3 personnes ce qui correspond à une répétition identique et aléatoire. On peut donc modéliser la situation par une loi binomiale. + \[ + X \sim \mathcal{B}(3; 0.76) + \] + \item Probabilité qu'une seule personne fasse sonner le portique. On voit qu'il y a 3 branches qui correspondent à cette situation dont + \[ + P(X = 1) = 3 \times 0.31^1 \times 0.69^2 \approx 0.443 + \] + \item + \[ + P(X = 0) = 0.69^3 \approx 0.329 + \] + \[ + P(X \geq 2) = P(X = 2) + P(X = 3) = 3 \times 0.31^2 \times 0.69^1 + 0.31^3 \approx 0.229 + \] + + \item Il faut d'abord tracer le tableau résumant la loi de probabilité: + \begin{center} + \begin{tabular}{|c|*{4}{c|}} + \hline + Valeur & 0 & 1 & 2 & 3 \\ + \hline + Probabilité & $0.329$ & $0.443$ & $0.199$ &$0.03$ \\ + \hline + \end{tabular} + \end{center} + On peut alors calculer l'espérance + \[ + E[X] = 0 \times 0.329 + 1 \times 0.443 + 2 \times 0.199 + 3 \times 0.03 = 0.93 + \] + On peut donc estimer qu'il y aura en moyenne $0.93$ personnes qui feront sonner le portique sur les 3 personnes. + \end{enumerate} +\end{solution} + +\begin{exercise}[subtitle={Équation puissance}] + Résoudre les équations et inéquations suivantes + \begin{multicols}{2} + \begin{enumerate} + \item $10^x = 10$ + \item $11^x = 20$ + \item $0.09^x \leq 22$ + \item $6 \times 0.16^x = 45$ + \end{enumerate} + \end{multicols} +\end{exercise} + +\begin{solution} + Les solutions ci-dessous ne sont pas justifiée car l'ordinateur ne sait pas faire. Par contre, vous vous devez savoir justifier vos réponses! + \begin{enumerate} + \item $x = \log(10)$ + \item $x = \frac{\log(20)}{\log(11)}$ + \item Il faut faire attention quand on divise par un log car ce dernier peut être négatif ce qui est le cas ici. Il faut donc pense à changer le sens de l'inégalité. + + $x \geq \frac{\log(22)}{\log(0.09)}$ + + \item Il faut penser à faire la division à par $6$ avant d'utiliser le log car sinon, on ne peut pas utiliser la formule $\log(a^n) = n\times \log(a)$. + + $x = \frac{\log(7.5)}{\log(0.16)}$ + \end{enumerate} +\end{solution} + +\begin{exercise}[subtitle={Étude de fonctions}] + Soit $f(x) = 3x^3 - 225x^2 + 4896x + 16$ une fonction définie sur $\R$. + \begin{enumerate} + \item Calculer $f'(x)$ la dérivée de $f(x)$. + \item Calculer $f'(34)$ et $f'(16)$. + \item En déduire une forme factorisée de $f'(x)$. + \item Étudier le signe de $f'(x)$ et en déduire les variations de $f(x)$. + \item Est-ce que la fonction $f(x)$ admet un maximum ou un minimum? Si oui, calculer sa valeur. + \end{enumerate} +\end{exercise} + +\begin{solution} + \begin{enumerate} + \item Dérivée de $f(x)$: $f'(x) = 9x^2 - 450x + 4896$ + \item + \begin{align*} + f'(34) &= 9 \times 34^{2} - 450 \times 34 + 4896\\&= 9 \times 1156 - 15300 + 4896\\&= 10404 - 10404\\&= 0 + \end{align*} + \begin{align*} + f'(16) &= 9 \times 16^{2} - 450 \times 16 + 4896\\&= 9 \times 256 - 7200 + 4896\\&= 2304 - 2304\\&= 0 + \end{align*} + Donc $x = 34$ et $x=16$ sont des racines de $f'(x) = 9x^2 - 450x + 4896$. + \item On en déduit la forme factorisée suivante + \[ + f'(x) = 9 (x - 34)(x-16) + \] + \item Pas de correction disponible + \item À causes des branches extérieurs, la fonction $f(x)$ n'a pas de maximum ou de minimum. + \end{enumerate} +\end{solution} + + + +%\printsolutionstype{exercise} + + + +\end{document} + +%%% Local Variables: +%%% mode: latex +%%% TeX-master: "master" +%%% End: diff --git a/TST/DM/2102_DM2/TST3/corr_12_2102_DM2.tex b/TST/DM/2102_DM2/TST3/corr_12_2102_DM2.tex new file mode 100644 index 0000000..e4688e7 --- /dev/null +++ b/TST/DM/2102_DM2/TST3/corr_12_2102_DM2.tex @@ -0,0 +1,203 @@ +\documentclass[a5paper,10pt]{article} +\usepackage{myXsim} +\usepackage{tasks} + +% Title Page +\title{DM2 \hfill HENRIST Maxime} +\tribe{TST} +\date{\hfillÀ render pour le Mercredi 24 février} + +\xsimsetup{ + solution/print = true +} + +\begin{document} +\maketitle + +\begin{exercise}[subtitle={Loi binomiale}] + Trois personnes s'apprêtent à passer le portique de sécurité. On suppose que pour chaque personne la probabilité que le portique sonne est égale à $0.41$. + + Soit $X$ la variable aléatoire donnant le nombre de personnes faisant sonner le portique, parmi les 3 personnes de ce groupe. + \begin{enumerate} + \item Tracer l'arbre représentant le situation. + \item Justifier que $X$ suit une loi binomiale dont on précisera les paramètres. + \item Quelle est la probabilité qu'une seule personne fasse sonner le portique? + \item Calculer puis interpréter les probabilités suivantes + \[ + P(X = 0) \qquad \qquad P(X \geq 2) + \] + \item Calculer l'espérance de $X$ et interpréter le résultat. + \end{enumerate} +\end{exercise} + +\begin{solution} + \begin{enumerate} + \item + \begin{tikzpicture}[sloped] + \node {.} + child {node {$0$} + child {node {$0$} + child {node {$0$} + edge from parent + node[above] {0.59} + } + child {node {$1$} + edge from parent + node[above] {0.41} + } + edge from parent + node[above] {0.59} + } + child[missing] {} + child {node {$1$} + child {node {$0$} + edge from parent + node[above] {0.59} + } + child {node {$1$} + edge from parent + node[above] {0.41} + } + edge from parent + node[above] {0.59} + } + edge from parent + node[above] {0.59} + } + child[missing] {} + child[missing] {} + child[missing] {} + child { node {$1$} + child {node {$0$} + child {node {$0$} + edge from parent + node[above] {0.59} + } + child {node {$1$} + edge from parent + node[above] {0.41} + } + edge from parent + node[above] {0.59} + } + child[missing] {} + child {node {$1$} + child {node {$0$} + edge from parent + node[above] {0.59} + } + child {node {$1$} + edge from parent + node[above] {0.41} + } + edge from parent + node[above] {0.59} + } + edge from parent + node[above] {0.41} + } ; + \end{tikzpicture} + \item Chaque personne a 2 possibilités (1: fait sonner ou 2: ne fait pas sonner) et l'on fait passer 3 personnes ce qui correspond à une répétition identique et aléatoire. On peut donc modéliser la situation par une loi binomiale. + \[ + X \sim \mathcal{B}(3; 0.76) + \] + \item Probabilité qu'une seule personne fasse sonner le portique. On voit qu'il y a 3 branches qui correspondent à cette situation dont + \[ + P(X = 1) = 3 \times 0.41^1 \times 0.59^2 \approx 0.428 + \] + \item + \[ + P(X = 0) = 0.59^3 \approx 0.205 + \] + \[ + P(X \geq 2) = P(X = 2) + P(X = 3) = 3 \times 0.41^2 \times 0.59^1 + 0.41^3 \approx 0.367 + \] + + \item Il faut d'abord tracer le tableau résumant la loi de probabilité: + \begin{center} + \begin{tabular}{|c|*{4}{c|}} + \hline + Valeur & 0 & 1 & 2 & 3 \\ + \hline + Probabilité & $0.205$ & $0.428$ & $0.298$ &$0.069$ \\ + \hline + \end{tabular} + \end{center} + On peut alors calculer l'espérance + \[ + E[X] = 0 \times 0.205 + 1 \times 0.428 + 2 \times 0.298 + 3 \times 0.069 = 1.23 + \] + On peut donc estimer qu'il y aura en moyenne $1.23$ personnes qui feront sonner le portique sur les 3 personnes. + \end{enumerate} +\end{solution} + +\begin{exercise}[subtitle={Équation puissance}] + Résoudre les équations et inéquations suivantes + \begin{multicols}{2} + \begin{enumerate} + \item $10^x = 24$ + \item $14^x = 16$ + \item $0.35^x \leq 34$ + \item $8 \times 0.25^x = 36$ + \end{enumerate} + \end{multicols} +\end{exercise} + +\begin{solution} + Les solutions ci-dessous ne sont pas justifiée car l'ordinateur ne sait pas faire. Par contre, vous vous devez savoir justifier vos réponses! + \begin{enumerate} + \item $x = \log(24)$ + \item $x = \frac{\log(16)}{\log(14)}$ + \item Il faut faire attention quand on divise par un log car ce dernier peut être négatif ce qui est le cas ici. Il faut donc pense à changer le sens de l'inégalité. + + $x \geq \frac{\log(34)}{\log(0.35)}$ + + \item Il faut penser à faire la division à par $8$ avant d'utiliser le log car sinon, on ne peut pas utiliser la formule $\log(a^n) = n\times \log(a)$. + + $x = \frac{\log(4.5)}{\log(0.25)}$ + \end{enumerate} +\end{solution} + +\begin{exercise}[subtitle={Étude de fonctions}] + Soit $f(x) = 7x^3 - 346.5x^2 - 4914x + 30$ une fonction définie sur $\R$. + \begin{enumerate} + \item Calculer $f'(x)$ la dérivée de $f(x)$. + \item Calculer $f'(39)$ et $f'(-6)$. + \item En déduire une forme factorisée de $f'(x)$. + \item Étudier le signe de $f'(x)$ et en déduire les variations de $f(x)$. + \item Est-ce que la fonction $f(x)$ admet un maximum ou un minimum? Si oui, calculer sa valeur. + \end{enumerate} +\end{exercise} + +\begin{solution} + \begin{enumerate} + \item Dérivée de $f(x)$: $f'(x) = 21x^2 - 693x - 4914$ + \item + \begin{align*} + f'(39) &= 21 \times 39^{2} - 693 \times 39 - 4914\\&= 21 \times 1521 - 27027 - 4914\\&= 31941 - 31941\\&= 0 + \end{align*} + \begin{align*} + f'(-6) &= 21 \times - 6^{2} - 693(- 6) - 4914\\&= 21 \times 36 + 4158 - 4914\\&= 756 - 756\\&= 0 + \end{align*} + Donc $x = 39$ et $x=-6$ sont des racines de $f'(x) = 21x^2 - 693x - 4914$. + \item On en déduit la forme factorisée suivante + \[ + f'(x) = 21 (x - 39)(x--6) + \] + \item Pas de correction disponible + \item À causes des branches extérieurs, la fonction $f(x)$ n'a pas de maximum ou de minimum. + \end{enumerate} +\end{solution} + + + +%\printsolutionstype{exercise} + + + +\end{document} + +%%% Local Variables: +%%% mode: latex +%%% TeX-master: "master" +%%% End: diff --git a/TST/DM/2102_DM2/TST3/corr_13_2102_DM2.tex b/TST/DM/2102_DM2/TST3/corr_13_2102_DM2.tex new file mode 100644 index 0000000..e5ded19 --- /dev/null +++ b/TST/DM/2102_DM2/TST3/corr_13_2102_DM2.tex @@ -0,0 +1,203 @@ +\documentclass[a5paper,10pt]{article} +\usepackage{myXsim} +\usepackage{tasks} + +% Title Page +\title{DM2 \hfill INFANTES Antoine} +\tribe{TST} +\date{\hfillÀ render pour le Mercredi 24 février} + +\xsimsetup{ + solution/print = true +} + +\begin{document} +\maketitle + +\begin{exercise}[subtitle={Loi binomiale}] + Trois personnes s'apprêtent à passer le portique de sécurité. On suppose que pour chaque personne la probabilité que le portique sonne est égale à $0.58$. + + Soit $X$ la variable aléatoire donnant le nombre de personnes faisant sonner le portique, parmi les 3 personnes de ce groupe. + \begin{enumerate} + \item Tracer l'arbre représentant le situation. + \item Justifier que $X$ suit une loi binomiale dont on précisera les paramètres. + \item Quelle est la probabilité qu'une seule personne fasse sonner le portique? + \item Calculer puis interpréter les probabilités suivantes + \[ + P(X = 0) \qquad \qquad P(X \geq 2) + \] + \item Calculer l'espérance de $X$ et interpréter le résultat. + \end{enumerate} +\end{exercise} + +\begin{solution} + \begin{enumerate} + \item + \begin{tikzpicture}[sloped] + \node {.} + child {node {$0$} + child {node {$0$} + child {node {$0$} + edge from parent + node[above] {0.42} + } + child {node {$1$} + edge from parent + node[above] {0.58} + } + edge from parent + node[above] {0.42} + } + child[missing] {} + child {node {$1$} + child {node {$0$} + edge from parent + node[above] {0.42} + } + child {node {$1$} + edge from parent + node[above] {0.58} + } + edge from parent + node[above] {0.42} + } + edge from parent + node[above] {0.42} + } + child[missing] {} + child[missing] {} + child[missing] {} + child { node {$1$} + child {node {$0$} + child {node {$0$} + edge from parent + node[above] {0.42} + } + child {node {$1$} + edge from parent + node[above] {0.58} + } + edge from parent + node[above] {0.42} + } + child[missing] {} + child {node {$1$} + child {node {$0$} + edge from parent + node[above] {0.42} + } + child {node {$1$} + edge from parent + node[above] {0.58} + } + edge from parent + node[above] {0.42} + } + edge from parent + node[above] {0.58} + } ; + \end{tikzpicture} + \item Chaque personne a 2 possibilités (1: fait sonner ou 2: ne fait pas sonner) et l'on fait passer 3 personnes ce qui correspond à une répétition identique et aléatoire. On peut donc modéliser la situation par une loi binomiale. + \[ + X \sim \mathcal{B}(3; 0.76) + \] + \item Probabilité qu'une seule personne fasse sonner le portique. On voit qu'il y a 3 branches qui correspondent à cette situation dont + \[ + P(X = 1) = 3 \times 0.58^1 \times 0.42^2 \approx 0.307 + \] + \item + \[ + P(X = 0) = 0.42^3 \approx 0.074 + \] + \[ + P(X \geq 2) = P(X = 2) + P(X = 3) = 3 \times 0.58^2 \times 0.42^1 + 0.58^3 \approx 0.619 + \] + + \item Il faut d'abord tracer le tableau résumant la loi de probabilité: + \begin{center} + \begin{tabular}{|c|*{4}{c|}} + \hline + Valeur & 0 & 1 & 2 & 3 \\ + \hline + Probabilité & $0.074$ & $0.307$ & $0.424$ &$0.195$ \\ + \hline + \end{tabular} + \end{center} + On peut alors calculer l'espérance + \[ + E[X] = 0 \times 0.074 + 1 \times 0.307 + 2 \times 0.424 + 3 \times 0.195 = 1.74 + \] + On peut donc estimer qu'il y aura en moyenne $1.74$ personnes qui feront sonner le portique sur les 3 personnes. + \end{enumerate} +\end{solution} + +\begin{exercise}[subtitle={Équation puissance}] + Résoudre les équations et inéquations suivantes + \begin{multicols}{2} + \begin{enumerate} + \item $10^x = 1$ + \item $11^x = 10$ + \item $0.52^x \leq 14$ + \item $5 \times 0.49^x = 45$ + \end{enumerate} + \end{multicols} +\end{exercise} + +\begin{solution} + Les solutions ci-dessous ne sont pas justifiée car l'ordinateur ne sait pas faire. Par contre, vous vous devez savoir justifier vos réponses! + \begin{enumerate} + \item $x = \log(1)$ + \item $x = \frac{\log(10)}{\log(11)}$ + \item Il faut faire attention quand on divise par un log car ce dernier peut être négatif ce qui est le cas ici. Il faut donc pense à changer le sens de l'inégalité. + + $x \geq \frac{\log(14)}{\log(0.52)}$ + + \item Il faut penser à faire la division à par $5$ avant d'utiliser le log car sinon, on ne peut pas utiliser la formule $\log(a^n) = n\times \log(a)$. + + $x = \frac{\log(9.0)}{\log(0.49)}$ + \end{enumerate} +\end{solution} + +\begin{exercise}[subtitle={Étude de fonctions}] + Soit $f(x) = - 9x^3 + 364.5x^2 + 756x - 21$ une fonction définie sur $\R$. + \begin{enumerate} + \item Calculer $f'(x)$ la dérivée de $f(x)$. + \item Calculer $f'(28)$ et $f'(-1)$. + \item En déduire une forme factorisée de $f'(x)$. + \item Étudier le signe de $f'(x)$ et en déduire les variations de $f(x)$. + \item Est-ce que la fonction $f(x)$ admet un maximum ou un minimum? Si oui, calculer sa valeur. + \end{enumerate} +\end{exercise} + +\begin{solution} + \begin{enumerate} + \item Dérivée de $f(x)$: $f'(x) = - 27x^2 + 729x + 756$ + \item + \begin{align*} + f'(28) &= - 27 \times 28^{2} + 729 \times 28 + 756\\&= - 27 \times 784 + 20412 + 756\\&= - 21168 + 21168\\&= 0 + \end{align*} + \begin{align*} + f'(-1) &= - 27 \times - 1^{2} + 729(- 1) + 756\\&= - 27 \times 1 - 729 + 756\\&= - 27 + 27\\&= 0 + \end{align*} + Donc $x = 28$ et $x=-1$ sont des racines de $f'(x) = - 27x^2 + 729x + 756$. + \item On en déduit la forme factorisée suivante + \[ + f'(x) = -27 (x - 28)(x--1) + \] + \item Pas de correction disponible + \item À causes des branches extérieurs, la fonction $f(x)$ n'a pas de maximum ou de minimum. + \end{enumerate} +\end{solution} + + + +%\printsolutionstype{exercise} + + + +\end{document} + +%%% Local Variables: +%%% mode: latex +%%% TeX-master: "master" +%%% End: diff --git a/TST/DM/2102_DM2/TST3/corr_14_2102_DM2.tex b/TST/DM/2102_DM2/TST3/corr_14_2102_DM2.tex new file mode 100644 index 0000000..e59ce3f --- /dev/null +++ b/TST/DM/2102_DM2/TST3/corr_14_2102_DM2.tex @@ -0,0 +1,203 @@ +\documentclass[a5paper,10pt]{article} +\usepackage{myXsim} +\usepackage{tasks} + +% Title Page +\title{DM2 \hfill MAGRO Robin} +\tribe{TST} +\date{\hfillÀ render pour le Mercredi 24 février} + +\xsimsetup{ + solution/print = true +} + +\begin{document} +\maketitle + +\begin{exercise}[subtitle={Loi binomiale}] + Trois personnes s'apprêtent à passer le portique de sécurité. On suppose que pour chaque personne la probabilité que le portique sonne est égale à $0.59$. + + Soit $X$ la variable aléatoire donnant le nombre de personnes faisant sonner le portique, parmi les 3 personnes de ce groupe. + \begin{enumerate} + \item Tracer l'arbre représentant le situation. + \item Justifier que $X$ suit une loi binomiale dont on précisera les paramètres. + \item Quelle est la probabilité qu'une seule personne fasse sonner le portique? + \item Calculer puis interpréter les probabilités suivantes + \[ + P(X = 0) \qquad \qquad P(X \geq 2) + \] + \item Calculer l'espérance de $X$ et interpréter le résultat. + \end{enumerate} +\end{exercise} + +\begin{solution} + \begin{enumerate} + \item + \begin{tikzpicture}[sloped] + \node {.} + child {node {$0$} + child {node {$0$} + child {node {$0$} + edge from parent + node[above] {0.41} + } + child {node {$1$} + edge from parent + node[above] {0.59} + } + edge from parent + node[above] {0.41} + } + child[missing] {} + child {node {$1$} + child {node {$0$} + edge from parent + node[above] {0.41} + } + child {node {$1$} + edge from parent + node[above] {0.59} + } + edge from parent + node[above] {0.41} + } + edge from parent + node[above] {0.41} + } + child[missing] {} + child[missing] {} + child[missing] {} + child { node {$1$} + child {node {$0$} + child {node {$0$} + edge from parent + node[above] {0.41} + } + child {node {$1$} + edge from parent + node[above] {0.59} + } + edge from parent + node[above] {0.41} + } + child[missing] {} + child {node {$1$} + child {node {$0$} + edge from parent + node[above] {0.41} + } + child {node {$1$} + edge from parent + node[above] {0.59} + } + edge from parent + node[above] {0.41} + } + edge from parent + node[above] {0.59} + } ; + \end{tikzpicture} + \item Chaque personne a 2 possibilités (1: fait sonner ou 2: ne fait pas sonner) et l'on fait passer 3 personnes ce qui correspond à une répétition identique et aléatoire. On peut donc modéliser la situation par une loi binomiale. + \[ + X \sim \mathcal{B}(3; 0.76) + \] + \item Probabilité qu'une seule personne fasse sonner le portique. On voit qu'il y a 3 branches qui correspondent à cette situation dont + \[ + P(X = 1) = 3 \times 0.59^1 \times 0.41^2 \approx 0.298 + \] + \item + \[ + P(X = 0) = 0.41^3 \approx 0.069 + \] + \[ + P(X \geq 2) = P(X = 2) + P(X = 3) = 3 \times 0.59^2 \times 0.41^1 + 0.59^3 \approx 0.633 + \] + + \item Il faut d'abord tracer le tableau résumant la loi de probabilité: + \begin{center} + \begin{tabular}{|c|*{4}{c|}} + \hline + Valeur & 0 & 1 & 2 & 3 \\ + \hline + Probabilité & $0.069$ & $0.298$ & $0.428$ &$0.205$ \\ + \hline + \end{tabular} + \end{center} + On peut alors calculer l'espérance + \[ + E[X] = 0 \times 0.069 + 1 \times 0.298 + 2 \times 0.428 + 3 \times 0.205 = 1.77 + \] + On peut donc estimer qu'il y aura en moyenne $1.77$ personnes qui feront sonner le portique sur les 3 personnes. + \end{enumerate} +\end{solution} + +\begin{exercise}[subtitle={Équation puissance}] + Résoudre les équations et inéquations suivantes + \begin{multicols}{2} + \begin{enumerate} + \item $10^x = 5$ + \item $15^x = 1$ + \item $0.57^x \leq 45$ + \item $3 \times 0.51^x = 21$ + \end{enumerate} + \end{multicols} +\end{exercise} + +\begin{solution} + Les solutions ci-dessous ne sont pas justifiée car l'ordinateur ne sait pas faire. Par contre, vous vous devez savoir justifier vos réponses! + \begin{enumerate} + \item $x = \log(5)$ + \item $x = \frac{\log(1)}{\log(15)}$ + \item Il faut faire attention quand on divise par un log car ce dernier peut être négatif ce qui est le cas ici. Il faut donc pense à changer le sens de l'inégalité. + + $x \geq \frac{\log(45)}{\log(0.57)}$ + + \item Il faut penser à faire la division à par $3$ avant d'utiliser le log car sinon, on ne peut pas utiliser la formule $\log(a^n) = n\times \log(a)$. + + $x = \frac{\log(7.0)}{\log(0.51)}$ + \end{enumerate} +\end{solution} + +\begin{exercise}[subtitle={Étude de fonctions}] + Soit $f(x) = - x^3 + 43.5x^2 + 510x - 37$ une fonction définie sur $\R$. + \begin{enumerate} + \item Calculer $f'(x)$ la dérivée de $f(x)$. + \item Calculer $f'(34)$ et $f'(-5)$. + \item En déduire une forme factorisée de $f'(x)$. + \item Étudier le signe de $f'(x)$ et en déduire les variations de $f(x)$. + \item Est-ce que la fonction $f(x)$ admet un maximum ou un minimum? Si oui, calculer sa valeur. + \end{enumerate} +\end{exercise} + +\begin{solution} + \begin{enumerate} + \item Dérivée de $f(x)$: $f'(x) = - 3x^2 + 87x + 510$ + \item + \begin{align*} + f'(34) &= - 3 \times 34^{2} + 87 \times 34 + 510\\&= - 3 \times 1156 + 2958 + 510\\&= - 3468 + 3468\\&= 0 + \end{align*} + \begin{align*} + f'(-5) &= - 3 \times - 5^{2} + 87(- 5) + 510\\&= - 3 \times 25 - 435 + 510\\&= - 75 + 75\\&= 0 + \end{align*} + Donc $x = 34$ et $x=-5$ sont des racines de $f'(x) = - 3x^2 + 87x + 510$. + \item On en déduit la forme factorisée suivante + \[ + f'(x) = -3 (x - 34)(x--5) + \] + \item Pas de correction disponible + \item À causes des branches extérieurs, la fonction $f(x)$ n'a pas de maximum ou de minimum. + \end{enumerate} +\end{solution} + + + +%\printsolutionstype{exercise} + + + +\end{document} + +%%% Local Variables: +%%% mode: latex +%%% TeX-master: "master" +%%% End: diff --git a/TST/DM/2102_DM2/TST3/corr_15_2102_DM2.tex b/TST/DM/2102_DM2/TST3/corr_15_2102_DM2.tex new file mode 100644 index 0000000..e9faf6a --- /dev/null +++ b/TST/DM/2102_DM2/TST3/corr_15_2102_DM2.tex @@ -0,0 +1,203 @@ +\documentclass[a5paper,10pt]{article} +\usepackage{myXsim} +\usepackage{tasks} + +% Title Page +\title{DM2 \hfill MORFIN Chloé} +\tribe{TST} +\date{\hfillÀ render pour le Mercredi 24 février} + +\xsimsetup{ + solution/print = true +} + +\begin{document} +\maketitle + +\begin{exercise}[subtitle={Loi binomiale}] + Trois personnes s'apprêtent à passer le portique de sécurité. On suppose que pour chaque personne la probabilité que le portique sonne est égale à $0.96$. + + Soit $X$ la variable aléatoire donnant le nombre de personnes faisant sonner le portique, parmi les 3 personnes de ce groupe. + \begin{enumerate} + \item Tracer l'arbre représentant le situation. + \item Justifier que $X$ suit une loi binomiale dont on précisera les paramètres. + \item Quelle est la probabilité qu'une seule personne fasse sonner le portique? + \item Calculer puis interpréter les probabilités suivantes + \[ + P(X = 0) \qquad \qquad P(X \geq 2) + \] + \item Calculer l'espérance de $X$ et interpréter le résultat. + \end{enumerate} +\end{exercise} + +\begin{solution} + \begin{enumerate} + \item + \begin{tikzpicture}[sloped] + \node {.} + child {node {$0$} + child {node {$0$} + child {node {$0$} + edge from parent + node[above] {0.04} + } + child {node {$1$} + edge from parent + node[above] {0.96} + } + edge from parent + node[above] {0.04} + } + child[missing] {} + child {node {$1$} + child {node {$0$} + edge from parent + node[above] {0.04} + } + child {node {$1$} + edge from parent + node[above] {0.96} + } + edge from parent + node[above] {0.04} + } + edge from parent + node[above] {0.04} + } + child[missing] {} + child[missing] {} + child[missing] {} + child { node {$1$} + child {node {$0$} + child {node {$0$} + edge from parent + node[above] {0.04} + } + child {node {$1$} + edge from parent + node[above] {0.96} + } + edge from parent + node[above] {0.04} + } + child[missing] {} + child {node {$1$} + child {node {$0$} + edge from parent + node[above] {0.04} + } + child {node {$1$} + edge from parent + node[above] {0.96} + } + edge from parent + node[above] {0.04} + } + edge from parent + node[above] {0.96} + } ; + \end{tikzpicture} + \item Chaque personne a 2 possibilités (1: fait sonner ou 2: ne fait pas sonner) et l'on fait passer 3 personnes ce qui correspond à une répétition identique et aléatoire. On peut donc modéliser la situation par une loi binomiale. + \[ + X \sim \mathcal{B}(3; 0.76) + \] + \item Probabilité qu'une seule personne fasse sonner le portique. On voit qu'il y a 3 branches qui correspondent à cette situation dont + \[ + P(X = 1) = 3 \times 0.96^1 \times 0.04^2 \approx 0.005 + \] + \item + \[ + P(X = 0) = 0.04^3 \approx 0.0 + \] + \[ + P(X \geq 2) = P(X = 2) + P(X = 3) = 3 \times 0.96^2 \times 0.04^1 + 0.96^3 \approx 0.996 + \] + + \item Il faut d'abord tracer le tableau résumant la loi de probabilité: + \begin{center} + \begin{tabular}{|c|*{4}{c|}} + \hline + Valeur & 0 & 1 & 2 & 3 \\ + \hline + Probabilité & $0.0$ & $0.005$ & $0.111$ &$0.885$ \\ + \hline + \end{tabular} + \end{center} + On peut alors calculer l'espérance + \[ + E[X] = 0 \times 0.0 + 1 \times 0.005 + 2 \times 0.111 + 3 \times 0.885 = 2.88 + \] + On peut donc estimer qu'il y aura en moyenne $2.88$ personnes qui feront sonner le portique sur les 3 personnes. + \end{enumerate} +\end{solution} + +\begin{exercise}[subtitle={Équation puissance}] + Résoudre les équations et inéquations suivantes + \begin{multicols}{2} + \begin{enumerate} + \item $10^x = 17$ + \item $3^x = 31$ + \item $0.44^x \leq 45$ + \item $9 \times 1.0^x = 17$ + \end{enumerate} + \end{multicols} +\end{exercise} + +\begin{solution} + Les solutions ci-dessous ne sont pas justifiée car l'ordinateur ne sait pas faire. Par contre, vous vous devez savoir justifier vos réponses! + \begin{enumerate} + \item $x = \log(17)$ + \item $x = \frac{\log(31)}{\log(3)}$ + \item Il faut faire attention quand on divise par un log car ce dernier peut être négatif ce qui est le cas ici. Il faut donc pense à changer le sens de l'inégalité. + + $x \geq \frac{\log(45)}{\log(0.44)}$ + + \item Il faut penser à faire la division à par $9$ avant d'utiliser le log car sinon, on ne peut pas utiliser la formule $\log(a^n) = n\times \log(a)$. + + $x = \frac{\log(1.89)}{\log(1.0)}$ + \end{enumerate} +\end{solution} + +\begin{exercise}[subtitle={Étude de fonctions}] + Soit $f(x) = - 8x^3 + 324x^2 + 1392x - 9$ une fonction définie sur $\R$. + \begin{enumerate} + \item Calculer $f'(x)$ la dérivée de $f(x)$. + \item Calculer $f'(29)$ et $f'(-2)$. + \item En déduire une forme factorisée de $f'(x)$. + \item Étudier le signe de $f'(x)$ et en déduire les variations de $f(x)$. + \item Est-ce que la fonction $f(x)$ admet un maximum ou un minimum? Si oui, calculer sa valeur. + \end{enumerate} +\end{exercise} + +\begin{solution} + \begin{enumerate} + \item Dérivée de $f(x)$: $f'(x) = - 24x^2 + 648x + 1392$ + \item + \begin{align*} + f'(29) &= - 24 \times 29^{2} + 648 \times 29 + 1392\\&= - 24 \times 841 + 18792 + 1392\\&= - 20184 + 20184\\&= 0 + \end{align*} + \begin{align*} + f'(-2) &= - 24 \times - 2^{2} + 648(- 2) + 1392\\&= - 24 \times 4 - 1296 + 1392\\&= - 96 + 96\\&= 0 + \end{align*} + Donc $x = 29$ et $x=-2$ sont des racines de $f'(x) = - 24x^2 + 648x + 1392$. + \item On en déduit la forme factorisée suivante + \[ + f'(x) = -24 (x - 29)(x--2) + \] + \item Pas de correction disponible + \item À causes des branches extérieurs, la fonction $f(x)$ n'a pas de maximum ou de minimum. + \end{enumerate} +\end{solution} + + + +%\printsolutionstype{exercise} + + + +\end{document} + +%%% Local Variables: +%%% mode: latex +%%% TeX-master: "master" +%%% End: diff --git a/TST/DM/2102_DM2/TST3/corr_16_2102_DM2.tex b/TST/DM/2102_DM2/TST3/corr_16_2102_DM2.tex new file mode 100644 index 0000000..8bf5443 --- /dev/null +++ b/TST/DM/2102_DM2/TST3/corr_16_2102_DM2.tex @@ -0,0 +1,203 @@ +\documentclass[a5paper,10pt]{article} +\usepackage{myXsim} +\usepackage{tasks} + +% Title Page +\title{DM2 \hfill PERES RAMALHO Emeric} +\tribe{TST} +\date{\hfillÀ render pour le Mercredi 24 février} + +\xsimsetup{ + solution/print = true +} + +\begin{document} +\maketitle + +\begin{exercise}[subtitle={Loi binomiale}] + Trois personnes s'apprêtent à passer le portique de sécurité. On suppose que pour chaque personne la probabilité que le portique sonne est égale à $0.19$. + + Soit $X$ la variable aléatoire donnant le nombre de personnes faisant sonner le portique, parmi les 3 personnes de ce groupe. + \begin{enumerate} + \item Tracer l'arbre représentant le situation. + \item Justifier que $X$ suit une loi binomiale dont on précisera les paramètres. + \item Quelle est la probabilité qu'une seule personne fasse sonner le portique? + \item Calculer puis interpréter les probabilités suivantes + \[ + P(X = 0) \qquad \qquad P(X \geq 2) + \] + \item Calculer l'espérance de $X$ et interpréter le résultat. + \end{enumerate} +\end{exercise} + +\begin{solution} + \begin{enumerate} + \item + \begin{tikzpicture}[sloped] + \node {.} + child {node {$0$} + child {node {$0$} + child {node {$0$} + edge from parent + node[above] {0.81} + } + child {node {$1$} + edge from parent + node[above] {0.19} + } + edge from parent + node[above] {0.81} + } + child[missing] {} + child {node {$1$} + child {node {$0$} + edge from parent + node[above] {0.81} + } + child {node {$1$} + edge from parent + node[above] {0.19} + } + edge from parent + node[above] {0.81} + } + edge from parent + node[above] {0.81} + } + child[missing] {} + child[missing] {} + child[missing] {} + child { node {$1$} + child {node {$0$} + child {node {$0$} + edge from parent + node[above] {0.81} + } + child {node {$1$} + edge from parent + node[above] {0.19} + } + edge from parent + node[above] {0.81} + } + child[missing] {} + child {node {$1$} + child {node {$0$} + edge from parent + node[above] {0.81} + } + child {node {$1$} + edge from parent + node[above] {0.19} + } + edge from parent + node[above] {0.81} + } + edge from parent + node[above] {0.19} + } ; + \end{tikzpicture} + \item Chaque personne a 2 possibilités (1: fait sonner ou 2: ne fait pas sonner) et l'on fait passer 3 personnes ce qui correspond à une répétition identique et aléatoire. On peut donc modéliser la situation par une loi binomiale. + \[ + X \sim \mathcal{B}(3; 0.76) + \] + \item Probabilité qu'une seule personne fasse sonner le portique. On voit qu'il y a 3 branches qui correspondent à cette situation dont + \[ + P(X = 1) = 3 \times 0.19^1 \times 0.81^2 \approx 0.374 + \] + \item + \[ + P(X = 0) = 0.81^3 \approx 0.531 + \] + \[ + P(X \geq 2) = P(X = 2) + P(X = 3) = 3 \times 0.19^2 \times 0.81^1 + 0.19^3 \approx 0.095 + \] + + \item Il faut d'abord tracer le tableau résumant la loi de probabilité: + \begin{center} + \begin{tabular}{|c|*{4}{c|}} + \hline + Valeur & 0 & 1 & 2 & 3 \\ + \hline + Probabilité & $0.531$ & $0.374$ & $0.088$ &$0.007$ \\ + \hline + \end{tabular} + \end{center} + On peut alors calculer l'espérance + \[ + E[X] = 0 \times 0.531 + 1 \times 0.374 + 2 \times 0.088 + 3 \times 0.007 = 0.57 + \] + On peut donc estimer qu'il y aura en moyenne $0.57$ personnes qui feront sonner le portique sur les 3 personnes. + \end{enumerate} +\end{solution} + +\begin{exercise}[subtitle={Équation puissance}] + Résoudre les équations et inéquations suivantes + \begin{multicols}{2} + \begin{enumerate} + \item $10^x = 5$ + \item $4^x = 39$ + \item $0.95^x \leq 21$ + \item $10 \times 0.74^x = 16$ + \end{enumerate} + \end{multicols} +\end{exercise} + +\begin{solution} + Les solutions ci-dessous ne sont pas justifiée car l'ordinateur ne sait pas faire. Par contre, vous vous devez savoir justifier vos réponses! + \begin{enumerate} + \item $x = \log(5)$ + \item $x = \frac{\log(39)}{\log(4)}$ + \item Il faut faire attention quand on divise par un log car ce dernier peut être négatif ce qui est le cas ici. Il faut donc pense à changer le sens de l'inégalité. + + $x \geq \frac{\log(21)}{\log(0.95)}$ + + \item Il faut penser à faire la division à par $10$ avant d'utiliser le log car sinon, on ne peut pas utiliser la formule $\log(a^n) = n\times \log(a)$. + + $x = \frac{\log(1.6)}{\log(0.74)}$ + \end{enumerate} +\end{solution} + +\begin{exercise}[subtitle={Étude de fonctions}] + Soit $f(x) = - 3x^3 + 108x^2 - 396x - 26$ une fonction définie sur $\R$. + \begin{enumerate} + \item Calculer $f'(x)$ la dérivée de $f(x)$. + \item Calculer $f'(22)$ et $f'(2)$. + \item En déduire une forme factorisée de $f'(x)$. + \item Étudier le signe de $f'(x)$ et en déduire les variations de $f(x)$. + \item Est-ce que la fonction $f(x)$ admet un maximum ou un minimum? Si oui, calculer sa valeur. + \end{enumerate} +\end{exercise} + +\begin{solution} + \begin{enumerate} + \item Dérivée de $f(x)$: $f'(x) = - 9x^2 + 216x - 396$ + \item + \begin{align*} + f'(22) &= - 9 \times 22^{2} + 216 \times 22 - 396\\&= - 9 \times 484 + 4752 - 396\\&= - 4356 + 4356\\&= 0 + \end{align*} + \begin{align*} + f'(2) &= - 9 \times 2^{2} + 216 \times 2 - 396\\&= - 9 \times 4 + 432 - 396\\&= - 36 + 36\\&= 0 + \end{align*} + Donc $x = 22$ et $x=2$ sont des racines de $f'(x) = - 9x^2 + 216x - 396$. + \item On en déduit la forme factorisée suivante + \[ + f'(x) = -9 (x - 22)(x-2) + \] + \item Pas de correction disponible + \item À causes des branches extérieurs, la fonction $f(x)$ n'a pas de maximum ou de minimum. + \end{enumerate} +\end{solution} + + + +%\printsolutionstype{exercise} + + + +\end{document} + +%%% Local Variables: +%%% mode: latex +%%% TeX-master: "master" +%%% End: diff --git a/TST/DM/2102_DM2/TST3/corr_17_2102_DM2.tex b/TST/DM/2102_DM2/TST3/corr_17_2102_DM2.tex new file mode 100644 index 0000000..c2e3265 --- /dev/null +++ b/TST/DM/2102_DM2/TST3/corr_17_2102_DM2.tex @@ -0,0 +1,203 @@ +\documentclass[a5paper,10pt]{article} +\usepackage{myXsim} +\usepackage{tasks} + +% Title Page +\title{DM2 \hfill RADOUAA Saleh} +\tribe{TST} +\date{\hfillÀ render pour le Mercredi 24 février} + +\xsimsetup{ + solution/print = true +} + +\begin{document} +\maketitle + +\begin{exercise}[subtitle={Loi binomiale}] + Trois personnes s'apprêtent à passer le portique de sécurité. On suppose que pour chaque personne la probabilité que le portique sonne est égale à $0.08$. + + Soit $X$ la variable aléatoire donnant le nombre de personnes faisant sonner le portique, parmi les 3 personnes de ce groupe. + \begin{enumerate} + \item Tracer l'arbre représentant le situation. + \item Justifier que $X$ suit une loi binomiale dont on précisera les paramètres. + \item Quelle est la probabilité qu'une seule personne fasse sonner le portique? + \item Calculer puis interpréter les probabilités suivantes + \[ + P(X = 0) \qquad \qquad P(X \geq 2) + \] + \item Calculer l'espérance de $X$ et interpréter le résultat. + \end{enumerate} +\end{exercise} + +\begin{solution} + \begin{enumerate} + \item + \begin{tikzpicture}[sloped] + \node {.} + child {node {$0$} + child {node {$0$} + child {node {$0$} + edge from parent + node[above] {0.92} + } + child {node {$1$} + edge from parent + node[above] {0.08} + } + edge from parent + node[above] {0.92} + } + child[missing] {} + child {node {$1$} + child {node {$0$} + edge from parent + node[above] {0.92} + } + child {node {$1$} + edge from parent + node[above] {0.08} + } + edge from parent + node[above] {0.92} + } + edge from parent + node[above] {0.92} + } + child[missing] {} + child[missing] {} + child[missing] {} + child { node {$1$} + child {node {$0$} + child {node {$0$} + edge from parent + node[above] {0.92} + } + child {node {$1$} + edge from parent + node[above] {0.08} + } + edge from parent + node[above] {0.92} + } + child[missing] {} + child {node {$1$} + child {node {$0$} + edge from parent + node[above] {0.92} + } + child {node {$1$} + edge from parent + node[above] {0.08} + } + edge from parent + node[above] {0.92} + } + edge from parent + node[above] {0.08} + } ; + \end{tikzpicture} + \item Chaque personne a 2 possibilités (1: fait sonner ou 2: ne fait pas sonner) et l'on fait passer 3 personnes ce qui correspond à une répétition identique et aléatoire. On peut donc modéliser la situation par une loi binomiale. + \[ + X \sim \mathcal{B}(3; 0.76) + \] + \item Probabilité qu'une seule personne fasse sonner le portique. On voit qu'il y a 3 branches qui correspondent à cette situation dont + \[ + P(X = 1) = 3 \times 0.08^1 \times 0.92^2 \approx 0.203 + \] + \item + \[ + P(X = 0) = 0.92^3 \approx 0.779 + \] + \[ + P(X \geq 2) = P(X = 2) + P(X = 3) = 3 \times 0.08^2 \times 0.92^1 + 0.08^3 \approx 0.019 + \] + + \item Il faut d'abord tracer le tableau résumant la loi de probabilité: + \begin{center} + \begin{tabular}{|c|*{4}{c|}} + \hline + Valeur & 0 & 1 & 2 & 3 \\ + \hline + Probabilité & $0.779$ & $0.203$ & $0.018$ &$0.001$ \\ + \hline + \end{tabular} + \end{center} + On peut alors calculer l'espérance + \[ + E[X] = 0 \times 0.779 + 1 \times 0.203 + 2 \times 0.018 + 3 \times 0.001 = 0.24 + \] + On peut donc estimer qu'il y aura en moyenne $0.24$ personnes qui feront sonner le portique sur les 3 personnes. + \end{enumerate} +\end{solution} + +\begin{exercise}[subtitle={Équation puissance}] + Résoudre les équations et inéquations suivantes + \begin{multicols}{2} + \begin{enumerate} + \item $10^x = 8$ + \item $17^x = 11$ + \item $0.84^x \leq 28$ + \item $8 \times 0.96^x = 22$ + \end{enumerate} + \end{multicols} +\end{exercise} + +\begin{solution} + Les solutions ci-dessous ne sont pas justifiée car l'ordinateur ne sait pas faire. Par contre, vous vous devez savoir justifier vos réponses! + \begin{enumerate} + \item $x = \log(8)$ + \item $x = \frac{\log(11)}{\log(17)}$ + \item Il faut faire attention quand on divise par un log car ce dernier peut être négatif ce qui est le cas ici. Il faut donc pense à changer le sens de l'inégalité. + + $x \geq \frac{\log(28)}{\log(0.84)}$ + + \item Il faut penser à faire la division à par $8$ avant d'utiliser le log car sinon, on ne peut pas utiliser la formule $\log(a^n) = n\times \log(a)$. + + $x = \frac{\log(2.75)}{\log(0.96)}$ + \end{enumerate} +\end{solution} + +\begin{exercise}[subtitle={Étude de fonctions}] + Soit $f(x) = 3x^3 - 207x^2 + 1161x + 46$ une fonction définie sur $\R$. + \begin{enumerate} + \item Calculer $f'(x)$ la dérivée de $f(x)$. + \item Calculer $f'(43)$ et $f'(3)$. + \item En déduire une forme factorisée de $f'(x)$. + \item Étudier le signe de $f'(x)$ et en déduire les variations de $f(x)$. + \item Est-ce que la fonction $f(x)$ admet un maximum ou un minimum? Si oui, calculer sa valeur. + \end{enumerate} +\end{exercise} + +\begin{solution} + \begin{enumerate} + \item Dérivée de $f(x)$: $f'(x) = 9x^2 - 414x + 1161$ + \item + \begin{align*} + f'(43) &= 9 \times 43^{2} - 414 \times 43 + 1161\\&= 9 \times 1849 - 17802 + 1161\\&= 16641 - 16641\\&= 0 + \end{align*} + \begin{align*} + f'(3) &= 9 \times 3^{2} - 414 \times 3 + 1161\\&= 9 \times 9 - 1242 + 1161\\&= 81 - 81\\&= 0 + \end{align*} + Donc $x = 43$ et $x=3$ sont des racines de $f'(x) = 9x^2 - 414x + 1161$. + \item On en déduit la forme factorisée suivante + \[ + f'(x) = 9 (x - 43)(x-3) + \] + \item Pas de correction disponible + \item À causes des branches extérieurs, la fonction $f(x)$ n'a pas de maximum ou de minimum. + \end{enumerate} +\end{solution} + + + +%\printsolutionstype{exercise} + + + +\end{document} + +%%% Local Variables: +%%% mode: latex +%%% TeX-master: "master" +%%% End: diff --git a/TST/DM/2102_DM2/TST3/corr_18_2102_DM2.tex b/TST/DM/2102_DM2/TST3/corr_18_2102_DM2.tex new file mode 100644 index 0000000..44def92 --- /dev/null +++ b/TST/DM/2102_DM2/TST3/corr_18_2102_DM2.tex @@ -0,0 +1,203 @@ +\documentclass[a5paper,10pt]{article} +\usepackage{myXsim} +\usepackage{tasks} + +% Title Page +\title{DM2 \hfill TAY Ummuhan} +\tribe{TST} +\date{\hfillÀ render pour le Mercredi 24 février} + +\xsimsetup{ + solution/print = true +} + +\begin{document} +\maketitle + +\begin{exercise}[subtitle={Loi binomiale}] + Trois personnes s'apprêtent à passer le portique de sécurité. On suppose que pour chaque personne la probabilité que le portique sonne est égale à $0.72$. + + Soit $X$ la variable aléatoire donnant le nombre de personnes faisant sonner le portique, parmi les 3 personnes de ce groupe. + \begin{enumerate} + \item Tracer l'arbre représentant le situation. + \item Justifier que $X$ suit une loi binomiale dont on précisera les paramètres. + \item Quelle est la probabilité qu'une seule personne fasse sonner le portique? + \item Calculer puis interpréter les probabilités suivantes + \[ + P(X = 0) \qquad \qquad P(X \geq 2) + \] + \item Calculer l'espérance de $X$ et interpréter le résultat. + \end{enumerate} +\end{exercise} + +\begin{solution} + \begin{enumerate} + \item + \begin{tikzpicture}[sloped] + \node {.} + child {node {$0$} + child {node {$0$} + child {node {$0$} + edge from parent + node[above] {0.28} + } + child {node {$1$} + edge from parent + node[above] {0.72} + } + edge from parent + node[above] {0.28} + } + child[missing] {} + child {node {$1$} + child {node {$0$} + edge from parent + node[above] {0.28} + } + child {node {$1$} + edge from parent + node[above] {0.72} + } + edge from parent + node[above] {0.28} + } + edge from parent + node[above] {0.28} + } + child[missing] {} + child[missing] {} + child[missing] {} + child { node {$1$} + child {node {$0$} + child {node {$0$} + edge from parent + node[above] {0.28} + } + child {node {$1$} + edge from parent + node[above] {0.72} + } + edge from parent + node[above] {0.28} + } + child[missing] {} + child {node {$1$} + child {node {$0$} + edge from parent + node[above] {0.28} + } + child {node {$1$} + edge from parent + node[above] {0.72} + } + edge from parent + node[above] {0.28} + } + edge from parent + node[above] {0.72} + } ; + \end{tikzpicture} + \item Chaque personne a 2 possibilités (1: fait sonner ou 2: ne fait pas sonner) et l'on fait passer 3 personnes ce qui correspond à une répétition identique et aléatoire. On peut donc modéliser la situation par une loi binomiale. + \[ + X \sim \mathcal{B}(3; 0.76) + \] + \item Probabilité qu'une seule personne fasse sonner le portique. On voit qu'il y a 3 branches qui correspondent à cette situation dont + \[ + P(X = 1) = 3 \times 0.72^1 \times 0.28^2 \approx 0.169 + \] + \item + \[ + P(X = 0) = 0.28^3 \approx 0.022 + \] + \[ + P(X \geq 2) = P(X = 2) + P(X = 3) = 3 \times 0.72^2 \times 0.28^1 + 0.72^3 \approx 0.808 + \] + + \item Il faut d'abord tracer le tableau résumant la loi de probabilité: + \begin{center} + \begin{tabular}{|c|*{4}{c|}} + \hline + Valeur & 0 & 1 & 2 & 3 \\ + \hline + Probabilité & $0.022$ & $0.169$ & $0.435$ &$0.373$ \\ + \hline + \end{tabular} + \end{center} + On peut alors calculer l'espérance + \[ + E[X] = 0 \times 0.022 + 1 \times 0.169 + 2 \times 0.435 + 3 \times 0.373 = 2.16 + \] + On peut donc estimer qu'il y aura en moyenne $2.16$ personnes qui feront sonner le portique sur les 3 personnes. + \end{enumerate} +\end{solution} + +\begin{exercise}[subtitle={Équation puissance}] + Résoudre les équations et inéquations suivantes + \begin{multicols}{2} + \begin{enumerate} + \item $10^x = 41$ + \item $2^x = 7$ + \item $0.44^x \leq 20$ + \item $3 \times 0.08^x = 24$ + \end{enumerate} + \end{multicols} +\end{exercise} + +\begin{solution} + Les solutions ci-dessous ne sont pas justifiée car l'ordinateur ne sait pas faire. Par contre, vous vous devez savoir justifier vos réponses! + \begin{enumerate} + \item $x = \log(41)$ + \item $x = \frac{\log(7)}{\log(2)}$ + \item Il faut faire attention quand on divise par un log car ce dernier peut être négatif ce qui est le cas ici. Il faut donc pense à changer le sens de l'inégalité. + + $x \geq \frac{\log(20)}{\log(0.44)}$ + + \item Il faut penser à faire la division à par $3$ avant d'utiliser le log car sinon, on ne peut pas utiliser la formule $\log(a^n) = n\times \log(a)$. + + $x = \frac{\log(8.0)}{\log(0.08)}$ + \end{enumerate} +\end{solution} + +\begin{exercise}[subtitle={Étude de fonctions}] + Soit $f(x) = 7x^3 - 472.5x^2 + 10374x + 15$ une fonction définie sur $\R$. + \begin{enumerate} + \item Calculer $f'(x)$ la dérivée de $f(x)$. + \item Calculer $f'(26)$ et $f'(19)$. + \item En déduire une forme factorisée de $f'(x)$. + \item Étudier le signe de $f'(x)$ et en déduire les variations de $f(x)$. + \item Est-ce que la fonction $f(x)$ admet un maximum ou un minimum? Si oui, calculer sa valeur. + \end{enumerate} +\end{exercise} + +\begin{solution} + \begin{enumerate} + \item Dérivée de $f(x)$: $f'(x) = 21x^2 - 945x + 10374$ + \item + \begin{align*} + f'(26) &= 21 \times 26^{2} - 945 \times 26 + 10374\\&= 21 \times 676 - 24570 + 10374\\&= 14196 - 14196\\&= 0 + \end{align*} + \begin{align*} + f'(19) &= 21 \times 19^{2} - 945 \times 19 + 10374\\&= 21 \times 361 - 17955 + 10374\\&= 7581 - 7581\\&= 0 + \end{align*} + Donc $x = 26$ et $x=19$ sont des racines de $f'(x) = 21x^2 - 945x + 10374$. + \item On en déduit la forme factorisée suivante + \[ + f'(x) = 21 (x - 26)(x-19) + \] + \item Pas de correction disponible + \item À causes des branches extérieurs, la fonction $f(x)$ n'a pas de maximum ou de minimum. + \end{enumerate} +\end{solution} + + + +%\printsolutionstype{exercise} + + + +\end{document} + +%%% Local Variables: +%%% mode: latex +%%% TeX-master: "master" +%%% End: diff --git a/TST/DM/2102_DM2/TST3/corr_19_2102_DM2.tex b/TST/DM/2102_DM2/TST3/corr_19_2102_DM2.tex new file mode 100644 index 0000000..634dca2 --- /dev/null +++ b/TST/DM/2102_DM2/TST3/corr_19_2102_DM2.tex @@ -0,0 +1,203 @@ +\documentclass[a5paper,10pt]{article} +\usepackage{myXsim} +\usepackage{tasks} + +% Title Page +\title{DM2 \hfill VIALON-DUPERRON Victorien} +\tribe{TST} +\date{\hfillÀ render pour le Mercredi 24 février} + +\xsimsetup{ + solution/print = true +} + +\begin{document} +\maketitle + +\begin{exercise}[subtitle={Loi binomiale}] + Trois personnes s'apprêtent à passer le portique de sécurité. On suppose que pour chaque personne la probabilité que le portique sonne est égale à $0.18$. + + Soit $X$ la variable aléatoire donnant le nombre de personnes faisant sonner le portique, parmi les 3 personnes de ce groupe. + \begin{enumerate} + \item Tracer l'arbre représentant le situation. + \item Justifier que $X$ suit une loi binomiale dont on précisera les paramètres. + \item Quelle est la probabilité qu'une seule personne fasse sonner le portique? + \item Calculer puis interpréter les probabilités suivantes + \[ + P(X = 0) \qquad \qquad P(X \geq 2) + \] + \item Calculer l'espérance de $X$ et interpréter le résultat. + \end{enumerate} +\end{exercise} + +\begin{solution} + \begin{enumerate} + \item + \begin{tikzpicture}[sloped] + \node {.} + child {node {$0$} + child {node {$0$} + child {node {$0$} + edge from parent + node[above] {0.82} + } + child {node {$1$} + edge from parent + node[above] {0.18} + } + edge from parent + node[above] {0.82} + } + child[missing] {} + child {node {$1$} + child {node {$0$} + edge from parent + node[above] {0.82} + } + child {node {$1$} + edge from parent + node[above] {0.18} + } + edge from parent + node[above] {0.82} + } + edge from parent + node[above] {0.82} + } + child[missing] {} + child[missing] {} + child[missing] {} + child { node {$1$} + child {node {$0$} + child {node {$0$} + edge from parent + node[above] {0.82} + } + child {node {$1$} + edge from parent + node[above] {0.18} + } + edge from parent + node[above] {0.82} + } + child[missing] {} + child {node {$1$} + child {node {$0$} + edge from parent + node[above] {0.82} + } + child {node {$1$} + edge from parent + node[above] {0.18} + } + edge from parent + node[above] {0.82} + } + edge from parent + node[above] {0.18} + } ; + \end{tikzpicture} + \item Chaque personne a 2 possibilités (1: fait sonner ou 2: ne fait pas sonner) et l'on fait passer 3 personnes ce qui correspond à une répétition identique et aléatoire. On peut donc modéliser la situation par une loi binomiale. + \[ + X \sim \mathcal{B}(3; 0.76) + \] + \item Probabilité qu'une seule personne fasse sonner le portique. On voit qu'il y a 3 branches qui correspondent à cette situation dont + \[ + P(X = 1) = 3 \times 0.18^1 \times 0.82^2 \approx 0.363 + \] + \item + \[ + P(X = 0) = 0.82^3 \approx 0.551 + \] + \[ + P(X \geq 2) = P(X = 2) + P(X = 3) = 3 \times 0.18^2 \times 0.82^1 + 0.18^3 \approx 0.086 + \] + + \item Il faut d'abord tracer le tableau résumant la loi de probabilité: + \begin{center} + \begin{tabular}{|c|*{4}{c|}} + \hline + Valeur & 0 & 1 & 2 & 3 \\ + \hline + Probabilité & $0.551$ & $0.363$ & $0.08$ &$0.006$ \\ + \hline + \end{tabular} + \end{center} + On peut alors calculer l'espérance + \[ + E[X] = 0 \times 0.551 + 1 \times 0.363 + 2 \times 0.08 + 3 \times 0.006 = 0.54 + \] + On peut donc estimer qu'il y aura en moyenne $0.54$ personnes qui feront sonner le portique sur les 3 personnes. + \end{enumerate} +\end{solution} + +\begin{exercise}[subtitle={Équation puissance}] + Résoudre les équations et inéquations suivantes + \begin{multicols}{2} + \begin{enumerate} + \item $10^x = 22$ + \item $4^x = 6$ + \item $0.01^x \leq 35$ + \item $10 \times 0.36^x = 19$ + \end{enumerate} + \end{multicols} +\end{exercise} + +\begin{solution} + Les solutions ci-dessous ne sont pas justifiée car l'ordinateur ne sait pas faire. Par contre, vous vous devez savoir justifier vos réponses! + \begin{enumerate} + \item $x = \log(22)$ + \item $x = \frac{\log(6)}{\log(4)}$ + \item Il faut faire attention quand on divise par un log car ce dernier peut être négatif ce qui est le cas ici. Il faut donc pense à changer le sens de l'inégalité. + + $x \geq \frac{\log(35)}{\log(0.01)}$ + + \item Il faut penser à faire la division à par $10$ avant d'utiliser le log car sinon, on ne peut pas utiliser la formule $\log(a^n) = n\times \log(a)$. + + $x = \frac{\log(1.9)}{\log(0.36)}$ + \end{enumerate} +\end{solution} + +\begin{exercise}[subtitle={Étude de fonctions}] + Soit $f(x) = - 4x^3 + 72x^2 + 1296x - 22$ une fonction définie sur $\R$. + \begin{enumerate} + \item Calculer $f'(x)$ la dérivée de $f(x)$. + \item Calculer $f'(18)$ et $f'(-6)$. + \item En déduire une forme factorisée de $f'(x)$. + \item Étudier le signe de $f'(x)$ et en déduire les variations de $f(x)$. + \item Est-ce que la fonction $f(x)$ admet un maximum ou un minimum? Si oui, calculer sa valeur. + \end{enumerate} +\end{exercise} + +\begin{solution} + \begin{enumerate} + \item Dérivée de $f(x)$: $f'(x) = - 12x^2 + 144x + 1296$ + \item + \begin{align*} + f'(18) &= - 12 \times 18^{2} + 144 \times 18 + 1296\\&= - 12 \times 324 + 2592 + 1296\\&= - 3888 + 3888\\&= 0 + \end{align*} + \begin{align*} + f'(-6) &= - 12 \times - 6^{2} + 144(- 6) + 1296\\&= - 12 \times 36 - 864 + 1296\\&= - 432 + 432\\&= 0 + \end{align*} + Donc $x = 18$ et $x=-6$ sont des racines de $f'(x) = - 12x^2 + 144x + 1296$. + \item On en déduit la forme factorisée suivante + \[ + f'(x) = -12 (x - 18)(x--6) + \] + \item Pas de correction disponible + \item À causes des branches extérieurs, la fonction $f(x)$ n'a pas de maximum ou de minimum. + \end{enumerate} +\end{solution} + + + +%\printsolutionstype{exercise} + + + +\end{document} + +%%% Local Variables: +%%% mode: latex +%%% TeX-master: "master" +%%% End: diff --git a/TST/DM/2102_DM2/TST3/corr_20_2102_DM2.tex b/TST/DM/2102_DM2/TST3/corr_20_2102_DM2.tex new file mode 100644 index 0000000..0a90620 --- /dev/null +++ b/TST/DM/2102_DM2/TST3/corr_20_2102_DM2.tex @@ -0,0 +1,203 @@ +\documentclass[a5paper,10pt]{article} +\usepackage{myXsim} +\usepackage{tasks} + +% Title Page +\title{DM2 \hfill ZENAGUI Yanis} +\tribe{TST} +\date{\hfillÀ render pour le Mercredi 24 février} + +\xsimsetup{ + solution/print = true +} + +\begin{document} +\maketitle + +\begin{exercise}[subtitle={Loi binomiale}] + Trois personnes s'apprêtent à passer le portique de sécurité. On suppose que pour chaque personne la probabilité que le portique sonne est égale à $0.23$. + + Soit $X$ la variable aléatoire donnant le nombre de personnes faisant sonner le portique, parmi les 3 personnes de ce groupe. + \begin{enumerate} + \item Tracer l'arbre représentant le situation. + \item Justifier que $X$ suit une loi binomiale dont on précisera les paramètres. + \item Quelle est la probabilité qu'une seule personne fasse sonner le portique? + \item Calculer puis interpréter les probabilités suivantes + \[ + P(X = 0) \qquad \qquad P(X \geq 2) + \] + \item Calculer l'espérance de $X$ et interpréter le résultat. + \end{enumerate} +\end{exercise} + +\begin{solution} + \begin{enumerate} + \item + \begin{tikzpicture}[sloped] + \node {.} + child {node {$0$} + child {node {$0$} + child {node {$0$} + edge from parent + node[above] {0.77} + } + child {node {$1$} + edge from parent + node[above] {0.23} + } + edge from parent + node[above] {0.77} + } + child[missing] {} + child {node {$1$} + child {node {$0$} + edge from parent + node[above] {0.77} + } + child {node {$1$} + edge from parent + node[above] {0.23} + } + edge from parent + node[above] {0.77} + } + edge from parent + node[above] {0.77} + } + child[missing] {} + child[missing] {} + child[missing] {} + child { node {$1$} + child {node {$0$} + child {node {$0$} + edge from parent + node[above] {0.77} + } + child {node {$1$} + edge from parent + node[above] {0.23} + } + edge from parent + node[above] {0.77} + } + child[missing] {} + child {node {$1$} + child {node {$0$} + edge from parent + node[above] {0.77} + } + child {node {$1$} + edge from parent + node[above] {0.23} + } + edge from parent + node[above] {0.77} + } + edge from parent + node[above] {0.23} + } ; + \end{tikzpicture} + \item Chaque personne a 2 possibilités (1: fait sonner ou 2: ne fait pas sonner) et l'on fait passer 3 personnes ce qui correspond à une répétition identique et aléatoire. On peut donc modéliser la situation par une loi binomiale. + \[ + X \sim \mathcal{B}(3; 0.76) + \] + \item Probabilité qu'une seule personne fasse sonner le portique. On voit qu'il y a 3 branches qui correspondent à cette situation dont + \[ + P(X = 1) = 3 \times 0.23^1 \times 0.77^2 \approx 0.409 + \] + \item + \[ + P(X = 0) = 0.77^3 \approx 0.457 + \] + \[ + P(X \geq 2) = P(X = 2) + P(X = 3) = 3 \times 0.23^2 \times 0.77^1 + 0.23^3 \approx 0.134 + \] + + \item Il faut d'abord tracer le tableau résumant la loi de probabilité: + \begin{center} + \begin{tabular}{|c|*{4}{c|}} + \hline + Valeur & 0 & 1 & 2 & 3 \\ + \hline + Probabilité & $0.457$ & $0.409$ & $0.122$ &$0.012$ \\ + \hline + \end{tabular} + \end{center} + On peut alors calculer l'espérance + \[ + E[X] = 0 \times 0.457 + 1 \times 0.409 + 2 \times 0.122 + 3 \times 0.012 = 0.69 + \] + On peut donc estimer qu'il y aura en moyenne $0.69$ personnes qui feront sonner le portique sur les 3 personnes. + \end{enumerate} +\end{solution} + +\begin{exercise}[subtitle={Équation puissance}] + Résoudre les équations et inéquations suivantes + \begin{multicols}{2} + \begin{enumerate} + \item $10^x = 15$ + \item $7^x = 38$ + \item $0.66^x \leq 14$ + \item $9 \times 0.36^x = 21$ + \end{enumerate} + \end{multicols} +\end{exercise} + +\begin{solution} + Les solutions ci-dessous ne sont pas justifiée car l'ordinateur ne sait pas faire. Par contre, vous vous devez savoir justifier vos réponses! + \begin{enumerate} + \item $x = \log(15)$ + \item $x = \frac{\log(38)}{\log(7)}$ + \item Il faut faire attention quand on divise par un log car ce dernier peut être négatif ce qui est le cas ici. Il faut donc pense à changer le sens de l'inégalité. + + $x \geq \frac{\log(14)}{\log(0.66)}$ + + \item Il faut penser à faire la division à par $9$ avant d'utiliser le log car sinon, on ne peut pas utiliser la formule $\log(a^n) = n\times \log(a)$. + + $x = \frac{\log(2.33)}{\log(0.36)}$ + \end{enumerate} +\end{solution} + +\begin{exercise}[subtitle={Étude de fonctions}] + Soit $f(x) = - 5x^3 + 375x^2 - 4515x + 9$ une fonction définie sur $\R$. + \begin{enumerate} + \item Calculer $f'(x)$ la dérivée de $f(x)$. + \item Calculer $f'(43)$ et $f'(7)$. + \item En déduire une forme factorisée de $f'(x)$. + \item Étudier le signe de $f'(x)$ et en déduire les variations de $f(x)$. + \item Est-ce que la fonction $f(x)$ admet un maximum ou un minimum? Si oui, calculer sa valeur. + \end{enumerate} +\end{exercise} + +\begin{solution} + \begin{enumerate} + \item Dérivée de $f(x)$: $f'(x) = - 15x^2 + 750x - 4515$ + \item + \begin{align*} + f'(43) &= - 15 \times 43^{2} + 750 \times 43 - 4515\\&= - 15 \times 1849 + 32250 - 4515\\&= - 27735 + 27735\\&= 0 + \end{align*} + \begin{align*} + f'(7) &= - 15 \times 7^{2} + 750 \times 7 - 4515\\&= - 15 \times 49 + 5250 - 4515\\&= - 735 + 735\\&= 0 + \end{align*} + Donc $x = 43$ et $x=7$ sont des racines de $f'(x) = - 15x^2 + 750x - 4515$. + \item On en déduit la forme factorisée suivante + \[ + f'(x) = -15 (x - 43)(x-7) + \] + \item Pas de correction disponible + \item À causes des branches extérieurs, la fonction $f(x)$ n'a pas de maximum ou de minimum. + \end{enumerate} +\end{solution} + + + +%\printsolutionstype{exercise} + + + +\end{document} + +%%% Local Variables: +%%% mode: latex +%%% TeX-master: "master" +%%% End: diff --git a/TST/DM/2102_DM2/TST3/corr_all_2102_DM2.pdf b/TST/DM/2102_DM2/TST3/corr_all_2102_DM2.pdf new file mode 100644 index 0000000..8284bb0 Binary files /dev/null and b/TST/DM/2102_DM2/TST3/corr_all_2102_DM2.pdf differ diff --git a/TST/DM/2102_DM2/TST3/tpl_2102_DM2.tex b/TST/DM/2102_DM2/TST3/tpl_2102_DM2.tex new file mode 100644 index 0000000..4226e5c --- /dev/null +++ b/TST/DM/2102_DM2/TST3/tpl_2102_DM2.tex @@ -0,0 +1,226 @@ +\documentclass[a5paper,10pt]{article} +\usepackage{myXsim} +\usepackage{tasks} + +% Title Page +\title{DM2 \hfill \Var{Nom}} +\tribe{TST} +\date{\hfillÀ render pour le Mercredi 24 février} + +\xsimsetup{ + solution/print = false +} + +\begin{document} +\maketitle + +\begin{exercise}[subtitle={Loi binomiale}] + %- set p = round(random(), 2) + Trois personnes s'apprêtent à passer le portique de sécurité. On suppose que pour chaque personne la probabilité que le portique sonne est égale à $\Var{p}$. + + Soit $X$ la variable aléatoire donnant le nombre de personnes faisant sonner le portique, parmi les 3 personnes de ce groupe. + \begin{enumerate} + \item Tracer l'arbre représentant le situation. + \item Justifier que $X$ suit une loi binomiale dont on précisera les paramètres. + \item Quelle est la probabilité qu'une seule personne fasse sonner le portique? + \item Calculer puis interpréter les probabilités suivantes + \[ + P(X = 0) \qquad \qquad P(X \geq 2) + \] + \item Calculer l'espérance de $X$ et interpréter le résultat. + \end{enumerate} +\end{exercise} + +\begin{solution} + %- set q = round(1-p, 2) + \begin{enumerate} + \item + \begin{tikzpicture}[sloped] + \node {.} + child {node {$0$} + child {node {$0$} + child {node {$0$} + edge from parent + node[above] {\Var{q}} + } + child {node {$1$} + edge from parent + node[above] {\Var{p}} + } + edge from parent + node[above] {\Var{q}} + } + child[missing] {} + child {node {$1$} + child {node {$0$} + edge from parent + node[above] {\Var{q}} + } + child {node {$1$} + edge from parent + node[above] {\Var{p}} + } + edge from parent + node[above] {\Var{q}} + } + edge from parent + node[above] {\Var{q}} + } + child[missing] {} + child[missing] {} + child[missing] {} + child { node {$1$} + child {node {$0$} + child {node {$0$} + edge from parent + node[above] {\Var{q}} + } + child {node {$1$} + edge from parent + node[above] {\Var{p}} + } + edge from parent + node[above] {\Var{q}} + } + child[missing] {} + child {node {$1$} + child {node {$0$} + edge from parent + node[above] {\Var{q}} + } + child {node {$1$} + edge from parent + node[above] {\Var{p}} + } + edge from parent + node[above] {\Var{q}} + } + edge from parent + node[above] {\Var{p}} + } ; + \end{tikzpicture} + \item Chaque personne a 2 possibilités (1: fait sonner ou 2: ne fait pas sonner) et l'on fait passer 3 personnes ce qui correspond à une répétition identique et aléatoire. On peut donc modéliser la situation par une loi binomiale. + \[ + X \sim \mathcal{B}(3; \Var{0.76}) + \] + %- set p0 = round(q**3, 3) + %- set p1 = round(3*p*q**2, 3) + %- set p2 = round(3*p**2*q, 3) + %- set p3 = round(p**3, 3) + \item Probabilité qu'une seule personne fasse sonner le portique. On voit qu'il y a 3 branches qui correspondent à cette situation dont + \[ + P(X = 1) = 3 \times \Var{p}^1 \times \Var{q}^2 \approx \Var{p1} + \] + \item + \[ + P(X = 0) = \Var{q}^3 \approx \Var{p0} + \] + \[ + P(X \geq 2) = P(X = 2) + P(X = 3) = 3 \times \Var{p}^2 \times \Var{q}^1 + \Var{p}^3 \approx \Var{round(p2 + p3, 3)} + \] + + \item Il faut d'abord tracer le tableau résumant la loi de probabilité: + \begin{center} + \begin{tabular}{|c|*{4}{c|}} + \hline + Valeur & 0 & 1 & 2 & 3 \\ + \hline + Probabilité & $\Var{p0}$ & $\Var{p1}$ & $\Var{p2}$ &$\Var{p3}$ \\ + \hline + \end{tabular} + \end{center} + On peut alors calculer l'espérance + %- set E = round(3*p, 3) + \[ + E[X] = 0 \times \Var{p0} + 1 \times \Var{p1} + 2 \times \Var{p2} + 3 \times \Var{p3} = \Var{E} + \] + On peut donc estimer qu'il y aura en moyenne $\Var{E}$ personnes qui feront sonner le portique sur les 3 personnes. + \end{enumerate} +\end{solution} + +\begin{exercise}[subtitle={Équation puissance}] + Résoudre les équations et inéquations suivantes + \begin{multicols}{2} + \begin{enumerate} + %- set a1 = randint(1, 50) + \item $10^x = \Var{a1}$ + %- set a2 = randint(1, 50) + %- set b2 = randint(2, 20) + \item $\Var{b2}^x = \Var{a2}$ + %- set a3 = randint(2, 50) + %- set b3 = round(random(), 2) + \item $\Var{b3}^x \leq \Var{a3}$ + %- set a4 = randint(2, 50) + %- set b4 = round(random(), 2) + %- set c4 = randint(2, 10) + \item $\Var{c4} \times \Var{b4}^x = \Var{a4}$ + \end{enumerate} + \end{multicols} +\end{exercise} + +\begin{solution} + Les solutions ci-dessous ne sont pas justifiée car l'ordinateur ne sait pas faire. Par contre, vous vous devez savoir justifier vos réponses! + \begin{enumerate} + \item $x = \log(\Var{a1})$ + \item $x = \frac{\log(\Var{a2})}{\log(\Var{b2})}$ + \item Il faut faire attention quand on divise par un log car ce dernier peut être négatif ce qui est le cas ici. Il faut donc pense à changer le sens de l'inégalité. + + $x \geq \frac{\log(\Var{a3})}{\log(\Var{b3})}$ + + \item Il faut penser à faire la division à par $\Var{c4}$ avant d'utiliser le log car sinon, on ne peut pas utiliser la formule $\log(a^n) = n\times \log(a)$. + + $x = \frac{\log(\Var{round(a4/c4, 2)})}{\log(\Var{b4})}$ + \end{enumerate} +\end{solution} + +\begin{exercise}[subtitle={Étude de fonctions}] + %- set a_ = randint(-10, 10) + %- set a = a_*3 + %- set x1 = randint(0, 50) + %- set x2 = randint(-20, 20) + %- set b = randint(-50, 50) + %- set f = Polynom.from_coefficients([b, a*x1*x2, -a*(x1+x2)/2, a_]) + Soit $f(x) = \Var{f}$ une fonction définie sur $\R$. + \begin{enumerate} + \item Calculer $f'(x)$ la dérivée de $f(x)$. + \item Calculer $f'(\Var{x1})$ et $f'(\Var{x2})$. + \item En déduire une forme factorisée de $f'(x)$. + \item Étudier le signe de $f'(x)$ et en déduire les variations de $f(x)$. + \item Est-ce que la fonction $f(x)$ admet un maximum ou un minimum? Si oui, calculer sa valeur. + \end{enumerate} +\end{exercise} + +\begin{solution} + \begin{enumerate} + %- set fp = f.differentiate() + \item Dérivée de $f(x)$: $f'(x) = \Var{fp}$ + \item + \begin{align*} + f'(\Var{x1}) &= \Var{fp(x1).explain() |join('\\\\&= ')} + \end{align*} + \begin{align*} + f'(\Var{x2}) &= \Var{fp(x2).explain() |join('\\\\&= ')} + \end{align*} + Donc $x = \Var{x1}$ et $x=\Var{x2}$ sont des racines de $f'(x) = \Var{fp}$. + \item On en déduit la forme factorisée suivante + \[ + f'(x) = \Var{a} (x - \Var{x1})(x-\Var{x2}) + \] + \item Pas de correction disponible + \item À causes des branches extérieurs, la fonction $f(x)$ n'a pas de maximum ou de minimum. + \end{enumerate} +\end{solution} + + + +%\printsolutionstype{exercise} + + + +\end{document} + +%%% Local Variables: +%%% mode: latex +%%% TeX-master: "master" +%%% End: +